SlideShare a Scribd company logo
ĐẠI HOC THÁI NGUYÊN
TRƯ NG ĐẠI HOC KHOA HOC
Tải tài liệu tại sividoc.com
Viết đề tài giá sinh viên – ZALO:0973.287.149-TEAMLUANVAN.COM
BÙI HữU MÊN
M T SO L P PHƯƠNG TRÌNH DIOPHANTINE
LU N VĂN THẠC SĨ TOÁN HOC
Thái Nguyên - 2017
ĐẠI HOC THÁI NGUYÊN
TRƯ NG ĐẠI HOC KHOA HOC
Tải tài liệu tại sividoc.com
Viết đề tài giá sinh viên – ZALO:0973.287.149-TEAMLUANVAN.COM
BÙI HữU MÊN
M T SO L P PHƯƠNG TRÌNH DIOPHANTINE
LU N VĂN THẠC SĨ TOÁN HOC
Chuyên ngành: Phương pháp Toán sơ cap
Mã so: 60 46 01 13
NGƯŐI HƯŐNG DAN KHOA HOC
GS.TSKH. Đ NG HÙNG THANG
Thái Nguyên - 2017
3
Viết đề tài giá sinh viên – ZALO:0973.287.149-TEAMLUANVAN.COM
Mục lục
Danh sách kí hi u 4
MƠ đau 5
Chương 1. Phương trình Diophantine tuyen tính 7
1.1 Phương trình b c nhat hai an . . . . . . . . . . . . . . . . . . . . . . . 8
1.2 Phương trình b c nhat nhieu an....................................................................... 15
Chương 2. M t so phương trình Diophantine phi tuyen 23
2.1 Phương trình Pell loại 1 .................................................................................. 23
2.2 Phương trình Pell loại 2 .................................................................................. 30
2.3 Phương trình Pythagoras................................................................................. 38
Chương 3. Liên phân so và ng dụng trong phương trình Diophantine 45
3.1 Liên phân so hữu hạn...................................................................................... 45
3.2 Liên phân so vô hạn........................................................................................ 49
3.3 Liên phân so vô hạn tuan hoàn ....................................................................... 50
3.4 Áp dụng vào phương trình Diophante ............................................................ 56
3.4.1 Phương trình b c nhat hai an Ax+By = C......................................... 56
3.4.2 Phương trình x2
− dy2
= ±1 ....................................................................57
Ket lu n 62
Tài li u tham khao 63
4
Viết đề tài giá sinh viên – ZALO:0973.287.149-TEAMLUANVAN.COM
Danh sách kí hi u
N t p hop các so tụ nhiên
Z vành các so nguyên
Q trưòng các so hữu tý
R trưòng các so thục
C trưòng các so phức
Fp trưòng có p phan tử
K[X] vành đa thức vói h so trên trưòng K
[x| tran của so x
degP(X) b c của đa thức P(X)
mod p modulo p
gcd(P(X),Q(X)) ưóc chung lón nhat của hai đa thức P(X) và Q(X)
5
Viết đề tài giá sinh viên – ZALO:0973.287.149-TEAMLUANVAN.COM
MƠ đau
Phương trình Diophantine là m t chủ đe lón của Lý thuyet so, chứa đụng nhieu lý
thuyet toán hoc sâu sac, gan lien vói nhieu tên tuoi của nhieu nhà toán hoc xuat
sac. Mục tiêu của đe tài lu n văn là: Tìm hieu m t so lóp phương trình Diophantine
như: phương trình Diophantine tuyen tính; m t so phương trình Diophantine phi
tuyen (phương trình Pell, phương trình Pell mỏ r ng, phương trình Pythagoras
Fermat). Liên phân so và ứng dụng trong phương trình Diophantine. Ve m t ứng
dụng, lu n văn sẽ áp dụng lý thuyet đe soi sáng những bài toán so hoc ỏ pho thông,
h thong hóa, tong quát hóa và sáng tác ra những bài toán so hoc mói.
Lu n văn sẽ co gang trỏ thành m t tài li u tham khảo tot, thiet thục phục vụ
cho vi c giảng dạy, nhat là vi c giảng dạy và boi dưõng hoc sinh giỏi. Ngoài ra
thông qua vi c viet lu n văn, tác giả lu n văn có cơ h i mỏ r ng nâng cao hieu biet
ve toán sơ cap nói chung và so hoc nói riêng, hình thành các kỹ năng chứng minh
các định lí so hoc và giải các bài toán so hoc, phục vụ tot cho vi c giảng dạy môn
Toán ỏ trưòng pho thông.
N i dung của lu n văn đưoc trình bày trong ba chương như sau:
• Chương 1. Phương trình Diophantine tuyen tính. Trong chương này chúng
tôi trình bày ve phương trình b c nhat hai an, nhieu an, và m t so bài toán
chon loc.
• Chương 2. M t so phương trình Diophantine phi tuyen. Trong chương này
chúng tôi trình bày n i dung chính ve các phương trình Pell loại 1, phương
trình Pell loại , và phương trình Pythagoras.
• Chương 3. Liên phân so và ŕng dnng trong phương trình Diophantine. Trong
6
Viết đề tài giá sinh viên – ZALO:0973.287.149-TEAMLUANVAN.COM
chương này chúng tôi trình bày m t cách ngan gon các sụ ki n ve liên phân
so, đ c bi t là các ứng dụng của chúng đe giải phương trình Pell.
Lu n văn này đưoc thục hi n tại Trưòng Đại hoc Khoa hoc - Đại hoc Thái
Nguyên và hoàn thành vói sụ hưóng dȁn của GS.TSKH. Đ ng Hùng Thang (Trưòng
ĐHKHTN - ĐHQG Hà N i). Tác giả xin đưoc bày tỏ lòng biet ơn chân thành và
sâu sac tói ngưòi hưóng dȁn khoa hoc của mình, ngưòi đã đ t van đe nghiên cứu,
dành nhieu thòi gian hưóng dȁn và t n tình giải đáp những thac mac của tác giả
trong suot quá trình làm lu n văn.
Tác giả xin trân trong cảm ơn Ban Giám hi u Trưòng Đại hoc Khoa hoc - Đại
hoc Thái Nguyên, Ban Chủ nhi m Khoa Toán–Tin, cùng các giảng viên đã tham
gia giảng dạy, đã tạo moi đieu ki n tot nhat đe tác giả hoc t p và nghiên cứu.
Tác giả muon gửi những lòi cảm ơn tot đep nhat tói t p the lóp Cao hoc Toán
khóa 9 (2015-2017) đã đ ng viên và giúp đõ tác giả rat nhieu trong suot quá trình
hoc t p.
Nhân dịp này, tác giả cũng xin chân thành cảm ơn Sỏ Giáo dục và Đào tạo Hải
Phòng, Ban Giám hi u và các đong nghi p ỏ Trưòng THPT Thái Phiên đã tạo đieu
ki n cho tác giả hoàn thành tot nhi m vụ hoc t p và công tác của mình.
Cuoi cùng, tác giả muon dành những lòi cảm ơn đ c bi t nhat đen đại gia đình
vì những đ ng viên và chia sẻ những khó khăn đe tác giả hoàn thành lu n văn này.
Thái Nguyên, ngày 10 tháng 11 năm 2017
Tác giả
Bùi H u Mên
7
Viết đề tài giá sinh viên – ZALO:0973.287.149-TEAMLUANVAN.COM
Chương 1
Phương trình Diophantine tuyen tính
Phương trình Diophantine là m t trong những chủ đe sâu sac và rat r ng của Lý
thuyet so. Mục đích của chương này là nghiên cứu ve phương trình Diophantine
b c nhat hai và nhieu an. Như m t minh hoa cho lý thuyet, các ví dụ là các bài
toán trích từ các đe thi sẽ đưoc trình bày.
Đ c tính của các phương trình Diophantine là chúng có m t hay nhieu an so mà
moi h so đeu là so nguyên và chỉ yêu cau tìm các nghi m nguyên (ho c nguyên
dương). Nhà toán hoc noi tieng thòi co đại Diophantine đã có công lón vì những
nghiên cứu tiên phong ve chúng.
Vói m t phương trình Diophantine cho trưóc ta có the đ t ra các câu hỏi sau
đây (xep theo thứ tụ từ de đen khó):
Câu hói 1. Nó có nghi m nguyên hay không ?
Câu hói 2. Nó có m t so hữu hạn nghi m hay có vô so nghi m?
Câu hói 3. Hãy tìm tat cả các nghi m của nó.
Chȁng hạn, ta hãy xét phương trình Diophantine
xn
+yn
= zn
trong đó n là so nguyên dương lón hơn hay bang 2. Vói n = 2 phương trình trên
có vô so nghi m và ta có the tìm đưoc tưòng minh tat cả các nghi m của nó. Vói
n > 2, nhà toán hoc thiên tài của the ký 17 Pierre de Fermat khȁng định rang
8
Viết đề tài giá sinh viên – ZALO:0973.287.149-TEAMLUANVAN.COM
phương trình trên không có nghi m nguyên dương. Ket lu n này ngày nay đưoc
mang tên là Định lí lớn Fermat hay Định lí cuoi cùng của Fermat. Ngưòi ta đã
không tìm thay dau vet của chứng minh khȁng định trên của Fermat mà chỉ thay
m t ghi chú của Fermat bên le cuon sách “So hoc” của Diophantine: “Tôi đã tìm
đưoc m t chứng minh th t là tuy t vòi nhưng vì le sách ỏ đây quá hep nên không
the viet ra”.
Năm 1983, nhà toán hoc 29 tuoi ngưòi Đức là Faltings đã chứng minh thành
công m t giả thuyet của Mordell trong lĩnh vục Hình hoc đại so roi từ đó suy ra
rang phương trình xn
+ yn
= zn
vói n > 2 chỉ có m t so hữu hạn nghi m nguyên.
Vói thành tụu này Faltings đã nh n đưoc Giải thưỏng Fields (giải thưỏng quoc te
cao nhat dành cho các nhà toán hoc không quá 40 tuoi).
Năm 1993 nhà toán hoc ngưòi Anh là Andrew Wiles đã công bo phép chứng
minh của Định lí lón Fermat. Đây là m t câu chuy n lón của Toán hoc, có the tham
khảo trong Amir D. Aczel [1].
Vói sụ ra đòi của máy tính, ngưòi ta cũng đ t câu hỏi: Có ton tại chăng m t
thu t toán đe vói moi phương trình Diophantine cho trưóc nhò đó có the khȁng
định đưoc rang phương trình này có nghi m nguyên hay không. Tiec thay câu trả
lòi lại là: không có m t thu t toán như v y (Định lí Machiakevich).
1.1 Phương trình b c nhat hai an
Phương trình Diophantine đơn giản nhat là phương trình b c nhat hai an
ax + by = c (1.1)
trong đó a, b, c là những so nguyên cho trưóc khác 0. Van đe đ t ra là vói đieu ki n
nào của a, b, c thì phương trình (1.1) có nghi m và neu có thì cách tìm nghi m the
nào.
Định lí 1.1.1. Đieu ki n can và đủ đe phương trình (1.1) có nghi m nguyên là
9
Viết đề tài giá sinh viên – ZALO:0973.287.149-TEAMLUANVAN.COM
a
x = x0 +
b
t,
(a,b) là ước của c.
Chŕng minh. Đieu ki n can. Giả sử (x0,y0) là m t nghi m nguyên của (1.1). Khi
đó ax0 +by0 = c. Neu d = (a,b) thì rõ ràng d | c.
Đieu ki n đủ. Giả sử d = (a,b) và d | c. Ta có a = da1, b = db1, c = dc1 trong đó
(a1, b1) = 1. Phương trình (1.1) tương đương vói a1x + b1y = c1.
Xét a1 so {b1k} vói k = 0,1,2,...,a1 − 1. Vì (a1,b1) = 1 nên các so này khi
chia cho a1 sẽ cho ta các so dư khác nhau. V y tại k0, 0 ≤ k0 ≤ a1 − 1 sao cho
b1k0 = c1 (mod a1). Đieu này có nghĩa là:
c1 −b1k0 = a1l0 vói l0 ∈ Z hay c1 = a1l0 +b1k0.
V y (l0,k0) là m t nghi m của phương trình (1.1). Phép chứng minh định lí đưoc
hoàn thành.
Tiep theo ta hãy đi tìm tat cả các nghi m của phương trình (1.1)
Định lí 1.1.2. Neu (x0,y0) là m t nghi m nguyên của (1.1) thì nó có vô so nghi m
nguyên và nghi m nguyên (x,y) của nó được cho bới công thŕc
trong đó t ∈ Z và d = (a,b).
d
y = y0 −
d
t,
(1.2)
Chŕng minh. Trưóc het ta kiem tra moi c p so (x,y) cho bỏi công thức (1.2) là
nghi m. Th t v y
ax + by = ax0 + by0 = c.
Đảo lại, giả sử (x1,y1) là m t nghi m của (1.1) tức là ax1 +by1 = c. Trừ đȁng thức
này vào đȁng thức ax0 +by0 = c ta thu đưoc
a(x1 − x0) = b(y0 − y1). (1.3)
10
Viết đề tài giá sinh viên – ZALO:0973.287.149-TEAMLUANVAN.COM
d d
ki
Vì d = (a,b) nên a = a1d, b = b1d vói (a1,b1) = 1. Thay vào (1.3) ta đưoc a1(x1 −
x0) = b1(y0 − y1). Vì (a1,b1) = 1 nên y0 −y1 = ta1 và x1 −x0 = tb1. V y
y1 = y0 −ta1 = y0 −
at
và x1 = x0 +tb1 = x0 +
bt
.
Phép chứng minh đưoc ket thúc.
Thu t toán tìm nghi m cua phương trình Diophantine b c nhat. Từ Định lí
1.1.2 ta thay rang đe tìm tat cả các nghi m của (1.1) ta chỉ can tìm m t nghi m
(x0, y0) nào đó của nó. Ta goi m t nghi m cụ the như the là m t nghi m riêng còn
công thức (1.2) đưoc goi là nghi m tőng quát. Sau đây ta sẽ trình bày m t thu t
toán cho phép xác định khá nhanh m t nghi m riêng của (1.1).
Giả sử q0,q1,... là m t dãy các so nguyên dương. Vói mői i ≥ 0 ta kí hi u
[q0,q1,...,qi] là phân so sau đây
[q0,q1,...,qi] = q0 +
1
.
1
q1 +
1
q2 +··· +
1
qi−1 +
qi
Bang phương pháp quy nạp có the de dàng chứng minh đưoc bo đe sau:
Bo đe 1.1.3. Giá sr {hn}, {kn} là hai dãy so nguyên được xác định như sau:
h−2 = 0, h−1 = 1, h1 = qihi−1 +hi−2, i ≥ 0,
k0 = 1, k1 = q1, ki = qihki−1 +ki−2, i ≥ 2.
Khi đó với moi i ≥ 1 ta có:
(a) hiki−1 − hi−1ki = (−1)i−1
;
(b) [q0,q1,...,qi] = hi
.
11
Viết đề tài giá sinh viên – ZALO:0973.287.149-TEAMLUANVAN.COM
Bây giò cho hai so dương a, b vói a > b. Ta hãy viet thu t toán Euclid tìm ưóc
chung lón nhat của a và b.
Từ h (1.4) ta suy ra
a = bq0 +r1
b = r1q1 +r2
...
rn−1 = rnqn−1 + rn+1
rn = rn+1qn.
a
b
= [q0,q1,...,qn]
(1.4)
Từ khȁng định (b) của Bo đe 1.1.3 ta có
a hn
b
=
kn
.
Từ (a) ta suy ra (hn,kn) = 1. Do đó neu (a,b) = 1 thì a = hn và b = kn. V y thì
akn−1 − bhn−1 = (−1)n−1
.
Thành thử ton tại x0 ∈ {kn−1} và y0 ∈ {hn−1} sao cho
ax0 + by0 = 1.
Ta thử từng trưòng hop (nhieu nhat là bon phép thử) đe xác định x0, y0.
Như v y đe giải phương trình (1.1) ta sẽ tien hành lan lưot các bưóc sau đây.
Bưóc 1. Tìm d = (a,b) sau đó chia hai ve cho d đe đưoc m t phương trình tương
đương a1x+b1y = c1 , ỏ đó
a = da1, b = db1, c = dc1, (a1,b1) = 1.
Bưóc 2. Viet thu t toán Euclid cho hai so |a1| và |b1|. Giả sử |a1| > |b1|.
|a1| = |b1|q0 + r1
12
Viết đề tài giá sinh viên – ZALO:0973.287.149-TEAMLUANVAN.COM
k
|b1| = r1q1 + r2
...
rn−1 = rnqn−1 + rn+1
rn = rn+1qn.
Bưóc 3. Tính [q0,q1,...,qn−1] = h
.
Bưóc 4. Lay nghi m riêng (x0
′ ,y0
′ ) của phương trình a1x + b1y = 1 thoả mãn đieu
ki n |x0
′ | = k, |y′
0| = h.
Ta xác định dau của x0
′ và y′
0 bang cách thử.
Bưóc 5. Ta có x0 = c1x0
′ , y0 = c1y′
0 là nghi m riêng của phương trình (1.1). Khi
đó nghi m tong quát là
x = x0 + b1t, y = y0 + a1t, vói t ∈ Z.
Ví dụ 1.1.4. Giải phương trình Diophantine 342x − 123y = 15.
Lời giái. Ta sẽ làm lan lưot theo các bưóc như trên.
Bưóc 1. Úóc chung lón nhat của 342 và 123 là 3. phương trình đã cho tương đương
vói 114x −41y = 5.
Bưóc 2. Ta viet thu t toán Euclid cho 114 và 41.
114 = 41 · 2 + 32
41 = 32 · 1 + 9
32 = 9 · 3 + 5
9 = 5 · 1 + 4
5 = 4 · 1 + 1
4 = 1 · 4.
13
Viết đề tài giá sinh viên – ZALO:0973.287.149-TEAMLUANVAN.COM
Bưóc 3. Ta bieu dien theo liên phân so
h 1
k
= 2+
1
1 +
3 +
25
=
9
.
1
1
1 +
1
Như v y h = 25, k = 9.
Bưóc 4. Lay nghi m riêng x0
′ , y′
0 của phương trình 114x − 41y = 1 thoả mãn đieu
ki n |x0
′ | = 9 và |y′
0| = 25. Bang cách thử ta tìm đưoc x0
′
= 9, y′0 = 25.
Bưóc 5. Nghi m riêng x0, y0 của phương trình đã cho là x0 = 9 · 5 = 45, y0 =
25 · 5 = 125 và nghi m tong quát là
x = 45 + 41t, y = 125 + 114t, vói t ∈ Z.
Nh n xét 1.1.5.
1. Neu a | c ta có the lay nghi m riêng
x
c
0 =
a
, y0 = 0.
2. Xét trưòng hop (a,b) = 1. Theo Định lí Euler ta có
aϕ(b) − 1 = kb vói k ∈ Z.
V y caϕ(b) −bkc = c. Do đó
x0 = caϕ(b) − 1, y0 = −kc
là m t nghi m riêng của phương trình đang xét.
14
Viết đề tài giá sinh viên – ZALO:0973.287.149-TEAMLUANVAN.COM
b b
=
ams
=
ams
=
as as
M t so bài toán chon loc
Bài toán 1.1.6. Cho hai so nguyên dương a, b. Chŕng minh rang (a,b) = 1 khi và
chỉ khi ton tại các so nguyên dương u, v sao cho au−bv = 1.
Lời giái. Đieu ki n đủ là hien nhiên.
Đảo lại, giả sử (a,b) = 1. Theo Định lí 1.1.1, ton tại các so nguyên x0, y0 đe
cho ax0 +by0 = 1. Đ t
u = x0 + bt,
v = at − y0,
trong đó t ∈ Z đưoc chon sao cho
t > −
x0
, t >
y0
.
Khi đó u và v là các so nguyên dương và au − bv = ax0 + by0 = 1.
Bài toán 1.1.7. Giá sr (l,m) = 1 và al
= bm
, trong đó a,b,l,m ∈ N∗. Khi đó ton
tại n đe a = nm
, b = nl
.
Lời giái. Theo Bài toán 1.1.6 ton tại các so r, s ∈ N∗ đe lr − ms = 1. Ta có
lr−ms
alr bmr
br m
Suy ra
√
m
a = br
. Vì
√
m
a là m t so hữu tỉ nên nó phải là so nguyên. V y n = br
∈ N∗.
Suy ra a = nm
. Từ đó bm
= nl
= nml
, nên b = nl
. Bài toán đưoc chứng minh xong.
Bài toán 1.1.8. Cho a ∈ N∗. Hãy tìm g = (am
− 1,an
− 1).
Lời giái. Trưóc het xét trưòng hop (m,n) = 1. Vì
(a − 1) | am
− 1, (a −1)an
− 1
nên a − 1 là ưóc của g.
Đảo lại, ta sẽ chứng minh g cũng là ưóc của a − 1. Theo Bài toán 1.1.6 ton tại
các so u, v ∈ N∗ sao cho mu−nv = 1.
as
a = a .
15
Viết đề tài giá sinh viên – ZALO:0973.287.149-TEAMLUANVAN.COM
Vì g | am
− 1, g | an
− 1 nên cũng có g | amu
− 1, g | anv
− 1. Suy ra
g | amu
− anv
= anv
(amu−nv
− 1) = anv
(a − 1).
M t khác, de thay (g,a) = 1. V y g | (a−1).
Như v y neu (m,n) = 1 thì (am
−1,an
−1) = a−1.
Vói m, n bat kì, giả sử d = (m,n). Khi ay m = dm1, n = dn1 và (m1,n1) = 1.
Ta có
(am
− 1,an
− 1) = ((ad
)m1 −1, (ad
)n1 − 1) = ad
− 1.
Tóm lại ta có công thức
(am
−1,an
−1) = a(m,n)−1.
Bài toán 1.1.9. Cho (a, b) = 1 trong đó a, b ∈ N∗. Tìm giá trị c ∈ N∗ lớn nhat đe
phương trình ax+by = c không có nghi m nguyên dương.
Lời giái. Ta chứng minh rang c = ab là giá trị can tìm. Giả sử c > ab. Xét b so
a, 2a,..., ba. Vì (a, b) = 1 nên các so này khi chia cho b sẽ cho các so dư khác
nhau. V y ton tại so k, 1 ≤ k ≤ b, sao cho ka ≡ c (mod b). Suy ra c − ka = lb trong
đó l ∈ Z. Neu c > ab thì c > ka, do đó lb > 0 hay l ∈ N∗. Như v y (k, l) là nghi m
nguyên dương của phương trình ax +by = c.
M t khác, giả sử (x0,y0) là nghi m nguyên dương của phương trình ax +by =
ab. Khi đó ax0 = ab− by0 = b(a− y0).
Vì (a,b) = 1 nên từ đó suy ra x0 . b. Tương tụ, y0 . a. Như v y x0 ≥ b, y0 ≥ a,
do đó ab = ax0 + by0 ≥ 2ab. Vô lý. Phép chứng minh đưoc ket thúc.
1.2 Phương trình b c nhat nhieu an
Trong mục này ta mỏ r ng ket quả của mục trưóc bang cách xét phương trình
Diophantine b c nhat n an
a1x1 + a2x2 + ... + anxn = c. (1.5)
16
Viết đề tài giá sinh viên – ZALO:0973.287.149-TEAMLUANVAN.COM
b
trong đó a1,a2,...,an và c là các so nguyên cho trưóc, n ≥ 2.
Đoi vói bat kỳ m t phương trình nào, câu hỏi đau tiên là, trong những tình
huong nào của h so, ta có the khȁng định ve tính ton tại nghi m của nó. Ve sụ ton
tại nghi m của phương trình Diophantine b c nhat n an này ta có định lí sau đây.
Định lí 1.2.1. Đieu ki n can và đủ đe phương trình (1.5) có nghi m là
(a1,a2,...,an) | c.
Chŕng minh. Đieu ki n can là hien nhiên. Ta sẽ chứng minh đieu ki n đủ bang
phương pháp quy nạp, vói n = 1 đieu khȁng định là đúng do Định lí 1.1.1. Giả sử
định lí đúng vói n = 1, ta chứng minh nó đúng vói n. Kí hi u
bi =
ai
, c1 =
c
, ỏ đó d = (a1,a2,...,an).
d d
Phương trình (1.5) tương đương vói
b1x1 + b2x2 + ... + bnxn = c1. (1.6)
trong đó (b1,b2,...,bn) = 1. Đ t b = (b1,b2,...,bn−1). Ta có (b,bn) = 1. Theo
Định lí 1.1.1 ton tại so nguyên ln và k sao cho:
bnln +bk = c1. (1.7)
Kí hi u b′
i = bi
vói i = 1,2,...,n− 1. Ta có (b1
′ ,b′
2,...s,b′
n−1) = 1.
Theo giả thiet quy nạp ton tại các so nguyên l1,l2,...,ln−1 sao cho
b′
1l1 +b′
2l2 +...+b′
n−1ln−1 = k
hay
b1l1 +b2l2 +...+bn−1ln−1 = bk. (1.8)
Từ (1.7) và (1.8) ta suy ra
b1l1 + b2l2 + ... + bn−1ln−1 + bnln = c1
tức là (l1,l2,...,ln) là nghi m của phương trình (1.6). Định lí đưoc chứng minh.
17
Viết đề tài giá sinh viên – ZALO:0973.287.149-TEAMLUANVAN.COM
x2 +α +ant2 2
Chúng ta không đi sâu vào vi c tìm bieu thức cho nghi m tong quát của nó
như đã làm đoi vói trưòng hop n = 2. Tuy nhiên có the thay rang neu phương trình
(1.5) có nghi m nguyên α1,α2,...,αn thì nó sẽ có vô so nghi m nguyên phụ thu c
vào n − 1 tham so. Th t v y, de dàng kiem tra đưoc tat cả các b n so nguyên
x1,x2,...,xn xác định như sau là nghi m của (1.5),
x1 = α1 + ant1
...
xn−1 = αn−1 +antn−1
xn = αn −a1t1 −a2t2 ...−an−1tn−1
trong đó ti ∈ Z, i = 1,2,...n −1 đưoc chon tuỳ ý.
Bây giò, ta sẽ thảo lu n ve cách giải phương trình (1.5). Ve m t thục hành ta
có the tien hành theo hai cách sau đây.
Cách 1. Đưa (1.5) ve trưòng hop có m t h so bang 1.
Cách 2. Neu phương trình (1.5) có hai h so nguyên to cùng nhau, chȁng hạn
(a1,a2) = 1 thì ta viet dưói dạng
a1x1 + a2x2 = c − a3x3 − ... − anxn
roi giải phương trình theo hai an x1, x2.
Ta xét hai ví dụ sau đây, nó sẽ lan lưot minh hoa cho hai cách trên.
Ví dụ 1.2.2. Tìm tat cả các nghi m nguyên của phương trình
6x + 45y + 6z − 10t = 13.
Lời giái. Phương trình đã cho đưoc viet dưói dạng
6(x+z)+10(4y−t)+ 5y = 13.
18
Viết đề tài giá sinh viên – ZALO:0973.287.149-TEAMLUANVAN.COM
Đ t
x + z = x1, 4y −t = x2
ta đưoc 6x1 + 10x2 + 5y = 13. Suy ra
x1 +10x2 +5(y+x1) = 13.
Đ t x1 +y = x3 ta đưoc x1 +10x2 +5x3 = 13. V y
x1 = 13 − 10x2 − 5x3.
Từ đây, bang tính toán trục tiep ta có
y = x3 −x1 = x3 −(13−10x2 −5x3) = 10x2 +6x3 −13;
t = 4y−x2 = 39x2 +24x3 −52;
x = x1 − z = 13 − 10x2 − 5x3 − z.
V y nghi m tong quát của phương trình đã cho là
x = 13 − 10x2 − 5x3 − x4,
y = 10x2 + 6x3 − 13,
z = x4,
t = 39x2 + 24x3 − 52.
trong đó x2, x3, x4 là các so nguyên tuỳ ý.
Ví dụ 1.2.3. Giải phương trình
6x + 15y + 10z = 3. (1.9)
Lời giái. Ta có the viet (1.9) dưói dạng
6(x + z) + 15y = 3 − 4z.
Đ t u = x+z ta có 15y +4z = 3− 6u. Ta thay (−1,4) là nghi m riêng của 15y+
4z = 1. Do đó (−3+6u,12−24u) là nghi m riêng của 15y+4z = 3−6u. Suy ra
nghi m tong quát của nó là
y = −3 + 6u + 4t, z = 12 − 24u − 15t.
19
Viết đề tài giá sinh viên – ZALO:0973.287.149-TEAMLUANVAN.COM
−
Từ u = x + z suy ra
x = u − z = u − (12 − 24u − 15t) = −12 + 25u + 15t.
V y nghi m tong quát của (1.9) là
x = 12 + 25u + 15t,
y = −3 +6u + 4t,
z = 12 −24u− 15,
vói u, t ∈ Z.
M t so bài toán chon loc
Bài toán 1.2.4 (Đe thi Vô định Toán Quoc te 1983). Cho a, b, c là các so nguyên
đôi m t nguyên to cùng nhau. Chŕng minh rang 2abc − ab − bc − ca là so nguyên
lớn nhat không viet được dưới dạng xbc + yca + zab với x, y, z là nhrng so không
âm.
Lời giái. Bài toán tương đương vói vi c chứng minh rang
Khȁng định 1. phương trình xbc + yca + zab = 2abc − ab − bc − ca không có
nghi m nguyên không âm;
Khȁng định 2. Neu n > 2b − ab − bc − ca thì phương trình có nghi m nguyên
không âm.
Chŕng minh Khȁng định 1. Giả sử ton tại x0,y0,z0 ∈ N∗ sao cho
x0bc + y0ca + z0ab = 2abc − ab − bc − ca.
Đieu này tương đương vói
bc(x0 +1)+ ca(y0 +1) +ab(z0 + 1) = 2abc.
20
Viết đề tài giá sinh viên – ZALO:0973.287.149-TEAMLUANVAN.COM
y+ 2z+5u = b
Suy ra ab(z0 + 1) . c. Vì (ab,c) = 1 nên z0 + 1 . c. M t khác z0 + 1 ∈ N∗ nên
z0 + 1 ≥ c. Tương tụ y0 + 1 ≥ b, x0 + 1 ≥ a. V y
bc(x0 +1)+ ca(y0 +1) +ab(z0 + 1) ≥ 3abc.
Đieu này vô lí.
Chŕng minh Khȁng định 2. Xét ab có dạng kbc + lac, vói 0 ≤ k ≤ a − 1, 0 ≤ l ≤
b−1..
Các so này khi chia cho ab sẽ cho ta các so dư khác nhau. Th t v y, giả sử
k1bc +l1ac ≡ k2bc + l2ac (mod ab).
Vì (ab,c) = 1 nên suy ra b(k1 −k2)−a(l1 −l2) . ab. Từ đó b(k1 −k2) . a và a(l1 −
l2) . b. Vì (a,b) = 1 nên k1 −k2 . a và l1 −l2 . b. M t khác |k1 −k2| < a, |l1 −l2| < b
nên l1 −l2 = 0, k1 −k2 = 0. Do đó ton tại x0, y0 ∈ Z+, 0 ≤ x0 ≤ a−1, 0 ≤ y0 ≤ b−1
đe cho
bcx0 +acy0 ≡ n (mod ab).
V y ton tại z0 ∈ Z đe bcx0 + acy0 + abz0 = n. Ta có
ab(z0 +1) = n−bcx0 −acy0 +ab
> 2abc − bcx0 − acy0 − bc − ca
= bc(a− 1−x0)+ac(b−1− y0) ≥ 0.
Do đó z0 + 1 > 0. Suy ra z0 ≥ 0.
Bài toán 1.2.5. Cho các so nguyên không âm a, b thoá mãn đieu ki n 5a ≥ 7b.
Chŕng minh rang h
x+2y+ 3z+ 7u = a,
luôn có nghi m nguyên không âm.
21
Viết đề tài giá sinh viên – ZALO:0973.287.149-TEAMLUANVAN.COM
5
5
5
5
5
5
5
5
5
5
5
5
Chŕng minh. Giả sử b = 5u + v vói 0 ≤ u, 0 ≤ v ≤ 4. Từ đieu ki n a ≥ 7b
suy ra
a ≥
7(5u+v)
. Từ đây ta có
a − 7u ≥
7v
. (1.10)
Như v y h phương trình có the viet là
,
x+2y+3z = a−yu y+2z = b−5u = v. (1.11)
Vói mői 0 ≤ v ≤ 4 ta sẽ chon y, z thích hop đe có x = z − 7u − 2y − 3z ≥ 0.
• Neu v = 0, ta lay y = z = 0. Khi đó x = a − 7u ≥ 7v
= 0 (theo (1.10))).
• Neu v = 1, ta thay y = 1, z = 0. Khi đó
V y x ≥ 0.
x = a − yu − 2 ≥
7v
− 2 =
7
− 2 > −1.
• Neu v = 2, ta lay y = 0, z = 1. Khi đó
V y x ≥ 0.
x = a − 7u − 3 ≥
7v
− 3 =
14
− 3 > −1.
• Neu v = 3, ta lay y = z = 1. Khi đó
V y x ≥ 0.
x = a − 7u − 5 ≥
7v
− 5 =
21
− 5 > −1.
• Neu v = 4, ta lay y = 0, z = 2. Khi đó
V y x ≥ 0.
x = a − 7u − 6 ≥
7v
− 6 =
28
− 6 > −1.
Tóm lại vói moi v = 0, 1, 2, 3, 4 ta đeu chon đưoc y, z, x không âm đe (1.11) đưoc
nghi m đúng. Bài toán đưoc giải xong.
22
Viết đề tài giá sinh viên – ZALO:0973.287.149-TEAMLUANVAN.COM
5
5
2b 2b
Bài toán 1.2.6 (Đe thi vô địch Mỹ 1982). Chŕng minh rang ton tại so tự nhiên k
sao cho tat cá các so kn
+1 với n = 1,2,... đeu là hợp so.
Lời giái. Xét các so Fermat Fm = 22m
+1.
Ta đã biet Fm là so nguyên to vói m = 0,1,2,3,4, trong khi Euler phát hi u
rang F5 là m t hop so, F5 là tích của hai so nguyên to 641 và 6700417.
Ký hi u
ai = Fi vói i = 0,1,2,3,4,
a5 = 641a6 = 6700417.
Theo Định lí Trung Hoa ve th ng dư, ton tại so tụ nhiên k > max(a0,...,a6) đe
k ≡ 1 (mod am) vói m = 0,1,2,...,5 và k ≡ −1 (mod a6).
Ta sẽ chứng minh k chính là so can tìm. Xét n bat kỳ, n có the viet dưói dạng
n = 2m
p vói 0 ≤ m và p lẻ.
(a) Neu m ≤ 4, ta có k · 2n
+ 1 ≡ 2n + 1 (mod am).
M t khác 2n + 1 = 22m
p + 1 = (22m
)p + 1 . 22m
+ 1 = am
. Do đó k · 2n
+ 1 . am
.
Vì k ·2n
+1 > k > am
nên k ·2n
+1 là hop so.
(b) Neu m = 5 thì k · 2n
+ 1 ≡ 2n + 1 (mod a5).
M t khác 2n
+1 = 225
p
+1 . 225
+1 = F . 641 = a . V y k·2n
+1 . a . M t
5 5
khác k · 2n
+ 1 > a5, suy ra k · 2n
+ 1 là hop so.
(c) Neu m ≥ 6, khi đó n = 26
b vói b ∈ Z. Ta có
k · 2n
+ 1 ≡ −2n
+ 1 (mod a6),
2n
= 22
5 2b
= (F −1) ≡ (−1) ≡ 1 (mod a
6) vì F5 . a6.
Do đó k ·2n
+1 ≡ −1+1 = 0 (mod a6). Vì k · 2n
+1 > a6 nên k ·2n
+1 là hop
so.
V y vói moi n, so k · 2n
+ 1 luôn là hop so.
23
Viết đề tài giá sinh viên – ZALO:0973.287.149-TEAMLUANVAN.COM
.α − . < .
h 1
Chương 2
M t so phương trình Diophantine
phi tuyen
2.1 Phương trình Pell loại 1
Phương trình Pell loại 1 là phương trình có dạng
x2
− Dy2
= 1, (2.1)
trong đó D ∈ N∗ và ta yêu cau tìm nghi m x,y ∈ N∗ . Trong tiet này khi nói đen
nghi m của (2.1) ta hieu là nghi m nguyên dương.
Định lí 2.1.1 (Đieu ki n ton tại nghi m). Phương trình (2.1) có nghi m nguyên
dương khi và chỉ khi D là so không chính phương.
Chŕng minh. Giả sử D = m2
. Khi đó
x2
−Dy2
= x2
−m2
y2
= 1 → (x − my)(x + my) = 1
→ x − my = x + my = 1 → x = 1,y = 0.
V y (2.1) không có nghi m nguyên dương.
Ngưoc lại giả sử D là so không chính phương. Ta có các bo đe sau
Bo đe 2.1.2. Cho α ∈
/ Q. Khi đó ton tại vô so c p nguyên (h,k) với k > 0 sao cho
. k. k2
24
Viết đề tài giá sinh viên – ZALO:0973.287.149-TEAMLUANVAN.COM
1
h0 1 1
k
h0 1 1 h0
√ x 1
√ √ √
x
x 1
√ √ √ √
x x
2
2 2
.α − . < .
A = (h,k) : .α −
h. <
1
.
a phải. chứng minh |A| = ∞. Giả sử trái lại |A| < ∞. Khi đó ton tại ε sao cho
.α −
h. > ε vói moi (h,k) ∈ A. Chon q ∈ N∗ sao cho
. k0
. k0q 2
0
Từ (2.3) ta có (h0,k0) ∈ A → .α − . > ε. Nhưng ≥ > .α − . > ε. Mâu
0 < . D− . < .
.y . .y
|x − Dy | =|x − y D||x + y D| = y . − D.. + D.
< y2 1
(
1
+2 D) =
1
+2 D < 1 +2 D.
Chŕng minh. Ta sử dụng nh n xét đã biet sau: Vói mői q ∈ N∗ ton tại c p so
nguyên (h,k) vói 1 ≤ k ≤ q sao cho
Ký hi u
. k. kq
. k. k2
. k.
q
< ε. (2.2)
Theo nh n xét trên ton tại c p so nguyên (h0,k0) vói 1 ≤ q sao cho
.α − . < ≤ . (2.3)
thuȁn vói (2.2).
. k0
. q k0q . k0
.
Bo đe 2.1.3. Ton tại vô so c p so nguyên dương (x,y) sao cho
|x2
− Dy2
| < 1 + 2
√
D.
Chŕng minh. Theo Bo đe 2.1.2 ton tại vô so c p so nguyên (x,y) sao cho
Suy ra
. y. y2
. + D. = . − D + 2 D. <
+ 2
√
D.
√
.y
√
..y
√
.
y2 y2 y2
V y
h 1
y2
25
Viết đề tài giá sinh viên – ZALO:0973.287.149-TEAMLUANVAN.COM
1 1
Chŕng minh định lý Từ bo đe 2.1.3 (x,y) ton tại vô so c p so nguyên dương
(x,y) sao cho
|x2
− Dy2
| < 1 + 2
√
D.
Đ t I = [−1−2
√
d;1+2
√
d]. Vói mői k ∈ I ký hi u
Ak = {(x,y) ∈ N∗ : x2
− Dy2
= k}
Do đó ton tại k ∈ I đe |Ak| = ∞. Suy ra ton tại (x1,y1) /= (x1,y1) ∈ Ak đe
x1 ≡ x2 (mod |k|) y1 ≡ y2 (mod |k|),
x2
− Dy2
= x2
− Dy2
= k,
Xét tích
1 1 2 2
(x1 − y1
√
D)(x2 + y2
√
D)x1x2 − Dy1y2 +
√
D(x1y2 − x2y1) (2.4)
Vì
x1x2 −Dy1y2 ≡ x2
−Dy2
≡ 0 (mod |k|)
x1y2 −x2y1 ≡ x1y1 −x1y1 ≡ 0 (mod |k|).
V y ton tại u,v ∈ Z sao cho
Từ (2.4), (2.5), (2.6) suy ra
x1x2 −Dy1y2 = ku (2.5)
x1y2 −Dy1x2 = kv (2.6)
(x1 − y1
√
D)(x2 + y2) = k(u + v
√
D),
(x1 + y1)(x2 − y2
√
D) = k(u − v
√
D).
Nhân hai đȁng thức trên vói nhau và chú ý rang (x1,y1,(x2,y2) ∈ Ak → x2
−dy2
=
1 1
x2
−dy2
= k ta đưoc
2 2
k2 = k2(u2 − Dv2) → u2 −Dv2 = 1.
26
Viết đề tài giá sinh viên – ZALO:0973.287.149-TEAMLUANVAN.COM
— −
1 1
n n
Ta chứng minh u,v > 0. Rõ ràng u > 0. Neu trái lại v = 0 thì u = ±1 → (x1 −
y1
√
D)(x2 + y2
√
D) = ±k = ±(x2
− Dy2
) = ±(x1 − y1
√
D)(x1 + y1
√
D) → x2 +
y2
√
D = x1 + y1
√
D → x1 = x2,y1 = y2. Ta có mâu thuȁn. V y (u,v) là nghi m
nguyên dương của phương trình (2.1).
Định lí 2.1.4 (Công thức nghi m). Ký hi u (a,b) là nghi m nhó nhat của phương
trình
Khi đó dãy (xn,yn) cho bới
x
x2
− Dy2
= 1.
(a + b
√
D)n
+ (a − b
√
D)n
n = 2
,
yn =
(a+ b
√
D)n
(a b
√
D)n
2
√
D
.
cho tat cá các nghi m của (2.2)
Dãy nghi m (xn,yn) cũng có the xác định theo công thức truy hoi sau
x0 = 1,x1 = a,xn+2 = 2axn+1 −xn, (2.7)
y0 = 0,y1 = b,yn+2 = 2ayn+1 −yn. (2.8)
Chŕng minh. Ta có
xn +yn
√
D = (a + b
√
d)n
,xn −yn = (a − b
√
d)n
. (2.9)
Suy ra (x2
− Dy2
) = (a2 − Db2)n
= 1.
Đảo lại giả sử (x,y) là nghi m bat kỳ của (2.1). Ta chứng minh ton tại n ∈ N∗
đe x = xn;y = yn. Vì D không chính phương nên đieu này tương đương Ton tại
n ∈ N∗ đe
x+y
√
D = xn +yn
√
D = (a+b
√
D)n
.
Chứng minh bang phản ứng. Giả sử trái lại
x + y
√
D /= (a +b
√
D)n
∀n ∈ N∗.
27
Viết đề tài giá sinh viên – ZALO:0973.287.149-TEAMLUANVAN.COM
m m
Khi đó ton tại m ∈ N∗ sao cho
(a +b
√
D)m
< x + y
√
D < (a + b
√
D)m+1
.
Nhân hai ve vói (a − b
√
D)m
ta đưoc
1 < (x+y
√
D)(a−b
√
D)m
< a+b
√
D.
Do (2.9) ta có
(x + y
√
D)(a − b
√
D)m
= (x + y
√
D)(xm − ym
√
D)
= (xxm − Dyym) + (xmy − ymx)
√
D
= u + v
√
D,
ỏ đó u = xxm − Dyym,v = xmy − ymx. V y
1 < u + v
√
d < a + b
√
d. (2.10)
Ta có
u2 − Dv2 = (xxm − Dyym)2
− D(xmy − ymx)2
,
= (x2
− Dy2
)(x2
− Dy2
) = 1.
Lại có x > y
√
d, xm > ym
√
d nên u > 0. Lại có (u − v
√
D)(u + v
√
D) = 1 và 1 <
u + v
√
d nên 0 < u − v
√
d < 1 < u + v
√
d → v > 0. V y (u, v) là nghi m của (2.1)
do đó a ≤ u,b ≤ u → a + b
√
d ≤ u + v
√
d trái vói (2.10). Định lý đưoc chứng
minh.
Từ định lý trên ta thay vi c tìm nghi m của phương trình Pell (2.1) quy ve tìm
nghi m nhỏ nhat (a, b) của nó. Cách đơn giản nhat là thử bang tay: Thay lan lưot
y = 1,2,... vào bieu thức 1 + Dy2
cho tói khi nào đưoc so chính phương thì dừng
lại. Vì phương trình (2.1) có nghi m nên chac chan quá trình này sẽ dừng lại sau
b phép thử. Khi đó nghi m nhỏ nhat là (a,b) vói a =
√
1+Db2 . Neu nghi m nhỏ
28
Viết đề tài giá sinh viên – ZALO:0973.287.149-TEAMLUANVAN.COM
nhat b này lón thì cách thử này không khả thi. Thí dụ vói phương trình x2
− 61y2
thì c p nghi m nhỏ nhat là
a = 1766319049, b = 226153980.
Sau đây ta sẽ trình bày m t thu t toán sử dụng liên phân so đe tìm m t nghi m nhỏ
nhat (a,b) của (2.1).
Định nghĩa 2.1.5. Cho a0,a1,a2,... là dãy so nguyên trong đó ai > 0,i ≥ 1. Đ t
Ck = [a0;a1,...,ak].
Khi đó ton tại giói hạn
limCk = α.
Ta goi α là giá trị của liên phân so vô hạn [a0,a1,a2,...]và viet
α = [a0,a1,a2,...].
Định lí 2.1.6. α = [a1, a1, a2, . . .] là m t so vô tý. Ngược lại mői so vô tý đeu bieu
dien duy nhat dưới dạng m t liên phân so vô hạn.
Định nghĩa 2.1.7. Ta goi liên phân so vô hạn α = [a1; a1, a2, . . .] là tuan hoàn neu
dãy (an) tuan hoàn ke từ m t chỉ so nào đó tức là: Ton tại các so nguyên dương m
và k sao cho moi n ≥ m ta có an = am+k. So nguyên dương k đưoc goi là chu kỳ
của liên phân so α = [a0;a1,a2,...]. Khi đó ta viet
α = [a0;a1,a2,...,am−1,am,am+1,...,am+k−1].
Neu D là so không chính phương, bieu dien liên phân so của
√
D đưoc cho bỏi
Định lí 2.1.8. Bieu dien liên phân so của
√
D là tuan hoàn và có dạng
√
D = [a;a1,a2,...,an,2a].
với a = [
√
D]. Hơn nra có công thŕc tường minh đe xác định dãy (a1,...,an). Chú
ý rang dãy (a1,...,an) là đoi xŕng tŕc là
a1 = an,a2 = an−1,...
29
Viết đề tài giá sinh viên – ZALO:0973.287.149-TEAMLUANVAN.COM
n−1
2n−1
Ví dụ 2.1.9.
√
23 = [4; 1,3,1,8]
√
29 = [5; 2, 1,1,2, 10]
√
31 = [5;1,1,3,5,3,1,1,10]
√
46 = [6;1,2,1,1,2,6,2,1,1,2,1,12]
√
76 = [8;1,2,1,1,5,4,5,1,1,2,1,16]
√
97 = [9;1,5,1,1,1,1,1,1,5,1,18].
Định lí 2.1.10. Cho phương trình Pell
x2
− Dy2
= 1. (I)
1. Bieu dien
√
D thành liên phân so
√
D = [a;a1,a2,...,an,2a].
2. Neu chu kỳ n là so chȁn ta tính gián phân thŕ n − 1
C =
pn−1
.
qn−1
3. Khi đó (pn−1, qn−1) là nghi m nhó nhat của (2.1)
4. Neu chu kỳ n là so lé ta tính gián phân thŕ 2n−1
C =
p2n−1
.
q2n−1
5. Khi đó (p2n−1, q2n−1) là nghi m nhó nhat của (2.1)
Ví dụ 2.1.11. Tìm nghi m nhỏ nhat của phương trình x2
− 14y2
= 1. Ta có
√
14 =
[3;1,2,1,6]. Chu kỳ n = 4 là so chȁn. V y ta tính giản phân
C3 = [3,1,2,1] = 3 +
1
1+
1
2 +
1
1
30
Viết đề tài giá sinh viên – ZALO:0973.287.149-TEAMLUANVAN.COM
15
=
4
.
V y nghi m nhỏ nhat là (15,4).
Ví dụ 2.1.12. Tìm nghi m nhỏ nhat của phương trình x2
− 13y2
= 1. Ta có
√
13 =
[3;1,1, 1,1,6] = [3,1, 1,1,1,6, 1,1,1, 1,6,... Chu kỳ n = 5 là so lẻ. V y ta tính giải
phân
C9 = [3,1,1,1,1,6,1,1,1] = 3 +
1
=
649
.
1+
1
1+
... +
1
1 +
1
1
180
V y nghi m nhỏ nhat là (649,180) Trỏ lại phương trình x2
− 61y2
= 1. Ta có
√
76 = [7;1,4,3,1,2,2,1,3,4,1,14]
Chu kỳ n = 11 là so lẻ. Ta tính giản phân
C21 = [7,1,4,3,1,2,2,1,3,4,1,14,1,4,3,1,2,2,1,3,4,1]
1
= 7+
1+
1
4+
1
3 +
... +
1
4 +
1
1
1766319049
=
226153980
V y nghi m nhỏ nhat là (1766319049,226153980)
2.2 Phương trình Pell loại 2
Phương trình Pell loại 2 là phương trình
x2
− Dy2
= −1 (2.11)
ỏ đó D ∈ N∗ . Nghi m của (2.11) luôn đưoc hieu là nghi m nguyên dương.
31
Viết đề tài giá sinh viên – ZALO:0973.287.149-TEAMLUANVAN.COM
1
Định lí 2.2.1. Đieu ki n can đe (2.11) có nghi m là D không là so chính phương
và D không có ước nguyên to dạng 4k+3.
Chŕng minh. i) neu D = m2
thì (2.11) trỏ thành (my− x)(my +x) = 1 → my−
x = my+x = 1 → x = 0 v y (2.11) vô nghi m.
ii) Neu D có ưóc nguyên to p = 4k+3 : Giả sử (x,y) là nghi m. Khi đó x2
+1 =
Dy2
→ p|x2
+ 1. Vì p = 4k + 3 nên p|1. Mâu thuȁn.
Tuy nhiên, đây không là đieu ki n đủ.
Định lí 2.2.2. Neu D = p là so nguyên to thì (2.11) có nghi m khi và chỉ khi p = 2
ho c p /= 4k + 3.
Chŕng minh. Neu (2.11) có nghi m thì từ Định lý 1 suy ra p = 2 ho c p /= 4k +3
.
Đảo lại neu p = 2 thì phương trình x2
= −2y2
= −1 có nghi m (x, y) = (1; 1). Xét
p ≡ 1( mod 4). Xét phương trình Pell loại I
x2
− Dy2
= 1 (2.12)
Goi (a,b) là nghi m nhỏ nhat của (2.12). Ta có a2
− 1 = pb2
. Neu a chȁn thì b lẻ
do đó b2
≡ 1( mod 4) → a2
≡ 1 + p ≡ 2( mod 4). Mâu thuȁn. V y a lẻ, b chȁn.
Đ t a = 2a1 + 1,b = 2b1. Ta có (a − 1)(a + 1) = pb2
⇔ a1(a1 + 1) = pb2
. Do p
nguyên to và (a1,a1 +1) = 1 nên a1 = u2
,a1 +1 = pv2
ho c a1 = pu2
,a1 +1 = v2
.
Neu a1 = u2
, a1 + 1 = pv2
→ u2
− pv2
= −1. V y (2.11) có nghi m (u, v). Neu
a1 = pu2
, a1 + 1 = v2
→ v2
− pu2
= 1. V y (v, u) là nghi m của (2.12). Suy ra
u ≥ a → a1 +1 = v2
≥ v ≥ a = 2a1 +1. Mâu thuȁn.
Định lí 2.2.3. Goi (a, b) là nghi m nhó nhat của (2.12). Khi đó (2.11) có nghi m
khi và chỉ khi
a = x2
+Dy2
b = 2xy
(2.13)
32
Viết đề tài giá sinh viên – ZALO:0973.287.149-TEAMLUANVAN.COM
1
2 2
2
1
1
2 2 2 2 2 2 2 2
1 1 1
có nghi m nguyên dương.
Hơn nữa neu (2.13) h có nghi m nó sẽ có nghi m duy nhat. Nghi m duy nhat
(x1,y1) này chính là nghi m nhỏ nhat của (2.11).
Chŕng minh. 1) Giả sử (2.11) có nghi m. Goi (x1,y1) là nghi m nhỏ nhat của
(2.11). Đ t u = x2
+Dy2
,v = 2x1,y1. Ta chứng minh u = a,v = b do đó (x1,y1)
1 1
chính là nghi m của (2.13). Ta có u2
−Dv2
= (x2
− Dy2
)2
= 1. V y (u,v) là
1 1
nghi m của (2.12). Suy ra u ≥ a;v ≥ b. Ta có
(a2
− Db2
)(x1 − Dy2
) = 1(−1) = −1
⇔(ax1 − Dby1) − D(ay1 −bx1) = −1.
De thay (ax1 − Dby1)2 > 0, (ay1 − bx1)2 > 0. Do (x1, y1) là nghi m của
(2.11) nên
(ax1 −Dby1)2
≥ x2
⇔ a2
x2
+D2
b2
y2
−x2
≥ 2abDx1y1
1 1 1 1
2 2 2 2 2
→ x1(a −1)+D b y1 ≥ 2abDx1y1
→ x1Db2
+ D2
b2
y2
≥ 2abDx1y1
2 2
→ b(x1 + dy1) ≥ 2ax1y1 → bu ≥ av
→ b u ≥ a v → b (Dv +1) ≥ v (Db +1)
→ b ≥ v → b = v → u = a.
2) Đảo lại giả sử (u, v) là nghi m của (2.13). Ta có a2
− Db2
= 1 ⇔ (u2
+ Dv2
)2
−
D(2uv)2
= (u2
− Dv2
)2
= 1. V y u2
− Dv2
= 1 ho c u2
− Dv2
= −1. Neu
u2
− Dv2
= 1 thì (u, v) là nghi m của (2.12) do đó u ≥ a = u2
+ Dv2
. Mâu
thuȁn. V y u2
− Dv2
= −1 do đó (2.11) có nghi m (u,v).
Tiep theo ta chứng minh (u,v) là nghi m nhỏ nhat của (2.11). Giả sử (x1,y1) là
nghi m nhỏ nhat của (2.11). Theo chứng minh ỏ 1) ta có a = u2
+Dv2
= x2
+
Dy2
;b = 2uv = 2x1y1 → u2
+Dv2
+2uv = x2
+Dy2
+2x1y1 → (u+v
√
D)2
=
(x1 +y1)2
→ u = x1;v = y1. Định lý đưoc chứng minh.
33
Viết đề tài giá sinh viên – ZALO:0973.287.149-TEAMLUANVAN.COM
6 = 2xy
n−1
Ví dụ là m t áp dụng của định lý 2. Nó cũng chỉ ra rang đieu ki n của định lý
1 chỉ là đieu ki n can.
Ví dụ 2.2.4. Chứng minh rang phương trình x2
− 34y2
= −1 vô nghi m (Ő đây
34 = 2.17 không so chính phương và cũng không có ưóc nguyên to dạng 4k + 3).
Giái Phương trình x2
−34y2
= 1 có nghi m nhỏ nhat là (a;b) = (35;6). Xét h
35 = x2
+34y2
Từ phương trình thứ nhat của h suy ra (x;y) = (1;1). Tuy nhiên (1;1) không thoả
mãn phương trình thứ hai. V y h vô nghi m do đó theo định lý 2 phương trình
x2
−34y2
= −1 vô nghi m.
Ta thừa nh n định lý
Định lí 2.2.5. Phương trình Pell loại 2
x2
− Dy2
= −1
có nghi m khi và chỉ khi trong bieu dien
√
D thành liên phân so
√
D = [a; a1, a2,...,an, 2a]
chu kỳ n là so lé. Trong trường hợp đó (pn−1, qn−1) là nghi m nhó nhat của phương
trình ớ đó.
là giái phân thŕ n−1.
C =
pn−1
qn−1
Ví dụ 2.2.6. i) Xét phương trình x2
− 13y2
= −1. Ta có
√
D = [3;1,1,1,1,6].
Chu kỳ n = 5 là so lẻ. V y phương trình có nghi m. Ta tính giản phân
C4 = [3,1,1,1,1]
34
Viết đề tài giá sinh viên – ZALO:0973.287.149-TEAMLUANVAN.COM
n=1
−
n n
1
= 3 +
1
1+
1
1+
1
3 18
= 3 +
5
=
5
V y nghi m nhỏ nhat là (18,5)
1 +
1
ii) Xét phương trình x2
−34y2
= −1. Ta có
√
34 = [5;1,4,1,10]. Chu kỳ n = 4
là so chȁn. V y phương trình vô nghi m.
Định lí 2.2.7 (Công thức nghi m). Giá sr phương trình Pell loại 2
x2
− Dy2
= −1 (II)
có nghi m. Goi (α,β ) là nghi m nhó nhat của nó. Khi đó dãy (xn, yn)∞ cho bới
x
(α + β
√
D)2n−1
+ (α − β
√
D)2n−1
n =
2
yn =
(α + β
√
D)2n−1
+ (α β
√
D)2n−1
2
√
D
cho ta tat cá các nghi m của (II).
Chŕng minh. Giả sử (xn,yn) cho bỏi công thức trên. Khi đó
xn + yn
√
D = (α + β
√
D)2n−1
xn −yn
√
D = (α −β
√
D)2n−1
→ x2
− Dy2
= (a2 − Dβ 2
)2n−1
= −1
Ngưoc lại giả sử (x,y) là m t nghi m của (2.11). Ta có
(x + y
√
D)(α + β
√
D) = (xα + Dyβ )(yα + xβ )
√
D
= s +t
√
D
ỏ đó s = xα + Dyβ,t = yα + xβ
→ s2
−Dt2
= (xα +Dyβ)2
−D(yα +xβ)2
35
Viết đề tài giá sinh viên – ZALO:0973.287.149-TEAMLUANVAN.COM
4
= (x2
− Dy2
)(α2
− Dβ 2
) = (−1)(−1) = 1
V y (s,t) là nghi m của phương trình Pell loại 1 s2
−Dt2
= 1. Goi (a,b) là nghi m
nhỏ nhat của nó. Theo công thức nghi m của phương trình Pell loại 1 và định lý 3
ton tại n ∈ N∗ sao cho
(x + y
√
D)(α + β
√
D) = s +t
√
D = (a + b
√
D)n
= (α2
+ Dβ 2
+ 2αβ
√
D)n
= (α + β
√
D)2
)n
= (α + β
√
D)2n
→ x + y
√
D = (α +β
√
D)2n−1
= xn + yn
√
D
→ x = xn,y = yn
M t so bài toán chon loc
Bài toán 2.2.8. So nguyên dương S được goi là so Heron neu nó là di n tích của
m t tam giác có ba cạnh là ba so nguyên liên tiep. Chŕng minh rang S là so Heron
khi và chỉ khi S khi là so hạng của dãy (Sn),n ≥ 1 xác định bới
S0 = 0,S1 = 6,S2 = 84,Sn+2 = 14Sn+1 − Sn
Lời giái. Goi S là di n tích của tam giác có ba cạnh là x − 1,x,x + 1 vói x > 2,x ∈
N∗. Theo công thức Heron.
S =
1
x
q
3(x2 − 4) → 16S2
= 3x2
(x2
− 1) (2.14)
V y S là so Heron khi và chỉ khi phương trình (2.14) có nghi m nguyên dương(S,x).
De thay x phải chȁn. Đ t x = 2y ta có
16S2
= 3x2
(x2
− 1) ⇔ S2
= 3y2
(y2
− 1)
→ S = y
q
3(y2 − 1) → 3(y2
= −1) = h2
36
Viết đề tài giá sinh viên – ZALO:0973.287.149-TEAMLUANVAN.COM
2 2
2 2
√
√ √
n
n
4
k
→ h = 3z → 3(y −1) = 9z
→ y −3z = 1,S = 3yz
Ngưoc lại neu (y,z) là nghi m của phương trình Pell
y2
− 3z2
= 1 (2.15)
thì x = 2y,y > 1,S = 3yz là nghi m của (2.14) Nghi m nhỏ nhat của (2.15) là (2,1)
V y tat cả nghi m của (2.15) (yn,zn) cho bỏi
Do đó
yn =
zn =
(2 +
√
3)n
+ (2 −
√
3)n
2
(2+ 3) −(2 − 3)
2 3
Sn = 3ynzn
=
√
3
((7+
√
3)n
−(7 − 4
√
3)n
)
→ Sn+2 = 14Sn+1 −Sn,S0 = 0,S1 = 6,S2 = 84
Bài toán 2.2.9. Tìm tat cá các so nguyên dương n sao cho 2n + 1 và 3n + 1 đeu là
so chính phương.
Lời giái. Vì (2n + 1, 3n + 1) = 1 nên 2n + 1 và 3n + 1 đeu là so chính phương
khi và chỉ khi (2n + 1)(3n + 1) = y2
, y ∈ N∗. Suy ra (12n + 5)2
− 24y2
= 1. Đ t
x = 12n + 5 ta có x2
− 24y2
= 1. Goi (xk,yk) là nghi m của nó. Nghi m nhỏ nhat
của phương trình Pell này là (5,1). Do đó (xk) cho bỏi h thức.
x0 = 1,x1 = 5,xk+2 = 10xk+1 − xk.
De chứng minh xk ≡ 5 (mod 12) khi vào chỉ khi k lẻ. V y n = nk,k ≥ 1 ỏ đó
n =
x2k+1 −5
12
Ta xác định h thức truy hoi của dãy (nk)
37
Viết đề tài giá sinh viên – ZALO:0973.287.149-TEAMLUANVAN.COM
√ √
2n 1 2n
1
Đ t uk = x2k+1 = 12nk +5. Ta có
x2k+3 = 10x2k+2 − x2k+1 = 10(10x2k+1 − x2k) − x2k+1
= 100x2k+1 − 10x2k − x2k+1 = 99x2k+1 = x2k+1 − x2k−1
= 98x2k+1 − x2k−1
→ uk+2 = 98uk+1 −uk
⇔ 12nk+2 + 5 = 98(12nk+1 + 5) = 12nk − 5
nk+2 = 98nk+1 −nk +40
vói n1 = 40,n2 = 3960
Bài toán 2.2.10. Cho dãy (xn,yn) xác định như sau (x0,y0) = (0,1),(x1,y1) =
(3,5) và
xn+1 = 3xn +2yn +1
yn+1 = 4xn +3yn +2
(2.16)
Chŕng minh rang (xn,yn) là tat cá các nghi m nguyên dương của phương trình
x2
+(x+ 1)2 = y2
Lời giái. Phương trình đã cho tương đương vói
(2x + 1)2
− 2y2
= −1 (2.17)
Đ t u = 2x + 1, → u2
− 2y2
= −1. Nghi m nhỏ nhat của phương trình này là (1.1).
Do v y dãy nghi m (un,yn) cho bỏi
un = (1 +
√
2)2n+1
+ (1 −
√
2)2n+1
2
(1+ 2) + −(1− 2) +
Từ đó
yn =
2
√
2
u0 = 1,u1 = 7,uk+2 = 6uk+1 −uk
38
Viết đề tài giá sinh viên – ZALO:0973.287.149-TEAMLUANVAN.COM
y0 = 1,y1 = 5,yk+2 = 6yk+1 −yk
Ta có un = 2xk+2 +1 → 2xk+2 +1 = 6(2nk+1 +1)−2nk −1 → x0 = 0,x1 = 3,xk+2 =
6xk+1 −xk +2. Thành thử dãy nghi m (xn,yn) của (2.17) cho bỏi
x0 = 0,x1 = 3,xn+2 = 6xn+1 −xn +2
y0 = 1,y1 = 5,yn+2 = 6yn+1 = yk
Thành thử ta chỉ can chứng minh dãy (2.16) thoả mãn quan h
xn+2 = 6xn+1 −xn +2
yn+2 = 6yn+1 − yk
Th t v y đ t zn = 2xn + 1. Khi đó de kiem tra
zn+1 = 3zn + 4yn
yn+1 = 2zn + 3yn
→ zn+2 = 3zn + 1 + 4yn+1 = 3zn+1 + 8zn + 12yn
= 3zn+1 + 8zn +3zn+1 −9zn = 6zn+1 −zn
→ 2xn+2 + 1 = 6(2xn+1 +1) − 2xn−1
→ xn+2 = 6xn+1 −xn +2
Tương tụ yn+2 = 6yn+1 −yk
2.3 Phương trình Pythagoras
Trong mục này chúng ta sẽ đi tìm tat cả các nghi m nguyên dương của phương
trình
x2
+y2
= z2
(2.18)
và xét m t so ứng dụng của nó. Những phương trình Diophantine này đưoc goi là
các phương trình Pythagoras.
39
Viết đề tài giá sinh viên – ZALO:0973.287.149-TEAMLUANVAN.COM
Định nghĩa 2.3.1. M t b ba so nguyên dương (x, y, z) thoả mãn phương trình
(2.18) goi là m t b ba so Pythagoras.
Các b ba so Pythagora bieu thị đ dài các cạnh của m t tam giác vuông. Từ
m t b ba so Pythagoras (x, y, z) nào đó ta suy ra de dàng m t t p hop vô hạn các
b ba so Pythagoras khác (tx,ty,tz) vói t là so nguyên dương.
Ngưoc lại, cho (x,y,z) là m t b ba so Pythagoras bat kỳ và giả sử d = (x,y,z).
Khi đó (x1,y1,z1) cũng là m t b ba so Pythagoras, ỏ đó
x
x y z
1 =
d
, y1 =
d
, z1 =
d
, và (x1, y1, z1) = 1.
Định nghĩa 2.3.2. M t b ba so Pythagoras (x,y,z) là nguyên thuý neu (x,y,z) = 1.
Rõ ràng từ lý lu n trên ta chỉ can đi tìm các b ba so Pythagoras nguyên thuý.
De dàng nh n thay neu (x, y, z) là m t b ba so Pythagoras nguyên thuý thì
chúng đôi m t nguyên to cùng nhau. Th t v y, giả sử d = (x, y). Khi đó d2
là ưóc
của z2
nên d là ưóc của z do đó d là ưóc chung của x, y, z. V y d = 1.
Định lí sau đây cho công thức mô tả tat cả các b ba so Pythagoras nguyên
thuý.
Định lí 2.3.3. Giá sr (x,y,z) là m t b ba so Pythagoras nguyên thuý. Khi đó x và
y có tính chȁn lé khác nhau. Neu x chȁn, y lé chȁng hạn (x,y,z) có dạng
x = 2mn, y = m2
− n2
, n = m2
+n2
trong đó m, n là hai so nguyên đương nguyên to cùng nhau, có tính chȁn lé khác
nhau.
Đáo lại, neu m, n là hai so nguyên dương nguyên to cùng nhau, m t chȁn, m t
lé thì ba b (x,y,z) xác định như trên là m t b ba so Pythagoras nguyên thuý.
Chŕng minh. Trưóc het vì (x, y) = 1 nên x và y không cùng chȁn. Hai so x và y
cũng không the cùng lẻ vì neu như v y thì z2
= x2
+ y2
≡ 2 (mod 4), đieu này
không the có vì m t so chính phương chỉ đong dư 0 ho c 1 theo modulo 4.
40
Viết đề tài giá sinh viên – ZALO:0973.287.149-TEAMLUANVAN.COM
0
0
2 2 2
2 2 2
0 0 0
x2
− y2
= t2
,
Giả sử x chȁn, y lẻ. Khi đó z lẻ và x2
= z2
− y2
(z + y)(z − y), đieu này tương
đương vói
x 2
=
z+y z−y
. (2.19)
De thay rang z+y
, z−y
= 1 vì (z,y) = 1. Suy ra ton tại m, n ∈ N∗ đe z+y
= m2
và
z−
2
y
= n2
. Từ đó x = 2mn, y = m2
− n2
, z = m2
+ n2
. M t khác (m2
,n2
) = 1 nên
(m,n) = 1. Hơn nữa vì y và z lẻ nên m, n có tính chȁn lẻ khác nhau.
Bây giò ta chứng minh phan đảo của định trong định lí trên. De dàng kiem tra
rang x, y, z xác định theo công thức trên là b ba so Pythagoras.
Ta còn phải chứng minh đó là b ba so Pythagoras nguyên thuý. Giả sử d =
(y,z). Khi đó vì y, z lẻ nên d lẻ. Ta có d | y+ z = 2m2
suy ra
d | m2
và d | z−y = 2n2
suy ra d | n2
.
Vì (m,n) = 1 nên từ đó d = 1. V y (y,z) = 1 suy ra (x,y,z) = 1. Định lí đưoc chứng
minh.
Sau đây là áp dụng quan trong của Định lí 2.3.3.
Định lí 2.3.4. Không ton tại hai so tự nhiên x và y đe tőng các bình phương và
hi u các bình phương của chúng đeu là các so chính phương.
Chŕng minh. Ta chứng minh bang phản chứng. Giả sử ton tại các so nguyên dương
x, y, có tính chat đã nêu.
Trong các c p so (x,y) ta chon (x0,y0) là c p so mà x nhỏ nhat. Giả sử
x2
+ y2
= z2
,
vói z0, t0 ≤ N∗.
0 0 0
Ta có (x0,y0) = 1. Th t v y, giả sử d = (x0,y0). Từ (2.20) suy ra d2
| z2
và
d2
| t2
. Từ đó d | z0 , d | t0 . Đ t
x0
, y1 =
y0
, z1 = y
z0
, t1 =
t0
.
d d d d
(2.20)
41
Viết đề tài giá sinh viên – ZALO:0973.287.149-TEAMLUANVAN.COM
0
0
1 1 1 1 1 1
1 1 0 0 1 1
0 0 0
Ta có
x2
+y2
= z2
, x2
− y2
= t2
.
Suy ra (x1,y1) là m t c p thoả mãn tính chat đã nêu. Do đó
x2
+y2
≥ x2
+y2
= d(x2
+y2
).
V y d = 1.
Từ (2.20) suy ra z2
+t2
= 2x2
. Do đó z0 và t0 có cùng tính chȁn lẻ.
Đ t
0 0 0
u =
z0 +t0
, v =
z0 −t0
2 2
vói u, v là các so nguyên dương.
Khi đó
u + v = z0, u − v = t0vu2
+ v2
= x2
. (2.21)
Ta có (u,v) = 1, vì neu d = (u,v) thì d2
| x2
. M t khác d | u + v = z0, suy ra
d2
| z2
− x2
= y2
suy ra d | y0. V y d = 1.
Theo (2.21) thì (u, v, x0) là m t b ba so Pythagoras nguyên thuý. Theo Định
lí 2.3.3 ton tại các so nguyên dương m, n chȁn lẻ khác nhau vói (m, n) = 1, m > n
sao cho
u = 2mn, v = m2
− n2
;
ho c u = 2m2
− n2
, v = 2mn.
Trong moi trưòng hop, uv = 2mn(m2
− n2
). Ta lại có
2y = (u + v)2
− (u − v)2
= 4uv = 8mn(m2
− n2
)
suy ra y = 4mn(m2
− n2
), tức là y0 = 2k. V y
mn(m2
− n2
) = k2. (2.22)
Ta có (m,n) = 1 nên (m + n,m) = 1 và (m − n,m) = 1. V y (m2
− n2
,m) = 1.
42
Viết đề tài giá sinh viên – ZALO:0973.287.149-TEAMLUANVAN.COM
4
2
r
2
r
2
r
Tương tụ, (m2
− n2
,n) = 1. Do đieu này nên từ (2.3) suy ra ton tại a,b,c ∈ N∗
đe
m = a2, n = b2, m2 − n2 = c2. (2.23)
Goi d = (m + n,m −n). Do m, n chȁn lẻ khác nhau nên m + n, m − n lẻ, từ đó d lẻ.
Vì d | m+n, d | m− n nên d | 2m, d | 2n suy ra d | m, d | n (do d lẻ). Suy ra d = 1.
Do đieu này nên từ (2.23) suy ra ton tại r, s ∈ N∗ đe
m − n = r2
, m + n = s2
.
V y
a2 +b2 = s2
, a2 − b2 = r2
,
tức là c p (a,b) thoả mãn tính chat đã nêu trong định lí.
M t khác
a2
+ b2
= m + n ≤ 2m ≤ 2mn ≤ u =
z0 +t0
< z0 ≤ z2
= x2
= y2
.
2 0 0 0
Đieu này trái vói cách chon c p (x0,y0). Định lí đưoc chứng minh xong.
M t so bài toán chon loc
Bài toán 2.3.5 (Đe dụ tuyen thi Toán quoc te năm 1979). Chŕng minh rang không
ton tại hình chóp tŕ giác đeu mà các cạnh, di n tích toàn phan và the tích của nó
đeu là các so nguyên.
Chŕng minh. Giả sử g, f , h, S và V theo thứ tụ là cạnh đáy, cạnh bên, chieu cao,
di n tích toàn phan và the tích của m t hình chóp tứ giác đeu và chúng là những
so nguyên. Ta có
f = h2 +
g
,
2
g2
h
V =
3
, S = g2 +2g h2 +
g
.
4
Vì g, S là so nguyên nên 2g
q
h2 + g2
là so nguyên. Kí hi u
x = 2g2
h2 +
g
,
4
y = g3
.
43
Viết đề tài giá sinh viên – ZALO:0973.287.149-TEAMLUANVAN.COM
3
0 0
0 0 0 0 0 0 0 0
0 0 0 0 0 0 0 0
0 0
0 0 0 0
0 0 0 0 0 0
1 1 0 1 1 0
De thay x, y ∈ N∗. Ta có
x2
− y2 = 4g4
22
+
g
4
—g6
= 4h2
g4
= (2g2
h)2
là so chính phương, vì V = g
2h
∈ N∗. Do đó g2
∈ N∗. M t khác,
x2
+y2
= 4g4
h2
+2g6 = 4g4
h2
+
g
2
= 4g4
f2
= (2g2
f )2
.
Vì 2g2
f ∈ N∗ nên x2
+ y2
là so chính phương. Song đieu này mâu thuȁn vói Định
lí 2.3.4. Ta có đieu can chứng minh.
Bài toán 2.3.6. Chŕng minh rang phương trình
x4
− y4
= z2
không có nghi m nguyên dương.
Chŕng minh. Ta sẽ chứng minh bài toán này bang phương pháp phản chứng. Giả
sử (x,y,z) là nghi m nguyên dương của phương trình đã cho và d = (x,y). Ta có
x = dx0, y = dy0, (x0,y0) = 1.
Thay vào phương trình cho ta
d4
(x4
−y4
) = z2
suy ra d2
| z.
Đ t z = d2
z0 suy ra x4
−y4
= z2
, tức là (x2
−y2
)(x2
+y2
) = z2
. Neu x0 và y0 chȁn
lẻ khác nhau thì (x2
−y2
,x2
+y2
) = 1 bỏi vì neu d1 | (x2
−y2
) và d1 | (x2
+y2
) thì
d1 | 2y2
và d1 | 2x2
. V y d1 = 1 (do d1 lẻ).
Do đó x2
− y2
= a2
, x2
− y2
= b2
, trái vói Định lí 2.3.4.
Neu x0 và y0 cùng lẻ thì x2
+y2
= 2a, x2
+y2
= 2b Suy ra x2
= a+b, y2
= a−b.
Ta có 4ab = z2
suy ra ab = z2
. Vì (x0,y0) = 1 nên (a,b) = 1. Do đó a = a2
, b = b2
.
0 1 1 1
V y a2
+ b2
= x2
, a2
− b2
= y2
, và đieu này mâu thuȁn vói Định lí 2.3.4.
2
2
44
Viết đề tài giá sinh viên – ZALO:0973.287.149-TEAMLUANVAN.COM
Bài toán 2.3.7. Chŕng minh rang phương trình
x4
+y4
= z2
không có nghi m nguyên dương.
Chŕng minh. Giả sử trái lại, phương trình đã cho có nghi m nguyên dương. Goi
(x0, y0) là nghi m nguyên dương sao cho tong x là nhỏ nhat. L p lu n tương tụ
như trong chứng minh Định lí 2.3.4 ta thay (x0,y0) = 1. Như v y (x2
,y2
,z0) là m t
0 0
b ba so Pythagoras nguyên thuý. Không làm giảm tong quát ta giả sử x0 lẻ. Theo
Định lí 2.3.3 ton tại m, n vói (m,n) = 1 sao cho
,
x = m2
−n2
, y = 2mn,z0 = m2
+n2
. (2.24)
Vì (m,n) = 1 nên từ (2.24) ta có (x0,n,m) là m t b ba so Pythagoras nguyên thuý.
V y lại theo Định lí 2.3.3 ton tại a, b vói (a,b) = 1 sao cho
,
x0 = a2
− b2
, n = 2ab,m = a2
+ b2
. (2.25)
Ta có y2
s = 2mn = 4abm suy ra abm = y2
(ỏ đây y0 = 2y1) suy ra a = a2
, b = b2
,
0 1 1 1
m = m2
. Thay vào (2.25) ta có m2
= a4
+b4
. M t khác
1 1 1 1
a4
+b4
= m2
= m < m2
+n2
= z0z2
= x4
+y4
.
1 1 1 0 0 0
Đieu này trái vói cách chon (x0,y0). V y bài toán đưoc chứng minh.
Nh n xét 2.3.8. Qua chứng minh 2.3.4. và lòi giải của các Bài toán 2.3.5, 2.3.6,
2.3.7, ta có the nh n thay m t phương pháp chung sau đây trong lí thuyet phương
trình Diophantine: Đe chứng minh m t phương trình Diophantine đã cho không
có nghi m nguyên dương, ta hãy giả sử nó có và khi đó sẽ có m t nghi m nhỏ
nhat (theo m t nghĩa nào đó). Sau đó ta sẽ co gang kien thiet m t nghi m nhỏ hơn
nghi m nhỏ nhat và như v y ta dȁn đen mâu thuȁn.
45
Viết đề tài giá sinh viên – ZALO:0973.287.149-TEAMLUANVAN.COM
Chương 3
Liên phân so và ng dụng trong
phương trình Diophantine
Như chúng ta đã biet, vói m t so thục cho trưóc, ta có the bieu dien nó theo nhieu
cách, chȁng hạn bieu dien nó như là tích của các so nguyên to (Định lí cơ bản của
Lý thuyet so), hay viet nó theo các h cơ so khác nhau. Trong các cách bieu dien
ay, ngưòi ta quan tâm đ c bi t đen các liên phân so, hay còn goi là các phân so liên
tnc (continued fractions). Các liên phân so cho phép ngưòi ta bieu dien các so hữu
tý và vô tý thành các phân so nhieu tang. Liên phân so là m t chủ đe rat r ng lón
và có nhieu ứng dụng đa dạng trong Lý thuyet so. Chương này dành đe trình này
m t cách ngan gon các sụ ki n ve liên phân so và đ c bi t là ứng dụng của chúng
đe giải phương trình Pell.
3.1 Liên phân so h u hạn
Định nghĩa 3.1.1. Liên phân so hay phân so liên tnc là m t bieu thức có dạng
1
a0 +
a1 +
1
1
a2 + ··· +
1
ai−1 +
an
trong đó a0,a1,...,an là các so thục và a1,...,an
đưoc hi u là [a0,a1,...,an].
0. M t liên phân so như v y
46
Viết đề tài giá sinh viên – ZALO:0973.287.149-TEAMLUANVAN.COM
b
+
Từ định nghĩa de thay
1
[a0,a1,...,ak+1] = a0 +
[a0,a1,..., ak 1]
.
Neu a0 ∈ Z và a1,...,an là các so nguyên dương thì ta nói [a0;a1,...an] là m t
liên phân so hru hạn có đ dài n. Rõ ràng m t liên phân so hữu hạn là m t so hữu
tý. Ngưoc lại ta có:
Định lí 3.1.2. Mői so hru tý đeu có the bieu dien dưới dạng m t liên phân so hru
hạn.
Chŕng minh. Giả sử x = a
trong đó a, b ∈ Z và b > 0. Đ t r0 = a, r1 = b. Thu t
chia Euclide cho ta
r0 = r1q1 +r2, 0 < r2 < r1,
r1 = r2q2 +r3, 0 < r3 < r2,
...
rn−2 = rn−1qn−1 + rn, 0 < rn < rn−1,
Từ đó de thay
rn−1 = rnqn.
a
b
= [q1;q2,...,qn].
Ta có đieu phải chứng minh.
Cho liên phân so hữu hạn [a0;a1,...,an]. Vói mői k ≤ n liên phân so Ck =
[a0;a1,...,ak] goi là gián phân thŕ k của [a0;a1,...,an].
Công thức tính các giản phân đưoc cho bỏi định lí sau.
Định lí 3.1.3. Cho liên phân so hru hạn [a0;a1,...,an]. Giá sr dãy so nguyên
dương p0, p1,..., pn và q0,q1,...,qn được xác định truy hoi như sau
p0 = a0, q0 = 1,
47
Viết đề tài giá sinh viên – ZALO:0973.287.149-TEAMLUANVAN.COM
ak+1
k a
k a
p1 = a0a1 +1 q1 = a1,
...
pk = ak pk−1 + pk−2, qk = akqk−1 + qk−2.
Khi đó gián phân thŕ k là Ck = [a0;a1,...,ak) được cho bới
Ck =
pk
.
qk
Chŕng minh. Ta chứng minh bang quy nạp. Vói k = 0 ta có
C0 = [a0] = p0/q0.
Vói k = 1 ta có
C1 = [a0;a1] = a0 +
1
a1
=
a0a1 +1
= p1/q1.
a1
Giả sử định lí đúng cho moi 0 ≤ k < n. Khi đó vói 2 ≤ k < n,
Ck = [a0;a1,...,ak] ==
pk
=
ak pk−1 + pk−2
.
V y
qk akqk−1 + qk−2
Ck+1 = [a0; a1,...,ak,ak 1] = a0;a1,...,ak − 1,ak +
1
(a + 1
k+1
(a + 1
k+1
)pk−1 + pk−2
)qk−1 + qk−2
=
ak+1(ak pk−1 + pk−2) + pk−1
ak+1(akqk−1 + qk−2) + qk−1
=
ak+1 pk + pk−1
=
pk+1
.
Định lí đưoc chứng minh.
ak+1qk + qk−1 qk+1
Bang phương pháp quy nạp ta de dàng chứng minh đưoc đȁng thức quan trong
sau giữa các (pk) và (qk).
Định lí 3.1.4. pkqk−1 − pk−1qk = (−1)k−1
.
+
=
48
Viết đề tài giá sinh viên – ZALO:0973.287.149-TEAMLUANVAN.COM
−
k
k−2
H qua 3.1.5. (pk,qk) = 1.
Tiep theo, ta sẽ mô tả các quan h của giản phân.
Định lí 3.1.6. Giá sr (Ck) là dãy gián phân của liên phân so hru hạn [a0;a1 ...,an].
Ta có
Ck −Ck−1 =
Ck −Ck−2 =
(−1)k−1
qkqk−1
ak(−1)k
qkqk−2
, 1 ≤ k ≤ n,
, 2 ≤ k ≤ n.
Chŕng minh. Vói đȁng thức thứ nhat ta có
pkqk−1 − pk−1qk (−1)k−1
Ck −Ck−1 = qkqk
=
q q
.
Vói đȁng thức thứ hai ta có
−1 k k−1
C C =
pkqk−2 − pk−2qk
.
qkqk−2
Thay pk = ak pk−1 + pk−2, qk = akqk−1 = qk−2 vào tử so và áp dụng đȁng thức thứ
nhat ta thu đưoc đieu phải chứng minh.
Từ định lí trên ta thu đưoc ket quả quan trong sau.
Định lí 3.1.7. Ta có
C1 > C3 > C5 > ...,
C0 < C2 < C4 < ...
Hơn nra mői gián phân lé C2 j−1 đeu lớn hơn mői gián phân chȁn C2i.
Chŕng minh. Từ định lí trên ta thay neu k lẻ thì Ck < Ck−2 và neu k chȁn thì
Ck > Ck−2. Cũng theo định lí trên, ta có)
(−1)2m−1
C2m −C2m−1 =
q2mq2m
< 0 suy ra C2m < C2m−1.
−
V y C2 j−1 > C2 j−1+2i > C2 j+2i > C2i.
1
49
Viết đề tài giá sinh viên – ZALO:0973.287.149-TEAMLUANVAN.COM
+
+ −
3.2 Liên phân so vô hạn
Như ket quả mục trên, ta biet rang mői so hữu tý sẽ đưoc bieu dien m t cách duy
nhat bang m t liên phân so hữu hạn. Chuyen sang tình huong các so vô tý, ta sẽ
thay, mői so vô tý sẽ đưoc bieu thị duy nhat dưói dạng m t liên phân so vô hạn.
Định lí 3.2.1. Cho a0, a1, a2 . . . là dãy vô hạn các so nguyên với ai > 0 với i ≥ 1.
Đ t
Khi đó ton tại giới hạn
Ck = [a0; a1,...,ak].
lim Ck = α.
k→∞
Ta goi α là giá trị của liên phân so vô hạn [a0;a1;a2 ...] và viet
α = [a0; a1; a2,...].
Chŕng minh. Theo Định lí 3.1.7 ta có
C1 > C3 > C5 > ... > C2n−1 > C2n+1 > ...
C0 < C2 < C4 < ... < C2n−2 < C2n < ...
Hơn nữa dãy (C2k+1) là dãy giản và bị ch n dưói bỏi C0 còn dãy (C2k) tăng và bị
ch n trên bỏi C1. V y ton tại
lim C2k+1 = α1 và lim C2k = α2.
k→∞ k→∞
Ta can chứng minh α1 = α2. Th t v y theo Định lí 3.1.7 ta có
C2k
1
+1 −C2k =
q2k 1q2k
.
Bang quy nạp) , ta có qk ≥ k. Do đó
lim C2k 1 C2k = 0.
k→∞
V y α1 = α2. Định lí đưoc chứng minh.
50
Viết đề tài giá sinh viên – ZALO:0973.287.149-TEAMLUANVAN.COM
+ q2 q
n+1
2
n
+
⇔ α −
q2n
⇔ α −
q2n
Ket lu n quan trong cho ứng dụng là
Định lí 3.2.2. α = [a0;a1;a2,...] là m t so vô tý.
Chŕng minh. Ta sẽ chứng minh bang phản chứng. Giả sử trái lại α = a/b ∈ Q.
Theo Định lí 3.1.7 ta có C2n < α < C2n+1. V y
0 < α −C2n < C2n 1 −C2n =
1
.
Đieu này tương đương vói
0 < α −
p2n
<
1
q2n q2n+1q2n
1
⇔ 0 < αq2n − p2n <
q2n 1
1
0 < q2n bp2n <
+1
1
1 < q2n bp2n < .
+1
Cho k → ∞ ta có đieu mâu thuȁn. Như v y phép chứng minh đưoc ket thúc.
Định lí 3.2.3. Mői so vô tý đeu bieu dien m t cách duy nhat dưới dạng m t liên
phân so vô hạn.
3.3 Liên phân so vô hạn tuan hoàn
Ta goi liên phân so vô hạn [a0; a1, a2, . . .] là tuan hoàn neu dãy (an)là tuan hoàn ke
từ m t chỉ so nào đó tức là: ton tại so nguyên dương m và k vói moi n ≥ m ta có
an = an+k. So nguyên dương k đưoc goi là chu kỳ. Trong trưòng hop đó ta viet
[a0;a1,a2,...,am−1,am,am+1,...,am+k−1]
Bài toán đ t ra là đ c trưng tat cả các so vô tý có bieu dien liên phân so vô hạn
tuan hoàn. Ta có khái ni m sau
51
Viết đề tài giá sinh viên – ZALO:0973.287.149-TEAMLUANVAN.COM
Định nghĩa 3.3.1. So vô tý α đưoc goi là so vô tý b c hai neu nó là nghi m của
m t tam thức b c hai vói h so nguyên.
Ví dụ 3.3.2. So vô tý α = 2+
√
3 là so vô tý b c hai vì nó là nghi m của x2
−4x+
1 = 0.
Bo đe 3.3.3. So thực α là vô tý b c hai neu và chỉ neu có ton tại các so nguyên
a,b,c với b > 0 và không chính phương, c /= 0 sao cho
a +
√
b
a =
c
.
Chŕng minh. Giả sử α là so vô tý b c hai. Khi đó ton tại các so nguyên A,B,C sao
cho α là nghi m của phương trình Ax2
+Bx+C = 0. V y
α ==
−B ±
√
B2 − AC
2A
Đ t a = −B, b = B2
− 4AC, c = 2A ho c a = B, b = B2
−4AC, c = −2A ngưoc lại
neu
α =
a +
√
b
.
c
Thì α là so vô tý và nó là nghi m của phương trình b c hai c2
x2
− 2acx + a2
− b =
0.
Bo đe 3.3.4. Neu α là so vô tý b c hai thì (rα +s)/(tα +u) cũng là so vô tý b c
hai neu r,s,t,u là các so nguyên.
Chŕng minh. Giả sử
α =
a +
√
b
.
c
Tính toán cho ta
rα +s
=
(ar +c)(at + cu)−rtb+ (r(at +cu)−t(ar +cs))
√
b
.
tα + u (at + cu)2
− tb2
.
52
Viết đề tài giá sinh viên – ZALO:0973.287.149-TEAMLUANVAN.COM
Định nghĩa 3.3.5. So vô tý
α =
a −
√
b
.
c
Đưoc goi là liên hop của α và ký hi u là α′.
Bo đe 3.3.6. Neu so vô tý b c hai α là nghi m của phương trình Ax2
+Bx +C = 0
thì liên hợp của nó cũng là nghi m của phương trình đó.
Chŕng minh. Th t v y
α + α′ = 2a/c = −BA, (α)(α′) = a2 − b/c2
= C/A.
Bang phép tính ta de thay
Bo đe 3.3.7. Ta có các h thŕc sau:
(α ±β)′ = α′ ±β′,
(αβ)′ = α′β ′,
α
′
=
α′
.
Ta có định lý cơ bản sau đây do Lagrange tìm ra
Định lí 3.3.8. So vô tý α có bieu dien liên phân so tuan hoàn khi và chỉ khi nó là
so vô tý b c hai.
Chŕng minh. Trưóc het ta chứng minh rang neu α có bieu dien liên phân so tuan
hoàn thì nó là so vô tý b c hai.
Giả sử
Đ t
[a0;a1,a2,...,am−1,am,am+1,...,am+k−1]
β = [am,am+1,...,am+k−1]
β β′
53
Viết đề tài giá sinh viên – ZALO:0973.287.149-TEAMLUANVAN.COM
y
Khi đó β = [am,am+1,...,am+k,β] do đó
β =
βPk
+ pk−1
. (3.1)
βPk + pk−1
trong đó pk/qk và pk−1/qk−1 là hai giản phân cuoi của [am, am+1, . . . am+k]. Từ
công thức (3.1) suy ra
qkβ2
+ (qk−1 − pk)β − pk−1 = 0.
V y β là so vô tý b c hai. Ta lại có α = [a0;a1;a2,...,am−1,β] do đó
α =
β pm−1 + pm−2
.
βqm−1 + qm−2
Do đó theo Bo đe 3.3.6 ta có α là so vô tý b c hai.
Ví dụ sau đây minh hoạ cách tìm so vô tý b c hai từ bieu dien liên phân so tuan
hoàn của nó.
Ví dụ 3.3.9. Tìm x biet rang x = [3;1,2].
Ta có x = [3;y] vói y = 1,2. Ta có y = [1;2,y] do đó
1
= 1 +
2 + 1
3y + 1
=
2y +1
.
Suy ra 2y2
− 2y − 1 = 0. Vì y > 0 nên y = (1 +
√
3)/2. Vì x = 3 + 1/y nên ta tìm
đưoc
2 4+
√
2
x = 3 +
1 +
√
2
=
2
.
Đe chứng minh phan ngưoc lại ta can bo đe sau
Bo đe 3.3.10. Neu α là so vô tý b c hai thì nó có the bieu dien dưới dạng
P+
√
d
α =
Q
trong đó P,Q,d là các so nguyên sao cho Q | (d −P2
).
54
Viết đề tài giá sinh viên – ZALO:0973.287.149-TEAMLUANVAN.COM
Chŕng minh. Ta có α = (a +
√
b)/c. Nhân cả tử và mȁu vói |c| ta đưoc α =
(a|c|+)/c|c| . Đ t P = a|c|, d = bc2
, Q = c|c| = ±c2
. Khi đó d − P2
= c2
(b − a2)
chia het Q = ±c2
.
Giả sử α = α0 là so vô tý b c hai. Ta xây dụng dãy (a0, a1, a2 . . .) như sau.
Theo Bo đe 3.3.10 ta có các so nguyên P0,Q0 và d sao cho α0 = (P0 +
√
d)/Q0.
Q0 | (d − P). Ta đ t a0 = [α0] và xác định P1 = a0Q0 − P1,Q1 = (d − P)/Q0,
α1 = (P1)/Q1. Tiep đó đ t a1 = [α1]. M t cách tong quát neu có
Pk +
√
d
Pk ∈ Z, Qk ∈ Z,Qk | (d −P), ak =
Qk
ak = [αk].
Ta sẽ đ t
Pk+1 = akQk − Pk, Qk+1 = (d −P)/Qk, ak+1 = ak+1 = [αk+1].
Khi đó tính toán cho thay
sQk+1 = (d −P)/Qk +(2kPk −aQk)
do đó Qk+1 ∈ Z và vì Qk+1Qk = (d − P) nên Qk+1 | (d − P). Có the chứng minh
đưoc rang
α = [a0,a1,a2,...]
và hơn nữa dãy (an) xác định như trên là tuan hoàn.
Ví dụ 3.3.11. Khai trien liên phân so của so α = (6+
√
28)/4.
Chŕng minh. Ta có P0 = 6, Q0 = 4, d = 28, 4 | (28−62
) = −8, a0 = (6+
√
28)/4,
a0 = [α0] = 2 và
P1 = 2.4−6 = 2, Q1 = (28−22
)/4 = 6, α1 = (2+
√
28)/6, a1 = [α1] = 1
P2 = 1.6 − 2 − 4 = 4, Q2 = (28 − 42
)/6 = 2,α2 = (4 +
√
28)/2,a2 = [α2] = 4
P3 = 4.2 − 4 = 4,Q3 = (28 − 42
)/2 = 6,α3 = (4 +
√
28)/6,a3 = [α3] = 1
P4 = 1.6 − 4 = 2,Q4 = (28 − 22
)/6 = 4,α4 = (2 +
√
28)/4,a4 = [α4] = 1
55
Viết đề tài giá sinh viên – ZALO:0973.287.149-TEAMLUANVAN.COM
P5 = 1.4 − 2 = 2,Q5 = (28 − 22
)/6 = 4,α5 = (2 +
√
28)/4,a5 = [α4] = 1
Ta thay P1 = P5, Q1 = Q5 do đó a1 = a5 và dãy tuan hoàn chu kỳ 4. Ta có
6+
√
28
4
= [2;1,4,1,1,]
Tiep theo ta muon tìm đieu ki n đe so vô tý b c hai có bieu dien liên phân
so tuan hoàn ngay từ đau, tức là đieu ki n đe ton tại so nguyên dương k sao cho
an = an+k vói moi n ≥ 0. Ta có định lý sau
Định lí 3.3.12. So vô tý b c hai α có bieu dien tuan hoàn ngay tr đau neu và chỉ
neu α > 1 và −1 < α′ < 0.
Chứng minh định lý này khá phức tạp nên ta bỏ qua.
Bây giò ta sẽ xác định bieu dien liên phân so của
√
d.
Xét so α =
√
d + [
√
d]. Ta có α′ = [
√
d] −
√
d do đó α > 1 và −1 < α′ < 0.
V y α có bieu dien tuan hoàn ngay từ đau. So hạng đau tiên a0 = [
√
d + [
√
d]] =
2[
√
d] = 2a vói a = [
√
d]. Ta có
√
d +a =
√
d + [
√
d] = [2a;a1,a2,...,an]
= [2a; a1,a2,... an,2a; a1,a2,...,an]
Suy ra
√
d = [a; a1,a2,..., an,2a].
Phân tích can th n hơn ta còn có the chứng minh đưoc
a1 = an, a2 = an−1,...
tức là dãy (a1,...,an) đoi xứng, tức là nó có dạng
√
d = [a;a1;a2,...,a2,a1,2a]
ỏ đó a = [
√
d].
56
Viết đề tài giá sinh viên – ZALO:0973.287.149-TEAMLUANVAN.COM
3.4 Áp dụng vào phương trình Diophante
3.4.1 Phương trình b c nhat hai an Ax+ By = C
Chúng ta biet rang phương trình có nghi m neu và chỉ neu d = (A,B) là ưóc của
C. Trong trưòng hop này giả sử A = ad, B = bd, C = cd thì (a,b) = 1 và phương
trình đã cho tương đương vói
ax + by = c. (3.2)
Neu (x0, y0) là m t nghi m của (3.2) thì tat cả các nghi m (x, y) của (3.2) đưoc
cho bỏi công thức x = x0 + bt; y = y0 − at. Như v y vi c giải phương trình (3.2)
quy ve tìm m t nghi m (x0,y0) của nó.
Xét phương trình
ax + by = 1. (3.3)
Neu (x0,y0) là m t nghi m của (3.3) thì (cx0,cy0) là nghi m của (3.2). Thành thử
ta quy ve bài toán: Cho (a,b) = 1. Hãy tìm m t nghi m của phương trình (3.3).
Ta bieu dien so a/|b| thành liên phân so hữu hạn
a
|b|
= [a0;a1;a2;......... ; an].
Goi pn−1/qn−1 và pn/qn là hai giản phân cuoi cùng của liên phân so này. Ta có
a/|b| = pn/qn, (a, b) = 1, (pn, qn) = 1 nên a = pn, |b| = qn. Theo Định lý 3.3.8 ta
có pnqn−1 − pn−1qn = (−1)n−1
suy ra aqn−1 − |b|qn−1 = (−1)n−1
. Đieu này kéo
theo a(−1)n−1
qn−1 +|b|(−1)n−1
pn−1 = 1.
V y, neu b > 0 thì phương trình (3.3) có m t nghi m là
x = (−1)n−1
qn−1, y = (−1)n
pn−1.
Neu b < 0 thì phương trình (3.3) có m t nghi m là
x = (−1)n−1
qn−1, y = (−1)n−1
pn−1.
57
Viết đề tài giá sinh viên – ZALO:0973.287.149-TEAMLUANVAN.COM
d −P
Qk
k k
Ví dụ 3.4.1. Giải phương trình 342x − 123y = 15.
Lời giái. Vì (342,123) = 5 nên phương trình đã cho tương đương vói 114x−41y =
5.
Ta bieu dien so 114/41 thành liên phân so. Ta có
114 = 2 · 41 + 32 41 = 1 · 32 + 932 = 3 · 9 + 5 9 = 1 · 5 + 45 = 1 · 4 + 1 4 = 4 · 1.
Do v y
114
41
= [2;1,3,1,1,4].
Ta có n = 5, p4/p4 = [2; 1, 3, 1] = 25/9. Vì b = −41 < 0 nên m t nghi m của
phương trình 114x − 41y = 1 là x = q4 = 9, y = 25. Suy ra m t nghi m của phương
trình 114x − 41y = 5 là x = 5 · 9 = 54, y = 5 · 25 = 125. Nghi m tong quát của
phương trình đã cho là:
x = 45 + 41t, y = 125 + 114t vói .t ∈ Z
3.4.2 Phương trình x2
− dy2
= ±1
Nhac lại, ta goi phương trình Pell là các phương trình có dạng
x2
− dy2
= ±1.
Bo đe 3.4.2. Cho d là so không chính phương. Giá sr Pk, Qk, αk, ak là các so xác
định trong vi c tìm khai trien liên phân so của
√
d
Pk +
√
d
αk =
Qk
, ak = [αk],
Pk+1 = akqk − pk, Qk+1 =
2
k+1
.
Qk
αk+1
Pk+1 +
√
d
= ,
+1
k+1 = [αk+1].
Giá sr pk/qk là gián phân thŕ k của
√
d. Khi đó
p2
− dq2
= (−1)k−1
Qk+1.
a
58
Viết đề tài giá sinh viên – ZALO:0973.287.149-TEAMLUANVAN.COM
Q
2
2tr−1
k k
Chŕng minh. Vì
√
d = α0 = [a0;a1,a2,...,ak,αk+1] nên ta có
√
d = α0 =
αk+1 pk + pk−1
.
αk+1qk + qk−1
Vì αk+1
= Pk+1+
√
d
ta có
k+1
√ (Pk+1 +
√
d)pk +Qk+1 pk−1
Do đó
d =
(Pk+1
+
√
d)pk
+ Qk+1
.
qk−1
npk = (Pk+1qk + Qk+1qk−1)
√
d = (Pk+1 pk + Qk+1 pk−1) + pk
√
d.
Từ đó suy ra (do
√
d /∈ Q)
nqk = Pk+1 pk + Qk+1 pk−1, Pk+1qk + Qk+1qk−1 = pk.
Từ đó (nhân phương trình đau vói qk, phương trình thứ hai vói pk, roi trừ cho nhau
ta đưoc.
p2
− dq2
= (pkqk−1 − pk−1qk)Qk−1 = (−1)k−1
Qk+1.
Ta có đieu phải chứng minh.
Định lí 3.4.3. Giá sr chu kỳ của bieu dien liên phân so của
√
d là r. Goi pk/qklà
gián phân thŕ k của
√
d. Neu r chȁn thì x = ptr−1, y = qtr−1 với t = 1,2... là
nghi m của phương trình Pellx2
− dy2
= 1. Neu r lé thì x = p2tr−1, y = q2tr−1, với
t = 1,2,... là nghi m của phương trình Pell x2
−dy2
= 1.
Chŕng minh. Vì
√
d = 0 +
√
d/1 nên Q0 = 1, suy ra Qkr = Q0 = 1 vói moi k.
Theo Bo đe 3.4.2 ta có
2
kr−1 — dqkr−1 = (−1)kr−2
Qkr = (−1)kr
.
Như v y neu n chȁn thì p2
kr−1
2
kr−1 = 1 vói moi k ∈ N, neu r lẻ thì p2
−
2
2tr−1 = 1.
p
— dq
dq
59
Viết đề tài giá sinh viên – ZALO:0973.287.149-TEAMLUANVAN.COM
r 1
.
.
α −
.
< .
1
≤
|spk − rqk|
= . pk
−
r.
. qk
1
.
s
.
1
Bo đe 3.4.4. Ta có Qi
cho r.
1 với moi i = 1,2... và Qk = 1 khi và chỉ khi k chia het
Đe chứng minh đây là tat cả các nghi m của phương trình Pell ta can các bo đe
sau
Bo đe 3.4.5. Cho α là m t so vô tý và r/s là so hru tý toi gián với r > 0 và
.
s
.
2s2
Khi đó r/s phái là m t gián phân của α.
Chŕng minh. Giả sử r/s không là giản phân khi đó ton tại k sao cho
qk ≤ s < qk+1.
Theo Bo đe 3.4.2 ta có
|qkα − pk| ≤ |sα − r| = s|α − r/s| < 1/(2s).
Suy ra
Vì |spk − rqk| ≥ 1 nên ta có
|α − pk/qk| < 1/(2sqk).
sqk sqk
.qk s.
= .α − k .+
.
α +
.
<
2sqk
+
2s2
.
V y 1
< 1
, tức là qk > s. Đieu mâu thuȁn này đã chứng minh phát bieu của bo
2sqk 2s2
đe là đúng.
Bo đe 3.4.6. Giá sr x, y là các so nguyên dương sao cho x2
−dy2
= n và |n| <
√
d.
Khi đó x/y là m t gián phân của
√
d.
p r
60
Viết đề tài giá sinh viên – ZALO:0973.287.149-TEAMLUANVAN.COM
y
i i
Chŕng minh. Xét trưòng hop n > 0. Ta có (x + y
√
d)(x − y
√
d) = n suy ra x >
y
√
d. V y 0 < x
−
√
d. Ta có
Lại có
x √ x −
√
d x2
− dy2
n
√
d 1
y
− d =
y
=
y(x + y
√
d
<
y(2y
√
d
<
2y2
√
d
=
2y2
.
Theo Bo đe 3.4.5 thì x/y là m t giản phân của
√
d.
Giả sử n < 0. Khi đó
y2
− (1/d)x2
= −n/d.
Ta có −n/d > 0, −|n|/d < 1/
√
d. V y theo bưóc trưóc y/x là m t giản phân của
1/
√
d. Nhưng khi đó x/y = 1/(y/x) là m t giản phân của 1/(1/(
√
d) =
√
d. Phép
chứng minh định lí đưoc ket thúc.
Định lí 3.4.7. Cho phương trình Pell
x2
− dy2
= 1.
Goi r là chu kỳ của bieu dien liên phân so của
√
d.
Neu r chȁn thì tat cá các nghi m của phương trình Pell là x = pkr−1, y = qkr−1.
Neu r lé thì tat cá các nghi m của phương trình Pell là x = p2tr−1, y = q2tr−1
với t ∈ N∗.
Chŕng minh. Giả sử (x,y) là nghi m của phương trình Pell. Theo Bo đe 3.4.6, ton
tại i đe x = pi, y = qi. Từ đó
p2
− dq2
= 1.
Từ Bo đe 3.4.2 rút ra (−1)i−1
Qi+1 = 1. Suy ra Qi+1 = ±1. Vì qk+1 /= −1 nên
Qi+1 = 1 và i lẻ. Theo Bo đe 3.4.4 ta rút ra ton tại ki + 1 = kr, kéo theo i = kr − 1
vàkr chȁn. Thành thử neu r lẻ thì k chȁn, k = 2t. Phép chứng minh định lí đưoc ket
thúc.
61
Viết đề tài giá sinh viên – ZALO:0973.287.149-TEAMLUANVAN.COM
i i
Xét phương trình
Ta có ket quả sau
x2
− dy2
= −1 (3.4)
Định lí 3.4.8. Phương trình x2
− dy2
= −1 có nghi m khi và chỉ khi chu kỳ r của
bieu dien liên phân so của
√
d là so lé. Trong trường hợp ay các nghi m của nó là
x = p(2tr−r−1), y = q(2tr−r−1) với t = 1,2,....
Chŕng minh. Từ Bo đe 3.4.2 de thay neu chu kỳ r của bieu dien liên phân so của
√
d là so lẻ thì x = p(2tr−r−1) , y = q(2tr−r−1) vói t = 1,2,... là nghi m.
Giả sử (x,y) là nghi m của phương trình (3.4). Theo Bo đe 3.4.6 ton tại i đe
x = pi, y = qi. Từ đó
p2
− dq2
= −1.
Từ Bo đe 3.4.2 ta rút ra (−1)i−1
Qi+1 = −1, suy ra Qi+1 = ±1. Vì Qi+1 /= −1
nên Qi+1 = 1 và i chȁn. Theo Bo đe 3.4.2 ton tại k ∈ N sao choi + 1 = kr, suy ra
i = kr − 1 và kr lẻ. Thành thử neu r chȁn thì kr luôn chȁn do đó phương trình vô
nghiêm.
Trong trưòng hop r lẻ lý lu n tương tụ như trong trưòng hop phương trình Pell
x2
− dy2
= 1 tat cả các nghi m phải có dạng x = pkr−1, y = qkr−1 trong đó kr lẻ
tức là khi k lẻ hay x = p(2tr−r−1), y = q(2tr−r−1) vói t = 1, 2, . . . Phép chứng minh
định lí đưoc ket thúc.
62
Viết đề tài giá sinh viên – ZALO:0973.287.149-TEAMLUANVAN.COM
Ket lu n
1 Nh ng ket qua đã đạt đư c
Lu n văn “M t so lớp phương trình Diophantine” đã đạt đưoc các ket quả sau:
1. Trình bày các kien thức cơ sỏ ve phương trình Diophantine tuyen tính (b c
nhat hai an và nhieu an).
2. Trình bày ve m t so phương trình Diophantine phi tuyen, như Phương trình
Pell loại 1, loại 2, phương trình Pell tong quát, Phương trình Pythagoras-
Fermat.
3. Úng dụng của liên phân so trong phương trình Diophantine.
2 Đe xuat m t so hưỚng nghiên c u tiep theo
Phương trình Diophantine là m t trong những chủ đe rat r ng của Lý thuyet so.
Sau những ket quả đã đạt đưoc trong lu n văn, chúng tôi hi vong và co gang sẽ tiep
tục nghiên cứu các chủ đe liên quan, chȁng hạn:
• Các phương pháp và kỹ thu t khác đe tan công các phương trình Diophan-
tine, ví dụ phương pháp xuong thang, các phương pháp so hoc, đại so và
hình hoc, ...
• Nghiên cứu ve các các lóp phương trình Diophantine phi tuyen khác như
phương trình có chứa hàm mũ.
63
Viết đề tài giá sinh viên – ZALO:0973.287.149-TEAMLUANVAN.COM
Tài li u tham khao
Tieng Vi t
[1] A.D. Aczel (2001), Câu chuy n hap dȁn ve bài toán Ferma (Ngưòi dịch: Tran
Văn Nhung, Đő Trung H u, Nguyen Kim Chi), NXB Giáo dục.
[2] Vũ Ngoc Khánh (2015), M t so van đe ve phương trình Diophantine, Lu n
văn thạc sĩ toán hoc,
[3] Hà Huy Khoái (2004), So hoc, NXB Giáo dục.
[4] Đ ng Hùng Thang (2010), Bài giáng so hoc, NXB Giáo dục.
Tieng Anh
[5] K.H. Rosen (1986), Elementary Number Theory and its Applications,
Addison–Wesley Publishing Company.
[6] W. Siepinski (1964), Elementary Theory of Number, North-Holland Mathe-
matical Library (volume 31).

More Related Content

Similar to M T So L P Phương Trình Diophantine.docx

Về phương trình hàm Loại giá trị trung bình và áp dụng.docx
Về phương trình hàm Loại giá trị trung bình và áp dụng.docxVề phương trình hàm Loại giá trị trung bình và áp dụng.docx
Về phương trình hàm Loại giá trị trung bình và áp dụng.docx
DV Viết Luận văn luanvanmaster.com ZALO 0973287149
 
Bài Toán Phân Hoạch So Nguyên Dương.docx
Bài Toán Phân Hoạch    So Nguyên Dương.docxBài Toán Phân Hoạch    So Nguyên Dương.docx
Bài Toán Phân Hoạch So Nguyên Dương.docx
DV Viết Luận văn luanvanmaster.com ZALO 0973287149
 
Bat Đang Thức Và Bài Toán Cực Tr± Trong L P Các Đa Thức Và Phân Thức H So Ngu...
Bat Đang Thức Và Bài Toán Cực Tr± Trong L P Các Đa Thức Và Phân Thức H So Ngu...Bat Đang Thức Và Bài Toán Cực Tr± Trong L P Các Đa Thức Và Phân Thức H So Ngu...
Bat Đang Thức Và Bài Toán Cực Tr± Trong L P Các Đa Thức Và Phân Thức H So Ngu...
DV Viết Luận văn luanvanmaster.com ZALO 0973287149
 
Bat Đang Thức V I Hàm Loi B Ph N Và Ứng Dụng.docx
Bat Đang Thức V I Hàm Loi B Ph N Và Ứng Dụng.docxBat Đang Thức V I Hàm Loi B Ph N Và Ứng Dụng.docx
Bat Đang Thức V I Hàm Loi B Ph N Và Ứng Dụng.docx
DV Viết Luận văn luanvanmaster.com ZALO 0973287149
 
Xấp xỉ hàm đa điều hòa dưới Bởi hàm green đa cực.doc
Xấp xỉ hàm đa điều hòa dưới Bởi hàm green đa cực.docXấp xỉ hàm đa điều hòa dưới Bởi hàm green đa cực.doc
Xấp xỉ hàm đa điều hòa dưới Bởi hàm green đa cực.doc
DV Viết Luận văn luanvanmaster.com ZALO 0973287149
 
Bồi dưỡng học sinh giỏi Toán Tiểu Học
Bồi dưỡng học sinh giỏi Toán Tiểu HọcBồi dưỡng học sinh giỏi Toán Tiểu Học
Bồi dưỡng học sinh giỏi Toán Tiểu Học
Bồi dưỡng Toán tiểu học
 
Ve H Phương Trình Phi Tuyen Và Ứng Dụng.docx
Ve H Phương Trình Phi Tuyen Và Ứng Dụng.docxVe H Phương Trình Phi Tuyen Và Ứng Dụng.docx
Ve H Phương Trình Phi Tuyen Và Ứng Dụng.docx
DV Viết Luận văn luanvanmaster.com ZALO 0973287149
 
Cđ đồng dư thức trong toán 7
Cđ đồng dư thức trong toán 7Cđ đồng dư thức trong toán 7
Cđ đồng dư thức trong toán 7
Cảnh
 
Luận văn thạc sĩ - Đa thức trong các bài toán thi học sinh giỏi.doc
Luận văn thạc sĩ - Đa thức trong các bài toán thi học sinh giỏi.docLuận văn thạc sĩ - Đa thức trong các bài toán thi học sinh giỏi.doc
Luận văn thạc sĩ - Đa thức trong các bài toán thi học sinh giỏi.doc
Dịch vụ viết thuê đề tài trọn gói ☎☎☎ Liên hệ ZALO/TELE: 0973.287.149 👍👍
 
Ứng Dụng Hình Học Giải Tích Vào Giải Phương Trình, Bất Phương Trình Và Hệ Phư...
Ứng Dụng Hình Học Giải Tích Vào Giải Phương Trình, Bất Phương Trình Và Hệ Phư...Ứng Dụng Hình Học Giải Tích Vào Giải Phương Trình, Bất Phương Trình Và Hệ Phư...
Ứng Dụng Hình Học Giải Tích Vào Giải Phương Trình, Bất Phương Trình Và Hệ Phư...
Dịch vụ viết đề tài trọn gói 0934.573.149
 
Bat đang thức trong so hoc và m t so Dạng toán liên quan.docx
Bat đang thức trong so hoc và m t so Dạng toán liên quan.docxBat đang thức trong so hoc và m t so Dạng toán liên quan.docx
Bat đang thức trong so hoc và m t so Dạng toán liên quan.docx
DV Viết Luận văn luanvanmaster.com ZALO 0973287149
 
Luận văn: Nghiên cứu DIDACTIC về dạy học các bài toán tối ưu trong chủ đề giả...
Luận văn: Nghiên cứu DIDACTIC về dạy học các bài toán tối ưu trong chủ đề giả...Luận văn: Nghiên cứu DIDACTIC về dạy học các bài toán tối ưu trong chủ đề giả...
Luận văn: Nghiên cứu DIDACTIC về dạy học các bài toán tối ưu trong chủ đề giả...
Viết thuê trọn gói ZALO 0934573149
 
Các ước so của so Mersenne.docx
Các ước so của so Mersenne.docxCác ước so của so Mersenne.docx
Các ước so của so Mersenne.docx
DV Viết Luận văn luanvanmaster.com ZALO 0973287149
 
Bat Phương Trình Hàm Sinh B I Các Đại Lư Ng Trung Bình B C Tùy Ý Và Các Dạng ...
Bat Phương Trình Hàm Sinh B I Các Đại Lư Ng Trung Bình B C Tùy Ý Và Các Dạng ...Bat Phương Trình Hàm Sinh B I Các Đại Lư Ng Trung Bình B C Tùy Ý Và Các Dạng ...
Bat Phương Trình Hàm Sinh B I Các Đại Lư Ng Trung Bình B C Tùy Ý Và Các Dạng ...
DV Viết Luận văn luanvanmaster.com ZALO 0973287149
 
M T So Dạng Toán Ve Dãy So Sinh B I Các Hàm So Sơ Cap.docx
M T So Dạng Toán Ve Dãy So Sinh B I Các Hàm So Sơ Cap.docxM T So Dạng Toán Ve Dãy So Sinh B I Các Hàm So Sơ Cap.docx
M T So Dạng Toán Ve Dãy So Sinh B I Các Hàm So Sơ Cap.docx
DV Viết Luận văn luanvanmaster.com ZALO 0973287149
 
Hàm Đơn Đi U, Tựa Đơn Đi U Và M T So Ứng Dụng Của Phép Đơn Đi U Hóa Hàm So.docx
Hàm Đơn Đi U, Tựa Đơn Đi U Và M T So Ứng Dụng Của Phép Đơn Đi U Hóa Hàm So.docxHàm Đơn Đi U, Tựa Đơn Đi U Và M T So Ứng Dụng Của Phép Đơn Đi U Hóa Hàm So.docx
Hàm Đơn Đi U, Tựa Đơn Đi U Và M T So Ứng Dụng Của Phép Đơn Đi U Hóa Hàm So.docx
DV Viết Luận văn luanvanmaster.com ZALO 0973287149
 
Bài Toán Đổi Tiền Của Frobenius.docx
Bài Toán Đổi Tiền Của Frobenius.docxBài Toán Đổi Tiền Của Frobenius.docx
Bài Toán Đổi Tiền Của Frobenius.docx
DV Viết Luận văn luanvanmaster.com ZALO 0973287149
 
Bất đẳng thức Trong lớp các hàm lượng giác và lượng giác ngược.docx
Bất đẳng thức Trong lớp các hàm lượng giác và lượng giác ngược.docxBất đẳng thức Trong lớp các hàm lượng giác và lượng giác ngược.docx
Bất đẳng thức Trong lớp các hàm lượng giác và lượng giác ngược.docx
DV Viết Luận văn luanvanmaster.com ZALO 0973287149
 
Xây Dựng Hệ Thống Phân Lịch Thi Tín Chỉ Tại Trường Cao Đẳng Thương Mại Đà Nẵn...
Xây Dựng Hệ Thống Phân Lịch Thi Tín Chỉ Tại Trường Cao Đẳng Thương Mại Đà Nẵn...Xây Dựng Hệ Thống Phân Lịch Thi Tín Chỉ Tại Trường Cao Đẳng Thương Mại Đà Nẵn...
Xây Dựng Hệ Thống Phân Lịch Thi Tín Chỉ Tại Trường Cao Đẳng Thương Mại Đà Nẵn...
Dịch vụ viết thuê Luận Văn - ZALO 0932091562
 
Vmo 2015-solution-1421633776
Vmo 2015-solution-1421633776Vmo 2015-solution-1421633776
Vmo 2015-solution-1421633776
Nguyen Van Tai
 

Similar to M T So L P Phương Trình Diophantine.docx (20)

Về phương trình hàm Loại giá trị trung bình và áp dụng.docx
Về phương trình hàm Loại giá trị trung bình và áp dụng.docxVề phương trình hàm Loại giá trị trung bình và áp dụng.docx
Về phương trình hàm Loại giá trị trung bình và áp dụng.docx
 
Bài Toán Phân Hoạch So Nguyên Dương.docx
Bài Toán Phân Hoạch    So Nguyên Dương.docxBài Toán Phân Hoạch    So Nguyên Dương.docx
Bài Toán Phân Hoạch So Nguyên Dương.docx
 
Bat Đang Thức Và Bài Toán Cực Tr± Trong L P Các Đa Thức Và Phân Thức H So Ngu...
Bat Đang Thức Và Bài Toán Cực Tr± Trong L P Các Đa Thức Và Phân Thức H So Ngu...Bat Đang Thức Và Bài Toán Cực Tr± Trong L P Các Đa Thức Và Phân Thức H So Ngu...
Bat Đang Thức Và Bài Toán Cực Tr± Trong L P Các Đa Thức Và Phân Thức H So Ngu...
 
Bat Đang Thức V I Hàm Loi B Ph N Và Ứng Dụng.docx
Bat Đang Thức V I Hàm Loi B Ph N Và Ứng Dụng.docxBat Đang Thức V I Hàm Loi B Ph N Và Ứng Dụng.docx
Bat Đang Thức V I Hàm Loi B Ph N Và Ứng Dụng.docx
 
Xấp xỉ hàm đa điều hòa dưới Bởi hàm green đa cực.doc
Xấp xỉ hàm đa điều hòa dưới Bởi hàm green đa cực.docXấp xỉ hàm đa điều hòa dưới Bởi hàm green đa cực.doc
Xấp xỉ hàm đa điều hòa dưới Bởi hàm green đa cực.doc
 
Bồi dưỡng học sinh giỏi Toán Tiểu Học
Bồi dưỡng học sinh giỏi Toán Tiểu HọcBồi dưỡng học sinh giỏi Toán Tiểu Học
Bồi dưỡng học sinh giỏi Toán Tiểu Học
 
Ve H Phương Trình Phi Tuyen Và Ứng Dụng.docx
Ve H Phương Trình Phi Tuyen Và Ứng Dụng.docxVe H Phương Trình Phi Tuyen Và Ứng Dụng.docx
Ve H Phương Trình Phi Tuyen Và Ứng Dụng.docx
 
Cđ đồng dư thức trong toán 7
Cđ đồng dư thức trong toán 7Cđ đồng dư thức trong toán 7
Cđ đồng dư thức trong toán 7
 
Luận văn thạc sĩ - Đa thức trong các bài toán thi học sinh giỏi.doc
Luận văn thạc sĩ - Đa thức trong các bài toán thi học sinh giỏi.docLuận văn thạc sĩ - Đa thức trong các bài toán thi học sinh giỏi.doc
Luận văn thạc sĩ - Đa thức trong các bài toán thi học sinh giỏi.doc
 
Ứng Dụng Hình Học Giải Tích Vào Giải Phương Trình, Bất Phương Trình Và Hệ Phư...
Ứng Dụng Hình Học Giải Tích Vào Giải Phương Trình, Bất Phương Trình Và Hệ Phư...Ứng Dụng Hình Học Giải Tích Vào Giải Phương Trình, Bất Phương Trình Và Hệ Phư...
Ứng Dụng Hình Học Giải Tích Vào Giải Phương Trình, Bất Phương Trình Và Hệ Phư...
 
Bat đang thức trong so hoc và m t so Dạng toán liên quan.docx
Bat đang thức trong so hoc và m t so Dạng toán liên quan.docxBat đang thức trong so hoc và m t so Dạng toán liên quan.docx
Bat đang thức trong so hoc và m t so Dạng toán liên quan.docx
 
Luận văn: Nghiên cứu DIDACTIC về dạy học các bài toán tối ưu trong chủ đề giả...
Luận văn: Nghiên cứu DIDACTIC về dạy học các bài toán tối ưu trong chủ đề giả...Luận văn: Nghiên cứu DIDACTIC về dạy học các bài toán tối ưu trong chủ đề giả...
Luận văn: Nghiên cứu DIDACTIC về dạy học các bài toán tối ưu trong chủ đề giả...
 
Các ước so của so Mersenne.docx
Các ước so của so Mersenne.docxCác ước so của so Mersenne.docx
Các ước so của so Mersenne.docx
 
Bat Phương Trình Hàm Sinh B I Các Đại Lư Ng Trung Bình B C Tùy Ý Và Các Dạng ...
Bat Phương Trình Hàm Sinh B I Các Đại Lư Ng Trung Bình B C Tùy Ý Và Các Dạng ...Bat Phương Trình Hàm Sinh B I Các Đại Lư Ng Trung Bình B C Tùy Ý Và Các Dạng ...
Bat Phương Trình Hàm Sinh B I Các Đại Lư Ng Trung Bình B C Tùy Ý Và Các Dạng ...
 
M T So Dạng Toán Ve Dãy So Sinh B I Các Hàm So Sơ Cap.docx
M T So Dạng Toán Ve Dãy So Sinh B I Các Hàm So Sơ Cap.docxM T So Dạng Toán Ve Dãy So Sinh B I Các Hàm So Sơ Cap.docx
M T So Dạng Toán Ve Dãy So Sinh B I Các Hàm So Sơ Cap.docx
 
Hàm Đơn Đi U, Tựa Đơn Đi U Và M T So Ứng Dụng Của Phép Đơn Đi U Hóa Hàm So.docx
Hàm Đơn Đi U, Tựa Đơn Đi U Và M T So Ứng Dụng Của Phép Đơn Đi U Hóa Hàm So.docxHàm Đơn Đi U, Tựa Đơn Đi U Và M T So Ứng Dụng Của Phép Đơn Đi U Hóa Hàm So.docx
Hàm Đơn Đi U, Tựa Đơn Đi U Và M T So Ứng Dụng Của Phép Đơn Đi U Hóa Hàm So.docx
 
Bài Toán Đổi Tiền Của Frobenius.docx
Bài Toán Đổi Tiền Của Frobenius.docxBài Toán Đổi Tiền Của Frobenius.docx
Bài Toán Đổi Tiền Của Frobenius.docx
 
Bất đẳng thức Trong lớp các hàm lượng giác và lượng giác ngược.docx
Bất đẳng thức Trong lớp các hàm lượng giác và lượng giác ngược.docxBất đẳng thức Trong lớp các hàm lượng giác và lượng giác ngược.docx
Bất đẳng thức Trong lớp các hàm lượng giác và lượng giác ngược.docx
 
Xây Dựng Hệ Thống Phân Lịch Thi Tín Chỉ Tại Trường Cao Đẳng Thương Mại Đà Nẵn...
Xây Dựng Hệ Thống Phân Lịch Thi Tín Chỉ Tại Trường Cao Đẳng Thương Mại Đà Nẵn...Xây Dựng Hệ Thống Phân Lịch Thi Tín Chỉ Tại Trường Cao Đẳng Thương Mại Đà Nẵn...
Xây Dựng Hệ Thống Phân Lịch Thi Tín Chỉ Tại Trường Cao Đẳng Thương Mại Đà Nẵn...
 
Vmo 2015-solution-1421633776
Vmo 2015-solution-1421633776Vmo 2015-solution-1421633776
Vmo 2015-solution-1421633776
 

More from DV Viết Luận văn luanvanmaster.com ZALO 0973287149

Ảnh Hưởng Của Marketing Quan Hệ Đến Lòng Trung Thành Của Khách Hàng.Tình Huốn...
Ảnh Hưởng Của Marketing Quan Hệ Đến Lòng Trung Thành Của Khách Hàng.Tình Huốn...Ảnh Hưởng Của Marketing Quan Hệ Đến Lòng Trung Thành Của Khách Hàng.Tình Huốn...
Ảnh Hưởng Của Marketing Quan Hệ Đến Lòng Trung Thành Của Khách Hàng.Tình Huốn...
DV Viết Luận văn luanvanmaster.com ZALO 0973287149
 
Phát triển nguồn nhân lực tại Uỷ ban nhân dân huyện Trà Bồng, tỉnh Quảng Ngãi...
Phát triển nguồn nhân lực tại Uỷ ban nhân dân huyện Trà Bồng, tỉnh Quảng Ngãi...Phát triển nguồn nhân lực tại Uỷ ban nhân dân huyện Trà Bồng, tỉnh Quảng Ngãi...
Phát triển nguồn nhân lực tại Uỷ ban nhân dân huyện Trà Bồng, tỉnh Quảng Ngãi...
DV Viết Luận văn luanvanmaster.com ZALO 0973287149
 
Báo cáo tốt Nghiệp tài chính hợp nhất tại tổng công ty Indochina gol...
Báo cáo tốt Nghiệp  tài chính hợp nhất tại tổng công ty Indochina gol...Báo cáo tốt Nghiệp  tài chính hợp nhất tại tổng công ty Indochina gol...
Báo cáo tốt Nghiệp tài chính hợp nhất tại tổng công ty Indochina gol...
DV Viết Luận văn luanvanmaster.com ZALO 0973287149
 
Tạo động lực thúc đẩy nhân viên làm việc tại ngân hàng TMCP Ngoại Thương Việt...
Tạo động lực thúc đẩy nhân viên làm việc tại ngân hàng TMCP Ngoại Thương Việt...Tạo động lực thúc đẩy nhân viên làm việc tại ngân hàng TMCP Ngoại Thương Việt...
Tạo động lực thúc đẩy nhân viên làm việc tại ngân hàng TMCP Ngoại Thương Việt...
DV Viết Luận văn luanvanmaster.com ZALO 0973287149
 
Phát triển công nghiệp trên địa bàn Thành phố Tam Kỳ, Tỉnh Quảng Na...
Phát triển công nghiệp trên địa bàn Thành phố Tam Kỳ, Tỉnh Quảng Na...Phát triển công nghiệp trên địa bàn Thành phố Tam Kỳ, Tỉnh Quảng Na...
Phát triển công nghiệp trên địa bàn Thành phố Tam Kỳ, Tỉnh Quảng Na...
DV Viết Luận văn luanvanmaster.com ZALO 0973287149
 
Giải pháp phát triển cho vay xuất nhập khẩu tại ngân hàng NN&PTNN ch...
Giải pháp phát triển cho vay xuất nhập khẩu tại ngân hàng NN&PTNN ch...Giải pháp phát triển cho vay xuất nhập khẩu tại ngân hàng NN&PTNN ch...
Giải pháp phát triển cho vay xuất nhập khẩu tại ngân hàng NN&PTNN ch...
DV Viết Luận văn luanvanmaster.com ZALO 0973287149
 
Hoàn thiện công tác lập báo cáo tài chính hợp nhất tại tổng công ...
Hoàn thiện công tác lập báo cáo tài chính hợp nhất tại tổng công ...Hoàn thiện công tác lập báo cáo tài chính hợp nhất tại tổng công ...
Hoàn thiện công tác lập báo cáo tài chính hợp nhất tại tổng công ...
DV Viết Luận văn luanvanmaster.com ZALO 0973287149
 
Luận Văn Thạc Sĩ Quản trị thành tích nhân viên tại Cục Hải quan TP Đà Nẵng.doc
Luận Văn Thạc Sĩ  Quản trị thành tích nhân viên tại Cục Hải quan TP Đà Nẵng.docLuận Văn Thạc Sĩ  Quản trị thành tích nhân viên tại Cục Hải quan TP Đà Nẵng.doc
Luận Văn Thạc Sĩ Quản trị thành tích nhân viên tại Cục Hải quan TP Đà Nẵng.doc
DV Viết Luận văn luanvanmaster.com ZALO 0973287149
 
Hoàn thiện công tác quản lý thuế thu nhập cá nhân tại cục thuế Tỉ...
Hoàn thiện công tác quản lý thuế thu nhập cá nhân tại cục thuế Tỉ...Hoàn thiện công tác quản lý thuế thu nhập cá nhân tại cục thuế Tỉ...
Hoàn thiện công tác quản lý thuế thu nhập cá nhân tại cục thuế Tỉ...
DV Viết Luận văn luanvanmaster.com ZALO 0973287149
 
Đề Tài Phát triển bền vững nông nghiệp Huyện Ba Tơ, Tỉnh Quảng Ngãi....
Đề Tài Phát triển bền vững nông nghiệp Huyện Ba Tơ, Tỉnh Quảng Ngãi....Đề Tài Phát triển bền vững nông nghiệp Huyện Ba Tơ, Tỉnh Quảng Ngãi....
Đề Tài Phát triển bền vững nông nghiệp Huyện Ba Tơ, Tỉnh Quảng Ngãi....
DV Viết Luận văn luanvanmaster.com ZALO 0973287149
 
Hoàn thiện công tác bảo trợ xã hội trên địa bàn huyện Phong Điền, tỉnh Thừa T...
Hoàn thiện công tác bảo trợ xã hội trên địa bàn huyện Phong Điền, tỉnh Thừa T...Hoàn thiện công tác bảo trợ xã hội trên địa bàn huyện Phong Điền, tỉnh Thừa T...
Hoàn thiện công tác bảo trợ xã hội trên địa bàn huyện Phong Điền, tỉnh Thừa T...
DV Viết Luận văn luanvanmaster.com ZALO 0973287149
 
Đề Tài Luận VănPhát triển sản phẩm du lịch tại thành phố Đà Nẵng.doc
Đề Tài Luận VănPhát triển sản phẩm du lịch tại thành phố Đà Nẵng.docĐề Tài Luận VănPhát triển sản phẩm du lịch tại thành phố Đà Nẵng.doc
Đề Tài Luận VănPhát triển sản phẩm du lịch tại thành phố Đà Nẵng.doc
DV Viết Luận văn luanvanmaster.com ZALO 0973287149
 
Đào tạo nghề cho lao động thuộc diện thu hồi đất trên địa bàn Thàn...
Đào tạo nghề cho lao động thuộc diện thu hồi đất trên địa bàn Thàn...Đào tạo nghề cho lao động thuộc diện thu hồi đất trên địa bàn Thàn...
Đào tạo nghề cho lao động thuộc diện thu hồi đất trên địa bàn Thàn...
DV Viết Luận văn luanvanmaster.com ZALO 0973287149
 
Tóm Tắt Luận Văn Thạc Sĩ Quản Trị Kinh Doanh Xây dựng chính sách Marketing tạ...
Tóm Tắt Luận Văn Thạc Sĩ Quản Trị Kinh Doanh Xây dựng chính sách Marketing tạ...Tóm Tắt Luận Văn Thạc Sĩ Quản Trị Kinh Doanh Xây dựng chính sách Marketing tạ...
Tóm Tắt Luận Văn Thạc Sĩ Quản Trị Kinh Doanh Xây dựng chính sách Marketing tạ...
DV Viết Luận văn luanvanmaster.com ZALO 0973287149
 
Đề Tài Nghiên cứu rủi ro cảm nhận đối với mua hàng thời trang trực tuyến.docx
Đề Tài Nghiên cứu rủi ro cảm nhận đối với mua hàng thời trang trực tuyến.docxĐề Tài Nghiên cứu rủi ro cảm nhận đối với mua hàng thời trang trực tuyến.docx
Đề Tài Nghiên cứu rủi ro cảm nhận đối với mua hàng thời trang trực tuyến.docx
DV Viết Luận văn luanvanmaster.com ZALO 0973287149
 
Giải pháp nâng cao động lực thúc đẩy người lao động tại công ty khai...
Giải pháp nâng cao động lực thúc đẩy người lao động tại công ty khai...Giải pháp nâng cao động lực thúc đẩy người lao động tại công ty khai...
Giải pháp nâng cao động lực thúc đẩy người lao động tại công ty khai...
DV Viết Luận văn luanvanmaster.com ZALO 0973287149
 
Giải pháp phát triển dịch vụ ngân hàng điện tử tại ngân hàng đầu ...
Giải pháp phát triển dịch vụ ngân hàng điện tử tại ngân hàng đầu ...Giải pháp phát triển dịch vụ ngân hàng điện tử tại ngân hàng đầu ...
Giải pháp phát triển dịch vụ ngân hàng điện tử tại ngân hàng đầu ...
DV Viết Luận văn luanvanmaster.com ZALO 0973287149
 
Giải pháp phát triển dịch vụ ngân hàng điện tử tại ngân hàng đầu ...
Giải pháp phát triển dịch vụ ngân hàng điện tử tại ngân hàng đầu ...Giải pháp phát triển dịch vụ ngân hàng điện tử tại ngân hàng đầu ...
Giải pháp phát triển dịch vụ ngân hàng điện tử tại ngân hàng đầu ...
DV Viết Luận văn luanvanmaster.com ZALO 0973287149
 
Quản trị quan hệ khách hàng tại Chi nhánh Viettel Đà Nẵng – Tập đoàn Viễn thô...
Quản trị quan hệ khách hàng tại Chi nhánh Viettel Đà Nẵng – Tập đoàn Viễn thô...Quản trị quan hệ khách hàng tại Chi nhánh Viettel Đà Nẵng – Tập đoàn Viễn thô...
Quản trị quan hệ khách hàng tại Chi nhánh Viettel Đà Nẵng – Tập đoàn Viễn thô...
DV Viết Luận văn luanvanmaster.com ZALO 0973287149
 
Đề Tài Đánh giá thành tích đội ngũ giảng viên trường Đại Học Phạm ...
Đề Tài Đánh giá thành tích đội ngũ giảng viên trường Đại Học Phạm ...Đề Tài Đánh giá thành tích đội ngũ giảng viên trường Đại Học Phạm ...
Đề Tài Đánh giá thành tích đội ngũ giảng viên trường Đại Học Phạm ...
DV Viết Luận văn luanvanmaster.com ZALO 0973287149
 

More from DV Viết Luận văn luanvanmaster.com ZALO 0973287149 (20)

Ảnh Hưởng Của Marketing Quan Hệ Đến Lòng Trung Thành Của Khách Hàng.Tình Huốn...
Ảnh Hưởng Của Marketing Quan Hệ Đến Lòng Trung Thành Của Khách Hàng.Tình Huốn...Ảnh Hưởng Của Marketing Quan Hệ Đến Lòng Trung Thành Của Khách Hàng.Tình Huốn...
Ảnh Hưởng Của Marketing Quan Hệ Đến Lòng Trung Thành Của Khách Hàng.Tình Huốn...
 
Phát triển nguồn nhân lực tại Uỷ ban nhân dân huyện Trà Bồng, tỉnh Quảng Ngãi...
Phát triển nguồn nhân lực tại Uỷ ban nhân dân huyện Trà Bồng, tỉnh Quảng Ngãi...Phát triển nguồn nhân lực tại Uỷ ban nhân dân huyện Trà Bồng, tỉnh Quảng Ngãi...
Phát triển nguồn nhân lực tại Uỷ ban nhân dân huyện Trà Bồng, tỉnh Quảng Ngãi...
 
Báo cáo tốt Nghiệp tài chính hợp nhất tại tổng công ty Indochina gol...
Báo cáo tốt Nghiệp  tài chính hợp nhất tại tổng công ty Indochina gol...Báo cáo tốt Nghiệp  tài chính hợp nhất tại tổng công ty Indochina gol...
Báo cáo tốt Nghiệp tài chính hợp nhất tại tổng công ty Indochina gol...
 
Tạo động lực thúc đẩy nhân viên làm việc tại ngân hàng TMCP Ngoại Thương Việt...
Tạo động lực thúc đẩy nhân viên làm việc tại ngân hàng TMCP Ngoại Thương Việt...Tạo động lực thúc đẩy nhân viên làm việc tại ngân hàng TMCP Ngoại Thương Việt...
Tạo động lực thúc đẩy nhân viên làm việc tại ngân hàng TMCP Ngoại Thương Việt...
 
Phát triển công nghiệp trên địa bàn Thành phố Tam Kỳ, Tỉnh Quảng Na...
Phát triển công nghiệp trên địa bàn Thành phố Tam Kỳ, Tỉnh Quảng Na...Phát triển công nghiệp trên địa bàn Thành phố Tam Kỳ, Tỉnh Quảng Na...
Phát triển công nghiệp trên địa bàn Thành phố Tam Kỳ, Tỉnh Quảng Na...
 
Giải pháp phát triển cho vay xuất nhập khẩu tại ngân hàng NN&PTNN ch...
Giải pháp phát triển cho vay xuất nhập khẩu tại ngân hàng NN&PTNN ch...Giải pháp phát triển cho vay xuất nhập khẩu tại ngân hàng NN&PTNN ch...
Giải pháp phát triển cho vay xuất nhập khẩu tại ngân hàng NN&PTNN ch...
 
Hoàn thiện công tác lập báo cáo tài chính hợp nhất tại tổng công ...
Hoàn thiện công tác lập báo cáo tài chính hợp nhất tại tổng công ...Hoàn thiện công tác lập báo cáo tài chính hợp nhất tại tổng công ...
Hoàn thiện công tác lập báo cáo tài chính hợp nhất tại tổng công ...
 
Luận Văn Thạc Sĩ Quản trị thành tích nhân viên tại Cục Hải quan TP Đà Nẵng.doc
Luận Văn Thạc Sĩ  Quản trị thành tích nhân viên tại Cục Hải quan TP Đà Nẵng.docLuận Văn Thạc Sĩ  Quản trị thành tích nhân viên tại Cục Hải quan TP Đà Nẵng.doc
Luận Văn Thạc Sĩ Quản trị thành tích nhân viên tại Cục Hải quan TP Đà Nẵng.doc
 
Hoàn thiện công tác quản lý thuế thu nhập cá nhân tại cục thuế Tỉ...
Hoàn thiện công tác quản lý thuế thu nhập cá nhân tại cục thuế Tỉ...Hoàn thiện công tác quản lý thuế thu nhập cá nhân tại cục thuế Tỉ...
Hoàn thiện công tác quản lý thuế thu nhập cá nhân tại cục thuế Tỉ...
 
Đề Tài Phát triển bền vững nông nghiệp Huyện Ba Tơ, Tỉnh Quảng Ngãi....
Đề Tài Phát triển bền vững nông nghiệp Huyện Ba Tơ, Tỉnh Quảng Ngãi....Đề Tài Phát triển bền vững nông nghiệp Huyện Ba Tơ, Tỉnh Quảng Ngãi....
Đề Tài Phát triển bền vững nông nghiệp Huyện Ba Tơ, Tỉnh Quảng Ngãi....
 
Hoàn thiện công tác bảo trợ xã hội trên địa bàn huyện Phong Điền, tỉnh Thừa T...
Hoàn thiện công tác bảo trợ xã hội trên địa bàn huyện Phong Điền, tỉnh Thừa T...Hoàn thiện công tác bảo trợ xã hội trên địa bàn huyện Phong Điền, tỉnh Thừa T...
Hoàn thiện công tác bảo trợ xã hội trên địa bàn huyện Phong Điền, tỉnh Thừa T...
 
Đề Tài Luận VănPhát triển sản phẩm du lịch tại thành phố Đà Nẵng.doc
Đề Tài Luận VănPhát triển sản phẩm du lịch tại thành phố Đà Nẵng.docĐề Tài Luận VănPhát triển sản phẩm du lịch tại thành phố Đà Nẵng.doc
Đề Tài Luận VănPhát triển sản phẩm du lịch tại thành phố Đà Nẵng.doc
 
Đào tạo nghề cho lao động thuộc diện thu hồi đất trên địa bàn Thàn...
Đào tạo nghề cho lao động thuộc diện thu hồi đất trên địa bàn Thàn...Đào tạo nghề cho lao động thuộc diện thu hồi đất trên địa bàn Thàn...
Đào tạo nghề cho lao động thuộc diện thu hồi đất trên địa bàn Thàn...
 
Tóm Tắt Luận Văn Thạc Sĩ Quản Trị Kinh Doanh Xây dựng chính sách Marketing tạ...
Tóm Tắt Luận Văn Thạc Sĩ Quản Trị Kinh Doanh Xây dựng chính sách Marketing tạ...Tóm Tắt Luận Văn Thạc Sĩ Quản Trị Kinh Doanh Xây dựng chính sách Marketing tạ...
Tóm Tắt Luận Văn Thạc Sĩ Quản Trị Kinh Doanh Xây dựng chính sách Marketing tạ...
 
Đề Tài Nghiên cứu rủi ro cảm nhận đối với mua hàng thời trang trực tuyến.docx
Đề Tài Nghiên cứu rủi ro cảm nhận đối với mua hàng thời trang trực tuyến.docxĐề Tài Nghiên cứu rủi ro cảm nhận đối với mua hàng thời trang trực tuyến.docx
Đề Tài Nghiên cứu rủi ro cảm nhận đối với mua hàng thời trang trực tuyến.docx
 
Giải pháp nâng cao động lực thúc đẩy người lao động tại công ty khai...
Giải pháp nâng cao động lực thúc đẩy người lao động tại công ty khai...Giải pháp nâng cao động lực thúc đẩy người lao động tại công ty khai...
Giải pháp nâng cao động lực thúc đẩy người lao động tại công ty khai...
 
Giải pháp phát triển dịch vụ ngân hàng điện tử tại ngân hàng đầu ...
Giải pháp phát triển dịch vụ ngân hàng điện tử tại ngân hàng đầu ...Giải pháp phát triển dịch vụ ngân hàng điện tử tại ngân hàng đầu ...
Giải pháp phát triển dịch vụ ngân hàng điện tử tại ngân hàng đầu ...
 
Giải pháp phát triển dịch vụ ngân hàng điện tử tại ngân hàng đầu ...
Giải pháp phát triển dịch vụ ngân hàng điện tử tại ngân hàng đầu ...Giải pháp phát triển dịch vụ ngân hàng điện tử tại ngân hàng đầu ...
Giải pháp phát triển dịch vụ ngân hàng điện tử tại ngân hàng đầu ...
 
Quản trị quan hệ khách hàng tại Chi nhánh Viettel Đà Nẵng – Tập đoàn Viễn thô...
Quản trị quan hệ khách hàng tại Chi nhánh Viettel Đà Nẵng – Tập đoàn Viễn thô...Quản trị quan hệ khách hàng tại Chi nhánh Viettel Đà Nẵng – Tập đoàn Viễn thô...
Quản trị quan hệ khách hàng tại Chi nhánh Viettel Đà Nẵng – Tập đoàn Viễn thô...
 
Đề Tài Đánh giá thành tích đội ngũ giảng viên trường Đại Học Phạm ...
Đề Tài Đánh giá thành tích đội ngũ giảng viên trường Đại Học Phạm ...Đề Tài Đánh giá thành tích đội ngũ giảng viên trường Đại Học Phạm ...
Đề Tài Đánh giá thành tích đội ngũ giảng viên trường Đại Học Phạm ...
 

Recently uploaded

98 BÀI LUYỆN NGHE TUYỂN SINH VÀO LỚP 10 TIẾNG ANH DẠNG TRẮC NGHIỆM 4 CÂU TRẢ ...
98 BÀI LUYỆN NGHE TUYỂN SINH VÀO LỚP 10 TIẾNG ANH DẠNG TRẮC NGHIỆM 4 CÂU TRẢ ...98 BÀI LUYỆN NGHE TUYỂN SINH VÀO LỚP 10 TIẾNG ANH DẠNG TRẮC NGHIỆM 4 CÂU TRẢ ...
98 BÀI LUYỆN NGHE TUYỂN SINH VÀO LỚP 10 TIẾNG ANH DẠNG TRẮC NGHIỆM 4 CÂU TRẢ ...
Nguyen Thanh Tu Collection
 
Ảnh hưởng của nhân sinh quan Phật giáo đến đời sống tinh thần Việt Nam hiện nay
Ảnh hưởng của nhân sinh quan Phật giáo đến đời sống tinh thần Việt Nam hiện nayẢnh hưởng của nhân sinh quan Phật giáo đến đời sống tinh thần Việt Nam hiện nay
Ảnh hưởng của nhân sinh quan Phật giáo đến đời sống tinh thần Việt Nam hiện nay
chinhkt50
 
GIÁO TRÌNH 2-TÀI LIỆU SỬA CHỮA BOARD MONO TỦ LẠNH MÁY GIẶT ĐIỀU HÒA.pdf
GIÁO TRÌNH 2-TÀI LIỆU SỬA CHỮA BOARD MONO TỦ LẠNH MÁY GIẶT ĐIỀU HÒA.pdfGIÁO TRÌNH 2-TÀI LIỆU SỬA CHỮA BOARD MONO TỦ LẠNH MÁY GIẶT ĐIỀU HÒA.pdf
GIÁO TRÌNH 2-TÀI LIỆU SỬA CHỮA BOARD MONO TỦ LẠNH MÁY GIẶT ĐIỀU HÒA.pdf
Điện Lạnh Bách Khoa Hà Nội
 
Khoá luận tốt nghiệp ngành Truyền thông đa phương tiện Xây dựng kế hoạch truy...
Khoá luận tốt nghiệp ngành Truyền thông đa phương tiện Xây dựng kế hoạch truy...Khoá luận tốt nghiệp ngành Truyền thông đa phương tiện Xây dựng kế hoạch truy...
Khoá luận tốt nghiệp ngành Truyền thông đa phương tiện Xây dựng kế hoạch truy...
https://www.facebook.com/garmentspace
 
AV6 - PIE CHART WRITING skill in english
AV6 - PIE CHART WRITING skill in englishAV6 - PIE CHART WRITING skill in english
AV6 - PIE CHART WRITING skill in english
Qucbo964093
 
BAI TAP ON HE LOP 2 LEN 3 MON TIENG VIET.pdf
BAI TAP ON HE LOP 2 LEN 3 MON TIENG VIET.pdfBAI TAP ON HE LOP 2 LEN 3 MON TIENG VIET.pdf
BAI TAP ON HE LOP 2 LEN 3 MON TIENG VIET.pdf
phamthuhoai20102005
 
CHUYÊN ĐỀ BỒI DƯỠNG HỌC SINH GIỎI KHOA HỌC TỰ NHIÊN 9 CHƯƠNG TRÌNH MỚI - PHẦN...
CHUYÊN ĐỀ BỒI DƯỠNG HỌC SINH GIỎI KHOA HỌC TỰ NHIÊN 9 CHƯƠNG TRÌNH MỚI - PHẦN...CHUYÊN ĐỀ BỒI DƯỠNG HỌC SINH GIỎI KHOA HỌC TỰ NHIÊN 9 CHƯƠNG TRÌNH MỚI - PHẦN...
CHUYÊN ĐỀ BỒI DƯỠNG HỌC SINH GIỎI KHOA HỌC TỰ NHIÊN 9 CHƯƠNG TRÌNH MỚI - PHẦN...
Nguyen Thanh Tu Collection
 
Dẫn luận ngôn ngữ - Tu vung ngu nghia.pptx
Dẫn luận ngôn ngữ - Tu vung ngu nghia.pptxDẫn luận ngôn ngữ - Tu vung ngu nghia.pptx
Dẫn luận ngôn ngữ - Tu vung ngu nghia.pptx
nvlinhchi1612
 
Chương III (Nội dung vẽ sơ đồ tư duy chương 3)
Chương III (Nội dung vẽ sơ đồ tư duy chương 3)Chương III (Nội dung vẽ sơ đồ tư duy chương 3)
Chương III (Nội dung vẽ sơ đồ tư duy chương 3)
duykhoacao
 
30 - ĐỀ THI HSG - HÓA HỌC 9 - NĂM HỌC 2021 - 2022.pdf
30 - ĐỀ THI HSG - HÓA HỌC 9 - NĂM HỌC 2021 - 2022.pdf30 - ĐỀ THI HSG - HÓA HỌC 9 - NĂM HỌC 2021 - 2022.pdf
30 - ĐỀ THI HSG - HÓA HỌC 9 - NĂM HỌC 2021 - 2022.pdf
ngocnguyensp1
 

Recently uploaded (10)

98 BÀI LUYỆN NGHE TUYỂN SINH VÀO LỚP 10 TIẾNG ANH DẠNG TRẮC NGHIỆM 4 CÂU TRẢ ...
98 BÀI LUYỆN NGHE TUYỂN SINH VÀO LỚP 10 TIẾNG ANH DẠNG TRẮC NGHIỆM 4 CÂU TRẢ ...98 BÀI LUYỆN NGHE TUYỂN SINH VÀO LỚP 10 TIẾNG ANH DẠNG TRẮC NGHIỆM 4 CÂU TRẢ ...
98 BÀI LUYỆN NGHE TUYỂN SINH VÀO LỚP 10 TIẾNG ANH DẠNG TRẮC NGHIỆM 4 CÂU TRẢ ...
 
Ảnh hưởng của nhân sinh quan Phật giáo đến đời sống tinh thần Việt Nam hiện nay
Ảnh hưởng của nhân sinh quan Phật giáo đến đời sống tinh thần Việt Nam hiện nayẢnh hưởng của nhân sinh quan Phật giáo đến đời sống tinh thần Việt Nam hiện nay
Ảnh hưởng của nhân sinh quan Phật giáo đến đời sống tinh thần Việt Nam hiện nay
 
GIÁO TRÌNH 2-TÀI LIỆU SỬA CHỮA BOARD MONO TỦ LẠNH MÁY GIẶT ĐIỀU HÒA.pdf
GIÁO TRÌNH 2-TÀI LIỆU SỬA CHỮA BOARD MONO TỦ LẠNH MÁY GIẶT ĐIỀU HÒA.pdfGIÁO TRÌNH 2-TÀI LIỆU SỬA CHỮA BOARD MONO TỦ LẠNH MÁY GIẶT ĐIỀU HÒA.pdf
GIÁO TRÌNH 2-TÀI LIỆU SỬA CHỮA BOARD MONO TỦ LẠNH MÁY GIẶT ĐIỀU HÒA.pdf
 
Khoá luận tốt nghiệp ngành Truyền thông đa phương tiện Xây dựng kế hoạch truy...
Khoá luận tốt nghiệp ngành Truyền thông đa phương tiện Xây dựng kế hoạch truy...Khoá luận tốt nghiệp ngành Truyền thông đa phương tiện Xây dựng kế hoạch truy...
Khoá luận tốt nghiệp ngành Truyền thông đa phương tiện Xây dựng kế hoạch truy...
 
AV6 - PIE CHART WRITING skill in english
AV6 - PIE CHART WRITING skill in englishAV6 - PIE CHART WRITING skill in english
AV6 - PIE CHART WRITING skill in english
 
BAI TAP ON HE LOP 2 LEN 3 MON TIENG VIET.pdf
BAI TAP ON HE LOP 2 LEN 3 MON TIENG VIET.pdfBAI TAP ON HE LOP 2 LEN 3 MON TIENG VIET.pdf
BAI TAP ON HE LOP 2 LEN 3 MON TIENG VIET.pdf
 
CHUYÊN ĐỀ BỒI DƯỠNG HỌC SINH GIỎI KHOA HỌC TỰ NHIÊN 9 CHƯƠNG TRÌNH MỚI - PHẦN...
CHUYÊN ĐỀ BỒI DƯỠNG HỌC SINH GIỎI KHOA HỌC TỰ NHIÊN 9 CHƯƠNG TRÌNH MỚI - PHẦN...CHUYÊN ĐỀ BỒI DƯỠNG HỌC SINH GIỎI KHOA HỌC TỰ NHIÊN 9 CHƯƠNG TRÌNH MỚI - PHẦN...
CHUYÊN ĐỀ BỒI DƯỠNG HỌC SINH GIỎI KHOA HỌC TỰ NHIÊN 9 CHƯƠNG TRÌNH MỚI - PHẦN...
 
Dẫn luận ngôn ngữ - Tu vung ngu nghia.pptx
Dẫn luận ngôn ngữ - Tu vung ngu nghia.pptxDẫn luận ngôn ngữ - Tu vung ngu nghia.pptx
Dẫn luận ngôn ngữ - Tu vung ngu nghia.pptx
 
Chương III (Nội dung vẽ sơ đồ tư duy chương 3)
Chương III (Nội dung vẽ sơ đồ tư duy chương 3)Chương III (Nội dung vẽ sơ đồ tư duy chương 3)
Chương III (Nội dung vẽ sơ đồ tư duy chương 3)
 
30 - ĐỀ THI HSG - HÓA HỌC 9 - NĂM HỌC 2021 - 2022.pdf
30 - ĐỀ THI HSG - HÓA HỌC 9 - NĂM HỌC 2021 - 2022.pdf30 - ĐỀ THI HSG - HÓA HỌC 9 - NĂM HỌC 2021 - 2022.pdf
30 - ĐỀ THI HSG - HÓA HỌC 9 - NĂM HỌC 2021 - 2022.pdf
 

M T So L P Phương Trình Diophantine.docx

  • 1. ĐẠI HOC THÁI NGUYÊN TRƯ NG ĐẠI HOC KHOA HOC Tải tài liệu tại sividoc.com Viết đề tài giá sinh viên – ZALO:0973.287.149-TEAMLUANVAN.COM BÙI HữU MÊN M T SO L P PHƯƠNG TRÌNH DIOPHANTINE LU N VĂN THẠC SĨ TOÁN HOC Thái Nguyên - 2017
  • 2. ĐẠI HOC THÁI NGUYÊN TRƯ NG ĐẠI HOC KHOA HOC Tải tài liệu tại sividoc.com Viết đề tài giá sinh viên – ZALO:0973.287.149-TEAMLUANVAN.COM BÙI HữU MÊN M T SO L P PHƯƠNG TRÌNH DIOPHANTINE LU N VĂN THẠC SĨ TOÁN HOC Chuyên ngành: Phương pháp Toán sơ cap Mã so: 60 46 01 13 NGƯŐI HƯŐNG DAN KHOA HOC GS.TSKH. Đ NG HÙNG THANG Thái Nguyên - 2017
  • 3. 3 Viết đề tài giá sinh viên – ZALO:0973.287.149-TEAMLUANVAN.COM Mục lục Danh sách kí hi u 4 MƠ đau 5 Chương 1. Phương trình Diophantine tuyen tính 7 1.1 Phương trình b c nhat hai an . . . . . . . . . . . . . . . . . . . . . . . 8 1.2 Phương trình b c nhat nhieu an....................................................................... 15 Chương 2. M t so phương trình Diophantine phi tuyen 23 2.1 Phương trình Pell loại 1 .................................................................................. 23 2.2 Phương trình Pell loại 2 .................................................................................. 30 2.3 Phương trình Pythagoras................................................................................. 38 Chương 3. Liên phân so và ng dụng trong phương trình Diophantine 45 3.1 Liên phân so hữu hạn...................................................................................... 45 3.2 Liên phân so vô hạn........................................................................................ 49 3.3 Liên phân so vô hạn tuan hoàn ....................................................................... 50 3.4 Áp dụng vào phương trình Diophante ............................................................ 56 3.4.1 Phương trình b c nhat hai an Ax+By = C......................................... 56 3.4.2 Phương trình x2 − dy2 = ±1 ....................................................................57 Ket lu n 62 Tài li u tham khao 63
  • 4. 4 Viết đề tài giá sinh viên – ZALO:0973.287.149-TEAMLUANVAN.COM Danh sách kí hi u N t p hop các so tụ nhiên Z vành các so nguyên Q trưòng các so hữu tý R trưòng các so thục C trưòng các so phức Fp trưòng có p phan tử K[X] vành đa thức vói h so trên trưòng K [x| tran của so x degP(X) b c của đa thức P(X) mod p modulo p gcd(P(X),Q(X)) ưóc chung lón nhat của hai đa thức P(X) và Q(X)
  • 5. 5 Viết đề tài giá sinh viên – ZALO:0973.287.149-TEAMLUANVAN.COM MƠ đau Phương trình Diophantine là m t chủ đe lón của Lý thuyet so, chứa đụng nhieu lý thuyet toán hoc sâu sac, gan lien vói nhieu tên tuoi của nhieu nhà toán hoc xuat sac. Mục tiêu của đe tài lu n văn là: Tìm hieu m t so lóp phương trình Diophantine như: phương trình Diophantine tuyen tính; m t so phương trình Diophantine phi tuyen (phương trình Pell, phương trình Pell mỏ r ng, phương trình Pythagoras Fermat). Liên phân so và ứng dụng trong phương trình Diophantine. Ve m t ứng dụng, lu n văn sẽ áp dụng lý thuyet đe soi sáng những bài toán so hoc ỏ pho thông, h thong hóa, tong quát hóa và sáng tác ra những bài toán so hoc mói. Lu n văn sẽ co gang trỏ thành m t tài li u tham khảo tot, thiet thục phục vụ cho vi c giảng dạy, nhat là vi c giảng dạy và boi dưõng hoc sinh giỏi. Ngoài ra thông qua vi c viet lu n văn, tác giả lu n văn có cơ h i mỏ r ng nâng cao hieu biet ve toán sơ cap nói chung và so hoc nói riêng, hình thành các kỹ năng chứng minh các định lí so hoc và giải các bài toán so hoc, phục vụ tot cho vi c giảng dạy môn Toán ỏ trưòng pho thông. N i dung của lu n văn đưoc trình bày trong ba chương như sau: • Chương 1. Phương trình Diophantine tuyen tính. Trong chương này chúng tôi trình bày ve phương trình b c nhat hai an, nhieu an, và m t so bài toán chon loc. • Chương 2. M t so phương trình Diophantine phi tuyen. Trong chương này chúng tôi trình bày n i dung chính ve các phương trình Pell loại 1, phương trình Pell loại , và phương trình Pythagoras. • Chương 3. Liên phân so và ŕng dnng trong phương trình Diophantine. Trong
  • 6. 6 Viết đề tài giá sinh viên – ZALO:0973.287.149-TEAMLUANVAN.COM chương này chúng tôi trình bày m t cách ngan gon các sụ ki n ve liên phân so, đ c bi t là các ứng dụng của chúng đe giải phương trình Pell. Lu n văn này đưoc thục hi n tại Trưòng Đại hoc Khoa hoc - Đại hoc Thái Nguyên và hoàn thành vói sụ hưóng dȁn của GS.TSKH. Đ ng Hùng Thang (Trưòng ĐHKHTN - ĐHQG Hà N i). Tác giả xin đưoc bày tỏ lòng biet ơn chân thành và sâu sac tói ngưòi hưóng dȁn khoa hoc của mình, ngưòi đã đ t van đe nghiên cứu, dành nhieu thòi gian hưóng dȁn và t n tình giải đáp những thac mac của tác giả trong suot quá trình làm lu n văn. Tác giả xin trân trong cảm ơn Ban Giám hi u Trưòng Đại hoc Khoa hoc - Đại hoc Thái Nguyên, Ban Chủ nhi m Khoa Toán–Tin, cùng các giảng viên đã tham gia giảng dạy, đã tạo moi đieu ki n tot nhat đe tác giả hoc t p và nghiên cứu. Tác giả muon gửi những lòi cảm ơn tot đep nhat tói t p the lóp Cao hoc Toán khóa 9 (2015-2017) đã đ ng viên và giúp đõ tác giả rat nhieu trong suot quá trình hoc t p. Nhân dịp này, tác giả cũng xin chân thành cảm ơn Sỏ Giáo dục và Đào tạo Hải Phòng, Ban Giám hi u và các đong nghi p ỏ Trưòng THPT Thái Phiên đã tạo đieu ki n cho tác giả hoàn thành tot nhi m vụ hoc t p và công tác của mình. Cuoi cùng, tác giả muon dành những lòi cảm ơn đ c bi t nhat đen đại gia đình vì những đ ng viên và chia sẻ những khó khăn đe tác giả hoàn thành lu n văn này. Thái Nguyên, ngày 10 tháng 11 năm 2017 Tác giả Bùi H u Mên
  • 7. 7 Viết đề tài giá sinh viên – ZALO:0973.287.149-TEAMLUANVAN.COM Chương 1 Phương trình Diophantine tuyen tính Phương trình Diophantine là m t trong những chủ đe sâu sac và rat r ng của Lý thuyet so. Mục đích của chương này là nghiên cứu ve phương trình Diophantine b c nhat hai và nhieu an. Như m t minh hoa cho lý thuyet, các ví dụ là các bài toán trích từ các đe thi sẽ đưoc trình bày. Đ c tính của các phương trình Diophantine là chúng có m t hay nhieu an so mà moi h so đeu là so nguyên và chỉ yêu cau tìm các nghi m nguyên (ho c nguyên dương). Nhà toán hoc noi tieng thòi co đại Diophantine đã có công lón vì những nghiên cứu tiên phong ve chúng. Vói m t phương trình Diophantine cho trưóc ta có the đ t ra các câu hỏi sau đây (xep theo thứ tụ từ de đen khó): Câu hói 1. Nó có nghi m nguyên hay không ? Câu hói 2. Nó có m t so hữu hạn nghi m hay có vô so nghi m? Câu hói 3. Hãy tìm tat cả các nghi m của nó. Chȁng hạn, ta hãy xét phương trình Diophantine xn +yn = zn trong đó n là so nguyên dương lón hơn hay bang 2. Vói n = 2 phương trình trên có vô so nghi m và ta có the tìm đưoc tưòng minh tat cả các nghi m của nó. Vói n > 2, nhà toán hoc thiên tài của the ký 17 Pierre de Fermat khȁng định rang
  • 8. 8 Viết đề tài giá sinh viên – ZALO:0973.287.149-TEAMLUANVAN.COM phương trình trên không có nghi m nguyên dương. Ket lu n này ngày nay đưoc mang tên là Định lí lớn Fermat hay Định lí cuoi cùng của Fermat. Ngưòi ta đã không tìm thay dau vet của chứng minh khȁng định trên của Fermat mà chỉ thay m t ghi chú của Fermat bên le cuon sách “So hoc” của Diophantine: “Tôi đã tìm đưoc m t chứng minh th t là tuy t vòi nhưng vì le sách ỏ đây quá hep nên không the viet ra”. Năm 1983, nhà toán hoc 29 tuoi ngưòi Đức là Faltings đã chứng minh thành công m t giả thuyet của Mordell trong lĩnh vục Hình hoc đại so roi từ đó suy ra rang phương trình xn + yn = zn vói n > 2 chỉ có m t so hữu hạn nghi m nguyên. Vói thành tụu này Faltings đã nh n đưoc Giải thưỏng Fields (giải thưỏng quoc te cao nhat dành cho các nhà toán hoc không quá 40 tuoi). Năm 1993 nhà toán hoc ngưòi Anh là Andrew Wiles đã công bo phép chứng minh của Định lí lón Fermat. Đây là m t câu chuy n lón của Toán hoc, có the tham khảo trong Amir D. Aczel [1]. Vói sụ ra đòi của máy tính, ngưòi ta cũng đ t câu hỏi: Có ton tại chăng m t thu t toán đe vói moi phương trình Diophantine cho trưóc nhò đó có the khȁng định đưoc rang phương trình này có nghi m nguyên hay không. Tiec thay câu trả lòi lại là: không có m t thu t toán như v y (Định lí Machiakevich). 1.1 Phương trình b c nhat hai an Phương trình Diophantine đơn giản nhat là phương trình b c nhat hai an ax + by = c (1.1) trong đó a, b, c là những so nguyên cho trưóc khác 0. Van đe đ t ra là vói đieu ki n nào của a, b, c thì phương trình (1.1) có nghi m và neu có thì cách tìm nghi m the nào. Định lí 1.1.1. Đieu ki n can và đủ đe phương trình (1.1) có nghi m nguyên là
  • 9. 9 Viết đề tài giá sinh viên – ZALO:0973.287.149-TEAMLUANVAN.COM a x = x0 + b t, (a,b) là ước của c. Chŕng minh. Đieu ki n can. Giả sử (x0,y0) là m t nghi m nguyên của (1.1). Khi đó ax0 +by0 = c. Neu d = (a,b) thì rõ ràng d | c. Đieu ki n đủ. Giả sử d = (a,b) và d | c. Ta có a = da1, b = db1, c = dc1 trong đó (a1, b1) = 1. Phương trình (1.1) tương đương vói a1x + b1y = c1. Xét a1 so {b1k} vói k = 0,1,2,...,a1 − 1. Vì (a1,b1) = 1 nên các so này khi chia cho a1 sẽ cho ta các so dư khác nhau. V y tại k0, 0 ≤ k0 ≤ a1 − 1 sao cho b1k0 = c1 (mod a1). Đieu này có nghĩa là: c1 −b1k0 = a1l0 vói l0 ∈ Z hay c1 = a1l0 +b1k0. V y (l0,k0) là m t nghi m của phương trình (1.1). Phép chứng minh định lí đưoc hoàn thành. Tiep theo ta hãy đi tìm tat cả các nghi m của phương trình (1.1) Định lí 1.1.2. Neu (x0,y0) là m t nghi m nguyên của (1.1) thì nó có vô so nghi m nguyên và nghi m nguyên (x,y) của nó được cho bới công thŕc trong đó t ∈ Z và d = (a,b). d y = y0 − d t, (1.2) Chŕng minh. Trưóc het ta kiem tra moi c p so (x,y) cho bỏi công thức (1.2) là nghi m. Th t v y ax + by = ax0 + by0 = c. Đảo lại, giả sử (x1,y1) là m t nghi m của (1.1) tức là ax1 +by1 = c. Trừ đȁng thức này vào đȁng thức ax0 +by0 = c ta thu đưoc a(x1 − x0) = b(y0 − y1). (1.3)
  • 10. 10 Viết đề tài giá sinh viên – ZALO:0973.287.149-TEAMLUANVAN.COM d d ki Vì d = (a,b) nên a = a1d, b = b1d vói (a1,b1) = 1. Thay vào (1.3) ta đưoc a1(x1 − x0) = b1(y0 − y1). Vì (a1,b1) = 1 nên y0 −y1 = ta1 và x1 −x0 = tb1. V y y1 = y0 −ta1 = y0 − at và x1 = x0 +tb1 = x0 + bt . Phép chứng minh đưoc ket thúc. Thu t toán tìm nghi m cua phương trình Diophantine b c nhat. Từ Định lí 1.1.2 ta thay rang đe tìm tat cả các nghi m của (1.1) ta chỉ can tìm m t nghi m (x0, y0) nào đó của nó. Ta goi m t nghi m cụ the như the là m t nghi m riêng còn công thức (1.2) đưoc goi là nghi m tőng quát. Sau đây ta sẽ trình bày m t thu t toán cho phép xác định khá nhanh m t nghi m riêng của (1.1). Giả sử q0,q1,... là m t dãy các so nguyên dương. Vói mői i ≥ 0 ta kí hi u [q0,q1,...,qi] là phân so sau đây [q0,q1,...,qi] = q0 + 1 . 1 q1 + 1 q2 +··· + 1 qi−1 + qi Bang phương pháp quy nạp có the de dàng chứng minh đưoc bo đe sau: Bo đe 1.1.3. Giá sr {hn}, {kn} là hai dãy so nguyên được xác định như sau: h−2 = 0, h−1 = 1, h1 = qihi−1 +hi−2, i ≥ 0, k0 = 1, k1 = q1, ki = qihki−1 +ki−2, i ≥ 2. Khi đó với moi i ≥ 1 ta có: (a) hiki−1 − hi−1ki = (−1)i−1 ; (b) [q0,q1,...,qi] = hi .
  • 11. 11 Viết đề tài giá sinh viên – ZALO:0973.287.149-TEAMLUANVAN.COM Bây giò cho hai so dương a, b vói a > b. Ta hãy viet thu t toán Euclid tìm ưóc chung lón nhat của a và b. Từ h (1.4) ta suy ra a = bq0 +r1 b = r1q1 +r2 ... rn−1 = rnqn−1 + rn+1 rn = rn+1qn. a b = [q0,q1,...,qn] (1.4) Từ khȁng định (b) của Bo đe 1.1.3 ta có a hn b = kn . Từ (a) ta suy ra (hn,kn) = 1. Do đó neu (a,b) = 1 thì a = hn và b = kn. V y thì akn−1 − bhn−1 = (−1)n−1 . Thành thử ton tại x0 ∈ {kn−1} và y0 ∈ {hn−1} sao cho ax0 + by0 = 1. Ta thử từng trưòng hop (nhieu nhat là bon phép thử) đe xác định x0, y0. Như v y đe giải phương trình (1.1) ta sẽ tien hành lan lưot các bưóc sau đây. Bưóc 1. Tìm d = (a,b) sau đó chia hai ve cho d đe đưoc m t phương trình tương đương a1x+b1y = c1 , ỏ đó a = da1, b = db1, c = dc1, (a1,b1) = 1. Bưóc 2. Viet thu t toán Euclid cho hai so |a1| và |b1|. Giả sử |a1| > |b1|. |a1| = |b1|q0 + r1
  • 12. 12 Viết đề tài giá sinh viên – ZALO:0973.287.149-TEAMLUANVAN.COM k |b1| = r1q1 + r2 ... rn−1 = rnqn−1 + rn+1 rn = rn+1qn. Bưóc 3. Tính [q0,q1,...,qn−1] = h . Bưóc 4. Lay nghi m riêng (x0 ′ ,y0 ′ ) của phương trình a1x + b1y = 1 thoả mãn đieu ki n |x0 ′ | = k, |y′ 0| = h. Ta xác định dau của x0 ′ và y′ 0 bang cách thử. Bưóc 5. Ta có x0 = c1x0 ′ , y0 = c1y′ 0 là nghi m riêng của phương trình (1.1). Khi đó nghi m tong quát là x = x0 + b1t, y = y0 + a1t, vói t ∈ Z. Ví dụ 1.1.4. Giải phương trình Diophantine 342x − 123y = 15. Lời giái. Ta sẽ làm lan lưot theo các bưóc như trên. Bưóc 1. Úóc chung lón nhat của 342 và 123 là 3. phương trình đã cho tương đương vói 114x −41y = 5. Bưóc 2. Ta viet thu t toán Euclid cho 114 và 41. 114 = 41 · 2 + 32 41 = 32 · 1 + 9 32 = 9 · 3 + 5 9 = 5 · 1 + 4 5 = 4 · 1 + 1 4 = 1 · 4.
  • 13. 13 Viết đề tài giá sinh viên – ZALO:0973.287.149-TEAMLUANVAN.COM Bưóc 3. Ta bieu dien theo liên phân so h 1 k = 2+ 1 1 + 3 + 25 = 9 . 1 1 1 + 1 Như v y h = 25, k = 9. Bưóc 4. Lay nghi m riêng x0 ′ , y′ 0 của phương trình 114x − 41y = 1 thoả mãn đieu ki n |x0 ′ | = 9 và |y′ 0| = 25. Bang cách thử ta tìm đưoc x0 ′ = 9, y′0 = 25. Bưóc 5. Nghi m riêng x0, y0 của phương trình đã cho là x0 = 9 · 5 = 45, y0 = 25 · 5 = 125 và nghi m tong quát là x = 45 + 41t, y = 125 + 114t, vói t ∈ Z. Nh n xét 1.1.5. 1. Neu a | c ta có the lay nghi m riêng x c 0 = a , y0 = 0. 2. Xét trưòng hop (a,b) = 1. Theo Định lí Euler ta có aϕ(b) − 1 = kb vói k ∈ Z. V y caϕ(b) −bkc = c. Do đó x0 = caϕ(b) − 1, y0 = −kc là m t nghi m riêng của phương trình đang xét.
  • 14. 14 Viết đề tài giá sinh viên – ZALO:0973.287.149-TEAMLUANVAN.COM b b = ams = ams = as as M t so bài toán chon loc Bài toán 1.1.6. Cho hai so nguyên dương a, b. Chŕng minh rang (a,b) = 1 khi và chỉ khi ton tại các so nguyên dương u, v sao cho au−bv = 1. Lời giái. Đieu ki n đủ là hien nhiên. Đảo lại, giả sử (a,b) = 1. Theo Định lí 1.1.1, ton tại các so nguyên x0, y0 đe cho ax0 +by0 = 1. Đ t u = x0 + bt, v = at − y0, trong đó t ∈ Z đưoc chon sao cho t > − x0 , t > y0 . Khi đó u và v là các so nguyên dương và au − bv = ax0 + by0 = 1. Bài toán 1.1.7. Giá sr (l,m) = 1 và al = bm , trong đó a,b,l,m ∈ N∗. Khi đó ton tại n đe a = nm , b = nl . Lời giái. Theo Bài toán 1.1.6 ton tại các so r, s ∈ N∗ đe lr − ms = 1. Ta có lr−ms alr bmr br m Suy ra √ m a = br . Vì √ m a là m t so hữu tỉ nên nó phải là so nguyên. V y n = br ∈ N∗. Suy ra a = nm . Từ đó bm = nl = nml , nên b = nl . Bài toán đưoc chứng minh xong. Bài toán 1.1.8. Cho a ∈ N∗. Hãy tìm g = (am − 1,an − 1). Lời giái. Trưóc het xét trưòng hop (m,n) = 1. Vì (a − 1) | am − 1, (a −1)an − 1 nên a − 1 là ưóc của g. Đảo lại, ta sẽ chứng minh g cũng là ưóc của a − 1. Theo Bài toán 1.1.6 ton tại các so u, v ∈ N∗ sao cho mu−nv = 1. as a = a .
  • 15. 15 Viết đề tài giá sinh viên – ZALO:0973.287.149-TEAMLUANVAN.COM Vì g | am − 1, g | an − 1 nên cũng có g | amu − 1, g | anv − 1. Suy ra g | amu − anv = anv (amu−nv − 1) = anv (a − 1). M t khác, de thay (g,a) = 1. V y g | (a−1). Như v y neu (m,n) = 1 thì (am −1,an −1) = a−1. Vói m, n bat kì, giả sử d = (m,n). Khi ay m = dm1, n = dn1 và (m1,n1) = 1. Ta có (am − 1,an − 1) = ((ad )m1 −1, (ad )n1 − 1) = ad − 1. Tóm lại ta có công thức (am −1,an −1) = a(m,n)−1. Bài toán 1.1.9. Cho (a, b) = 1 trong đó a, b ∈ N∗. Tìm giá trị c ∈ N∗ lớn nhat đe phương trình ax+by = c không có nghi m nguyên dương. Lời giái. Ta chứng minh rang c = ab là giá trị can tìm. Giả sử c > ab. Xét b so a, 2a,..., ba. Vì (a, b) = 1 nên các so này khi chia cho b sẽ cho các so dư khác nhau. V y ton tại so k, 1 ≤ k ≤ b, sao cho ka ≡ c (mod b). Suy ra c − ka = lb trong đó l ∈ Z. Neu c > ab thì c > ka, do đó lb > 0 hay l ∈ N∗. Như v y (k, l) là nghi m nguyên dương của phương trình ax +by = c. M t khác, giả sử (x0,y0) là nghi m nguyên dương của phương trình ax +by = ab. Khi đó ax0 = ab− by0 = b(a− y0). Vì (a,b) = 1 nên từ đó suy ra x0 . b. Tương tụ, y0 . a. Như v y x0 ≥ b, y0 ≥ a, do đó ab = ax0 + by0 ≥ 2ab. Vô lý. Phép chứng minh đưoc ket thúc. 1.2 Phương trình b c nhat nhieu an Trong mục này ta mỏ r ng ket quả của mục trưóc bang cách xét phương trình Diophantine b c nhat n an a1x1 + a2x2 + ... + anxn = c. (1.5)
  • 16. 16 Viết đề tài giá sinh viên – ZALO:0973.287.149-TEAMLUANVAN.COM b trong đó a1,a2,...,an và c là các so nguyên cho trưóc, n ≥ 2. Đoi vói bat kỳ m t phương trình nào, câu hỏi đau tiên là, trong những tình huong nào của h so, ta có the khȁng định ve tính ton tại nghi m của nó. Ve sụ ton tại nghi m của phương trình Diophantine b c nhat n an này ta có định lí sau đây. Định lí 1.2.1. Đieu ki n can và đủ đe phương trình (1.5) có nghi m là (a1,a2,...,an) | c. Chŕng minh. Đieu ki n can là hien nhiên. Ta sẽ chứng minh đieu ki n đủ bang phương pháp quy nạp, vói n = 1 đieu khȁng định là đúng do Định lí 1.1.1. Giả sử định lí đúng vói n = 1, ta chứng minh nó đúng vói n. Kí hi u bi = ai , c1 = c , ỏ đó d = (a1,a2,...,an). d d Phương trình (1.5) tương đương vói b1x1 + b2x2 + ... + bnxn = c1. (1.6) trong đó (b1,b2,...,bn) = 1. Đ t b = (b1,b2,...,bn−1). Ta có (b,bn) = 1. Theo Định lí 1.1.1 ton tại so nguyên ln và k sao cho: bnln +bk = c1. (1.7) Kí hi u b′ i = bi vói i = 1,2,...,n− 1. Ta có (b1 ′ ,b′ 2,...s,b′ n−1) = 1. Theo giả thiet quy nạp ton tại các so nguyên l1,l2,...,ln−1 sao cho b′ 1l1 +b′ 2l2 +...+b′ n−1ln−1 = k hay b1l1 +b2l2 +...+bn−1ln−1 = bk. (1.8) Từ (1.7) và (1.8) ta suy ra b1l1 + b2l2 + ... + bn−1ln−1 + bnln = c1 tức là (l1,l2,...,ln) là nghi m của phương trình (1.6). Định lí đưoc chứng minh.
  • 17. 17 Viết đề tài giá sinh viên – ZALO:0973.287.149-TEAMLUANVAN.COM x2 +α +ant2 2 Chúng ta không đi sâu vào vi c tìm bieu thức cho nghi m tong quát của nó như đã làm đoi vói trưòng hop n = 2. Tuy nhiên có the thay rang neu phương trình (1.5) có nghi m nguyên α1,α2,...,αn thì nó sẽ có vô so nghi m nguyên phụ thu c vào n − 1 tham so. Th t v y, de dàng kiem tra đưoc tat cả các b n so nguyên x1,x2,...,xn xác định như sau là nghi m của (1.5), x1 = α1 + ant1 ... xn−1 = αn−1 +antn−1 xn = αn −a1t1 −a2t2 ...−an−1tn−1 trong đó ti ∈ Z, i = 1,2,...n −1 đưoc chon tuỳ ý. Bây giò, ta sẽ thảo lu n ve cách giải phương trình (1.5). Ve m t thục hành ta có the tien hành theo hai cách sau đây. Cách 1. Đưa (1.5) ve trưòng hop có m t h so bang 1. Cách 2. Neu phương trình (1.5) có hai h so nguyên to cùng nhau, chȁng hạn (a1,a2) = 1 thì ta viet dưói dạng a1x1 + a2x2 = c − a3x3 − ... − anxn roi giải phương trình theo hai an x1, x2. Ta xét hai ví dụ sau đây, nó sẽ lan lưot minh hoa cho hai cách trên. Ví dụ 1.2.2. Tìm tat cả các nghi m nguyên của phương trình 6x + 45y + 6z − 10t = 13. Lời giái. Phương trình đã cho đưoc viet dưói dạng 6(x+z)+10(4y−t)+ 5y = 13.
  • 18. 18 Viết đề tài giá sinh viên – ZALO:0973.287.149-TEAMLUANVAN.COM Đ t x + z = x1, 4y −t = x2 ta đưoc 6x1 + 10x2 + 5y = 13. Suy ra x1 +10x2 +5(y+x1) = 13. Đ t x1 +y = x3 ta đưoc x1 +10x2 +5x3 = 13. V y x1 = 13 − 10x2 − 5x3. Từ đây, bang tính toán trục tiep ta có y = x3 −x1 = x3 −(13−10x2 −5x3) = 10x2 +6x3 −13; t = 4y−x2 = 39x2 +24x3 −52; x = x1 − z = 13 − 10x2 − 5x3 − z. V y nghi m tong quát của phương trình đã cho là x = 13 − 10x2 − 5x3 − x4, y = 10x2 + 6x3 − 13, z = x4, t = 39x2 + 24x3 − 52. trong đó x2, x3, x4 là các so nguyên tuỳ ý. Ví dụ 1.2.3. Giải phương trình 6x + 15y + 10z = 3. (1.9) Lời giái. Ta có the viet (1.9) dưói dạng 6(x + z) + 15y = 3 − 4z. Đ t u = x+z ta có 15y +4z = 3− 6u. Ta thay (−1,4) là nghi m riêng của 15y+ 4z = 1. Do đó (−3+6u,12−24u) là nghi m riêng của 15y+4z = 3−6u. Suy ra nghi m tong quát của nó là y = −3 + 6u + 4t, z = 12 − 24u − 15t.
  • 19. 19 Viết đề tài giá sinh viên – ZALO:0973.287.149-TEAMLUANVAN.COM − Từ u = x + z suy ra x = u − z = u − (12 − 24u − 15t) = −12 + 25u + 15t. V y nghi m tong quát của (1.9) là x = 12 + 25u + 15t, y = −3 +6u + 4t, z = 12 −24u− 15, vói u, t ∈ Z. M t so bài toán chon loc Bài toán 1.2.4 (Đe thi Vô định Toán Quoc te 1983). Cho a, b, c là các so nguyên đôi m t nguyên to cùng nhau. Chŕng minh rang 2abc − ab − bc − ca là so nguyên lớn nhat không viet được dưới dạng xbc + yca + zab với x, y, z là nhrng so không âm. Lời giái. Bài toán tương đương vói vi c chứng minh rang Khȁng định 1. phương trình xbc + yca + zab = 2abc − ab − bc − ca không có nghi m nguyên không âm; Khȁng định 2. Neu n > 2b − ab − bc − ca thì phương trình có nghi m nguyên không âm. Chŕng minh Khȁng định 1. Giả sử ton tại x0,y0,z0 ∈ N∗ sao cho x0bc + y0ca + z0ab = 2abc − ab − bc − ca. Đieu này tương đương vói bc(x0 +1)+ ca(y0 +1) +ab(z0 + 1) = 2abc.
  • 20. 20 Viết đề tài giá sinh viên – ZALO:0973.287.149-TEAMLUANVAN.COM y+ 2z+5u = b Suy ra ab(z0 + 1) . c. Vì (ab,c) = 1 nên z0 + 1 . c. M t khác z0 + 1 ∈ N∗ nên z0 + 1 ≥ c. Tương tụ y0 + 1 ≥ b, x0 + 1 ≥ a. V y bc(x0 +1)+ ca(y0 +1) +ab(z0 + 1) ≥ 3abc. Đieu này vô lí. Chŕng minh Khȁng định 2. Xét ab có dạng kbc + lac, vói 0 ≤ k ≤ a − 1, 0 ≤ l ≤ b−1.. Các so này khi chia cho ab sẽ cho ta các so dư khác nhau. Th t v y, giả sử k1bc +l1ac ≡ k2bc + l2ac (mod ab). Vì (ab,c) = 1 nên suy ra b(k1 −k2)−a(l1 −l2) . ab. Từ đó b(k1 −k2) . a và a(l1 − l2) . b. Vì (a,b) = 1 nên k1 −k2 . a và l1 −l2 . b. M t khác |k1 −k2| < a, |l1 −l2| < b nên l1 −l2 = 0, k1 −k2 = 0. Do đó ton tại x0, y0 ∈ Z+, 0 ≤ x0 ≤ a−1, 0 ≤ y0 ≤ b−1 đe cho bcx0 +acy0 ≡ n (mod ab). V y ton tại z0 ∈ Z đe bcx0 + acy0 + abz0 = n. Ta có ab(z0 +1) = n−bcx0 −acy0 +ab > 2abc − bcx0 − acy0 − bc − ca = bc(a− 1−x0)+ac(b−1− y0) ≥ 0. Do đó z0 + 1 > 0. Suy ra z0 ≥ 0. Bài toán 1.2.5. Cho các so nguyên không âm a, b thoá mãn đieu ki n 5a ≥ 7b. Chŕng minh rang h x+2y+ 3z+ 7u = a, luôn có nghi m nguyên không âm.
  • 21. 21 Viết đề tài giá sinh viên – ZALO:0973.287.149-TEAMLUANVAN.COM 5 5 5 5 5 5 5 5 5 5 5 5 Chŕng minh. Giả sử b = 5u + v vói 0 ≤ u, 0 ≤ v ≤ 4. Từ đieu ki n a ≥ 7b suy ra a ≥ 7(5u+v) . Từ đây ta có a − 7u ≥ 7v . (1.10) Như v y h phương trình có the viet là , x+2y+3z = a−yu y+2z = b−5u = v. (1.11) Vói mői 0 ≤ v ≤ 4 ta sẽ chon y, z thích hop đe có x = z − 7u − 2y − 3z ≥ 0. • Neu v = 0, ta lay y = z = 0. Khi đó x = a − 7u ≥ 7v = 0 (theo (1.10))). • Neu v = 1, ta thay y = 1, z = 0. Khi đó V y x ≥ 0. x = a − yu − 2 ≥ 7v − 2 = 7 − 2 > −1. • Neu v = 2, ta lay y = 0, z = 1. Khi đó V y x ≥ 0. x = a − 7u − 3 ≥ 7v − 3 = 14 − 3 > −1. • Neu v = 3, ta lay y = z = 1. Khi đó V y x ≥ 0. x = a − 7u − 5 ≥ 7v − 5 = 21 − 5 > −1. • Neu v = 4, ta lay y = 0, z = 2. Khi đó V y x ≥ 0. x = a − 7u − 6 ≥ 7v − 6 = 28 − 6 > −1. Tóm lại vói moi v = 0, 1, 2, 3, 4 ta đeu chon đưoc y, z, x không âm đe (1.11) đưoc nghi m đúng. Bài toán đưoc giải xong.
  • 22. 22 Viết đề tài giá sinh viên – ZALO:0973.287.149-TEAMLUANVAN.COM 5 5 2b 2b Bài toán 1.2.6 (Đe thi vô địch Mỹ 1982). Chŕng minh rang ton tại so tự nhiên k sao cho tat cá các so kn +1 với n = 1,2,... đeu là hợp so. Lời giái. Xét các so Fermat Fm = 22m +1. Ta đã biet Fm là so nguyên to vói m = 0,1,2,3,4, trong khi Euler phát hi u rang F5 là m t hop so, F5 là tích của hai so nguyên to 641 và 6700417. Ký hi u ai = Fi vói i = 0,1,2,3,4, a5 = 641a6 = 6700417. Theo Định lí Trung Hoa ve th ng dư, ton tại so tụ nhiên k > max(a0,...,a6) đe k ≡ 1 (mod am) vói m = 0,1,2,...,5 và k ≡ −1 (mod a6). Ta sẽ chứng minh k chính là so can tìm. Xét n bat kỳ, n có the viet dưói dạng n = 2m p vói 0 ≤ m và p lẻ. (a) Neu m ≤ 4, ta có k · 2n + 1 ≡ 2n + 1 (mod am). M t khác 2n + 1 = 22m p + 1 = (22m )p + 1 . 22m + 1 = am . Do đó k · 2n + 1 . am . Vì k ·2n +1 > k > am nên k ·2n +1 là hop so. (b) Neu m = 5 thì k · 2n + 1 ≡ 2n + 1 (mod a5). M t khác 2n +1 = 225 p +1 . 225 +1 = F . 641 = a . V y k·2n +1 . a . M t 5 5 khác k · 2n + 1 > a5, suy ra k · 2n + 1 là hop so. (c) Neu m ≥ 6, khi đó n = 26 b vói b ∈ Z. Ta có k · 2n + 1 ≡ −2n + 1 (mod a6), 2n = 22 5 2b = (F −1) ≡ (−1) ≡ 1 (mod a 6) vì F5 . a6. Do đó k ·2n +1 ≡ −1+1 = 0 (mod a6). Vì k · 2n +1 > a6 nên k ·2n +1 là hop so. V y vói moi n, so k · 2n + 1 luôn là hop so.
  • 23. 23 Viết đề tài giá sinh viên – ZALO:0973.287.149-TEAMLUANVAN.COM .α − . < . h 1 Chương 2 M t so phương trình Diophantine phi tuyen 2.1 Phương trình Pell loại 1 Phương trình Pell loại 1 là phương trình có dạng x2 − Dy2 = 1, (2.1) trong đó D ∈ N∗ và ta yêu cau tìm nghi m x,y ∈ N∗ . Trong tiet này khi nói đen nghi m của (2.1) ta hieu là nghi m nguyên dương. Định lí 2.1.1 (Đieu ki n ton tại nghi m). Phương trình (2.1) có nghi m nguyên dương khi và chỉ khi D là so không chính phương. Chŕng minh. Giả sử D = m2 . Khi đó x2 −Dy2 = x2 −m2 y2 = 1 → (x − my)(x + my) = 1 → x − my = x + my = 1 → x = 1,y = 0. V y (2.1) không có nghi m nguyên dương. Ngưoc lại giả sử D là so không chính phương. Ta có các bo đe sau Bo đe 2.1.2. Cho α ∈ / Q. Khi đó ton tại vô so c p nguyên (h,k) với k > 0 sao cho . k. k2
  • 24. 24 Viết đề tài giá sinh viên – ZALO:0973.287.149-TEAMLUANVAN.COM 1 h0 1 1 k h0 1 1 h0 √ x 1 √ √ √ x x 1 √ √ √ √ x x 2 2 2 .α − . < . A = (h,k) : .α − h. < 1 . a phải. chứng minh |A| = ∞. Giả sử trái lại |A| < ∞. Khi đó ton tại ε sao cho .α − h. > ε vói moi (h,k) ∈ A. Chon q ∈ N∗ sao cho . k0 . k0q 2 0 Từ (2.3) ta có (h0,k0) ∈ A → .α − . > ε. Nhưng ≥ > .α − . > ε. Mâu 0 < . D− . < . .y . .y |x − Dy | =|x − y D||x + y D| = y . − D.. + D. < y2 1 ( 1 +2 D) = 1 +2 D < 1 +2 D. Chŕng minh. Ta sử dụng nh n xét đã biet sau: Vói mői q ∈ N∗ ton tại c p so nguyên (h,k) vói 1 ≤ k ≤ q sao cho Ký hi u . k. kq . k. k2 . k. q < ε. (2.2) Theo nh n xét trên ton tại c p so nguyên (h0,k0) vói 1 ≤ q sao cho .α − . < ≤ . (2.3) thuȁn vói (2.2). . k0 . q k0q . k0 . Bo đe 2.1.3. Ton tại vô so c p so nguyên dương (x,y) sao cho |x2 − Dy2 | < 1 + 2 √ D. Chŕng minh. Theo Bo đe 2.1.2 ton tại vô so c p so nguyên (x,y) sao cho Suy ra . y. y2 . + D. = . − D + 2 D. < + 2 √ D. √ .y √ ..y √ . y2 y2 y2 V y h 1 y2
  • 25. 25 Viết đề tài giá sinh viên – ZALO:0973.287.149-TEAMLUANVAN.COM 1 1 Chŕng minh định lý Từ bo đe 2.1.3 (x,y) ton tại vô so c p so nguyên dương (x,y) sao cho |x2 − Dy2 | < 1 + 2 √ D. Đ t I = [−1−2 √ d;1+2 √ d]. Vói mői k ∈ I ký hi u Ak = {(x,y) ∈ N∗ : x2 − Dy2 = k} Do đó ton tại k ∈ I đe |Ak| = ∞. Suy ra ton tại (x1,y1) /= (x1,y1) ∈ Ak đe x1 ≡ x2 (mod |k|) y1 ≡ y2 (mod |k|), x2 − Dy2 = x2 − Dy2 = k, Xét tích 1 1 2 2 (x1 − y1 √ D)(x2 + y2 √ D)x1x2 − Dy1y2 + √ D(x1y2 − x2y1) (2.4) Vì x1x2 −Dy1y2 ≡ x2 −Dy2 ≡ 0 (mod |k|) x1y2 −x2y1 ≡ x1y1 −x1y1 ≡ 0 (mod |k|). V y ton tại u,v ∈ Z sao cho Từ (2.4), (2.5), (2.6) suy ra x1x2 −Dy1y2 = ku (2.5) x1y2 −Dy1x2 = kv (2.6) (x1 − y1 √ D)(x2 + y2) = k(u + v √ D), (x1 + y1)(x2 − y2 √ D) = k(u − v √ D). Nhân hai đȁng thức trên vói nhau và chú ý rang (x1,y1,(x2,y2) ∈ Ak → x2 −dy2 = 1 1 x2 −dy2 = k ta đưoc 2 2 k2 = k2(u2 − Dv2) → u2 −Dv2 = 1.
  • 26. 26 Viết đề tài giá sinh viên – ZALO:0973.287.149-TEAMLUANVAN.COM — − 1 1 n n Ta chứng minh u,v > 0. Rõ ràng u > 0. Neu trái lại v = 0 thì u = ±1 → (x1 − y1 √ D)(x2 + y2 √ D) = ±k = ±(x2 − Dy2 ) = ±(x1 − y1 √ D)(x1 + y1 √ D) → x2 + y2 √ D = x1 + y1 √ D → x1 = x2,y1 = y2. Ta có mâu thuȁn. V y (u,v) là nghi m nguyên dương của phương trình (2.1). Định lí 2.1.4 (Công thức nghi m). Ký hi u (a,b) là nghi m nhó nhat của phương trình Khi đó dãy (xn,yn) cho bới x x2 − Dy2 = 1. (a + b √ D)n + (a − b √ D)n n = 2 , yn = (a+ b √ D)n (a b √ D)n 2 √ D . cho tat cá các nghi m của (2.2) Dãy nghi m (xn,yn) cũng có the xác định theo công thức truy hoi sau x0 = 1,x1 = a,xn+2 = 2axn+1 −xn, (2.7) y0 = 0,y1 = b,yn+2 = 2ayn+1 −yn. (2.8) Chŕng minh. Ta có xn +yn √ D = (a + b √ d)n ,xn −yn = (a − b √ d)n . (2.9) Suy ra (x2 − Dy2 ) = (a2 − Db2)n = 1. Đảo lại giả sử (x,y) là nghi m bat kỳ của (2.1). Ta chứng minh ton tại n ∈ N∗ đe x = xn;y = yn. Vì D không chính phương nên đieu này tương đương Ton tại n ∈ N∗ đe x+y √ D = xn +yn √ D = (a+b √ D)n . Chứng minh bang phản ứng. Giả sử trái lại x + y √ D /= (a +b √ D)n ∀n ∈ N∗.
  • 27. 27 Viết đề tài giá sinh viên – ZALO:0973.287.149-TEAMLUANVAN.COM m m Khi đó ton tại m ∈ N∗ sao cho (a +b √ D)m < x + y √ D < (a + b √ D)m+1 . Nhân hai ve vói (a − b √ D)m ta đưoc 1 < (x+y √ D)(a−b √ D)m < a+b √ D. Do (2.9) ta có (x + y √ D)(a − b √ D)m = (x + y √ D)(xm − ym √ D) = (xxm − Dyym) + (xmy − ymx) √ D = u + v √ D, ỏ đó u = xxm − Dyym,v = xmy − ymx. V y 1 < u + v √ d < a + b √ d. (2.10) Ta có u2 − Dv2 = (xxm − Dyym)2 − D(xmy − ymx)2 , = (x2 − Dy2 )(x2 − Dy2 ) = 1. Lại có x > y √ d, xm > ym √ d nên u > 0. Lại có (u − v √ D)(u + v √ D) = 1 và 1 < u + v √ d nên 0 < u − v √ d < 1 < u + v √ d → v > 0. V y (u, v) là nghi m của (2.1) do đó a ≤ u,b ≤ u → a + b √ d ≤ u + v √ d trái vói (2.10). Định lý đưoc chứng minh. Từ định lý trên ta thay vi c tìm nghi m của phương trình Pell (2.1) quy ve tìm nghi m nhỏ nhat (a, b) của nó. Cách đơn giản nhat là thử bang tay: Thay lan lưot y = 1,2,... vào bieu thức 1 + Dy2 cho tói khi nào đưoc so chính phương thì dừng lại. Vì phương trình (2.1) có nghi m nên chac chan quá trình này sẽ dừng lại sau b phép thử. Khi đó nghi m nhỏ nhat là (a,b) vói a = √ 1+Db2 . Neu nghi m nhỏ
  • 28. 28 Viết đề tài giá sinh viên – ZALO:0973.287.149-TEAMLUANVAN.COM nhat b này lón thì cách thử này không khả thi. Thí dụ vói phương trình x2 − 61y2 thì c p nghi m nhỏ nhat là a = 1766319049, b = 226153980. Sau đây ta sẽ trình bày m t thu t toán sử dụng liên phân so đe tìm m t nghi m nhỏ nhat (a,b) của (2.1). Định nghĩa 2.1.5. Cho a0,a1,a2,... là dãy so nguyên trong đó ai > 0,i ≥ 1. Đ t Ck = [a0;a1,...,ak]. Khi đó ton tại giói hạn limCk = α. Ta goi α là giá trị của liên phân so vô hạn [a0,a1,a2,...]và viet α = [a0,a1,a2,...]. Định lí 2.1.6. α = [a1, a1, a2, . . .] là m t so vô tý. Ngược lại mői so vô tý đeu bieu dien duy nhat dưới dạng m t liên phân so vô hạn. Định nghĩa 2.1.7. Ta goi liên phân so vô hạn α = [a1; a1, a2, . . .] là tuan hoàn neu dãy (an) tuan hoàn ke từ m t chỉ so nào đó tức là: Ton tại các so nguyên dương m và k sao cho moi n ≥ m ta có an = am+k. So nguyên dương k đưoc goi là chu kỳ của liên phân so α = [a0;a1,a2,...]. Khi đó ta viet α = [a0;a1,a2,...,am−1,am,am+1,...,am+k−1]. Neu D là so không chính phương, bieu dien liên phân so của √ D đưoc cho bỏi Định lí 2.1.8. Bieu dien liên phân so của √ D là tuan hoàn và có dạng √ D = [a;a1,a2,...,an,2a]. với a = [ √ D]. Hơn nra có công thŕc tường minh đe xác định dãy (a1,...,an). Chú ý rang dãy (a1,...,an) là đoi xŕng tŕc là a1 = an,a2 = an−1,...
  • 29. 29 Viết đề tài giá sinh viên – ZALO:0973.287.149-TEAMLUANVAN.COM n−1 2n−1 Ví dụ 2.1.9. √ 23 = [4; 1,3,1,8] √ 29 = [5; 2, 1,1,2, 10] √ 31 = [5;1,1,3,5,3,1,1,10] √ 46 = [6;1,2,1,1,2,6,2,1,1,2,1,12] √ 76 = [8;1,2,1,1,5,4,5,1,1,2,1,16] √ 97 = [9;1,5,1,1,1,1,1,1,5,1,18]. Định lí 2.1.10. Cho phương trình Pell x2 − Dy2 = 1. (I) 1. Bieu dien √ D thành liên phân so √ D = [a;a1,a2,...,an,2a]. 2. Neu chu kỳ n là so chȁn ta tính gián phân thŕ n − 1 C = pn−1 . qn−1 3. Khi đó (pn−1, qn−1) là nghi m nhó nhat của (2.1) 4. Neu chu kỳ n là so lé ta tính gián phân thŕ 2n−1 C = p2n−1 . q2n−1 5. Khi đó (p2n−1, q2n−1) là nghi m nhó nhat của (2.1) Ví dụ 2.1.11. Tìm nghi m nhỏ nhat của phương trình x2 − 14y2 = 1. Ta có √ 14 = [3;1,2,1,6]. Chu kỳ n = 4 là so chȁn. V y ta tính giản phân C3 = [3,1,2,1] = 3 + 1 1+ 1 2 + 1 1
  • 30. 30 Viết đề tài giá sinh viên – ZALO:0973.287.149-TEAMLUANVAN.COM 15 = 4 . V y nghi m nhỏ nhat là (15,4). Ví dụ 2.1.12. Tìm nghi m nhỏ nhat của phương trình x2 − 13y2 = 1. Ta có √ 13 = [3;1,1, 1,1,6] = [3,1, 1,1,1,6, 1,1,1, 1,6,... Chu kỳ n = 5 là so lẻ. V y ta tính giải phân C9 = [3,1,1,1,1,6,1,1,1] = 3 + 1 = 649 . 1+ 1 1+ ... + 1 1 + 1 1 180 V y nghi m nhỏ nhat là (649,180) Trỏ lại phương trình x2 − 61y2 = 1. Ta có √ 76 = [7;1,4,3,1,2,2,1,3,4,1,14] Chu kỳ n = 11 là so lẻ. Ta tính giản phân C21 = [7,1,4,3,1,2,2,1,3,4,1,14,1,4,3,1,2,2,1,3,4,1] 1 = 7+ 1+ 1 4+ 1 3 + ... + 1 4 + 1 1 1766319049 = 226153980 V y nghi m nhỏ nhat là (1766319049,226153980) 2.2 Phương trình Pell loại 2 Phương trình Pell loại 2 là phương trình x2 − Dy2 = −1 (2.11) ỏ đó D ∈ N∗ . Nghi m của (2.11) luôn đưoc hieu là nghi m nguyên dương.
  • 31. 31 Viết đề tài giá sinh viên – ZALO:0973.287.149-TEAMLUANVAN.COM 1 Định lí 2.2.1. Đieu ki n can đe (2.11) có nghi m là D không là so chính phương và D không có ước nguyên to dạng 4k+3. Chŕng minh. i) neu D = m2 thì (2.11) trỏ thành (my− x)(my +x) = 1 → my− x = my+x = 1 → x = 0 v y (2.11) vô nghi m. ii) Neu D có ưóc nguyên to p = 4k+3 : Giả sử (x,y) là nghi m. Khi đó x2 +1 = Dy2 → p|x2 + 1. Vì p = 4k + 3 nên p|1. Mâu thuȁn. Tuy nhiên, đây không là đieu ki n đủ. Định lí 2.2.2. Neu D = p là so nguyên to thì (2.11) có nghi m khi và chỉ khi p = 2 ho c p /= 4k + 3. Chŕng minh. Neu (2.11) có nghi m thì từ Định lý 1 suy ra p = 2 ho c p /= 4k +3 . Đảo lại neu p = 2 thì phương trình x2 = −2y2 = −1 có nghi m (x, y) = (1; 1). Xét p ≡ 1( mod 4). Xét phương trình Pell loại I x2 − Dy2 = 1 (2.12) Goi (a,b) là nghi m nhỏ nhat của (2.12). Ta có a2 − 1 = pb2 . Neu a chȁn thì b lẻ do đó b2 ≡ 1( mod 4) → a2 ≡ 1 + p ≡ 2( mod 4). Mâu thuȁn. V y a lẻ, b chȁn. Đ t a = 2a1 + 1,b = 2b1. Ta có (a − 1)(a + 1) = pb2 ⇔ a1(a1 + 1) = pb2 . Do p nguyên to và (a1,a1 +1) = 1 nên a1 = u2 ,a1 +1 = pv2 ho c a1 = pu2 ,a1 +1 = v2 . Neu a1 = u2 , a1 + 1 = pv2 → u2 − pv2 = −1. V y (2.11) có nghi m (u, v). Neu a1 = pu2 , a1 + 1 = v2 → v2 − pu2 = 1. V y (v, u) là nghi m của (2.12). Suy ra u ≥ a → a1 +1 = v2 ≥ v ≥ a = 2a1 +1. Mâu thuȁn. Định lí 2.2.3. Goi (a, b) là nghi m nhó nhat của (2.12). Khi đó (2.11) có nghi m khi và chỉ khi a = x2 +Dy2 b = 2xy (2.13)
  • 32. 32 Viết đề tài giá sinh viên – ZALO:0973.287.149-TEAMLUANVAN.COM 1 2 2 2 1 1 2 2 2 2 2 2 2 2 1 1 1 có nghi m nguyên dương. Hơn nữa neu (2.13) h có nghi m nó sẽ có nghi m duy nhat. Nghi m duy nhat (x1,y1) này chính là nghi m nhỏ nhat của (2.11). Chŕng minh. 1) Giả sử (2.11) có nghi m. Goi (x1,y1) là nghi m nhỏ nhat của (2.11). Đ t u = x2 +Dy2 ,v = 2x1,y1. Ta chứng minh u = a,v = b do đó (x1,y1) 1 1 chính là nghi m của (2.13). Ta có u2 −Dv2 = (x2 − Dy2 )2 = 1. V y (u,v) là 1 1 nghi m của (2.12). Suy ra u ≥ a;v ≥ b. Ta có (a2 − Db2 )(x1 − Dy2 ) = 1(−1) = −1 ⇔(ax1 − Dby1) − D(ay1 −bx1) = −1. De thay (ax1 − Dby1)2 > 0, (ay1 − bx1)2 > 0. Do (x1, y1) là nghi m của (2.11) nên (ax1 −Dby1)2 ≥ x2 ⇔ a2 x2 +D2 b2 y2 −x2 ≥ 2abDx1y1 1 1 1 1 2 2 2 2 2 → x1(a −1)+D b y1 ≥ 2abDx1y1 → x1Db2 + D2 b2 y2 ≥ 2abDx1y1 2 2 → b(x1 + dy1) ≥ 2ax1y1 → bu ≥ av → b u ≥ a v → b (Dv +1) ≥ v (Db +1) → b ≥ v → b = v → u = a. 2) Đảo lại giả sử (u, v) là nghi m của (2.13). Ta có a2 − Db2 = 1 ⇔ (u2 + Dv2 )2 − D(2uv)2 = (u2 − Dv2 )2 = 1. V y u2 − Dv2 = 1 ho c u2 − Dv2 = −1. Neu u2 − Dv2 = 1 thì (u, v) là nghi m của (2.12) do đó u ≥ a = u2 + Dv2 . Mâu thuȁn. V y u2 − Dv2 = −1 do đó (2.11) có nghi m (u,v). Tiep theo ta chứng minh (u,v) là nghi m nhỏ nhat của (2.11). Giả sử (x1,y1) là nghi m nhỏ nhat của (2.11). Theo chứng minh ỏ 1) ta có a = u2 +Dv2 = x2 + Dy2 ;b = 2uv = 2x1y1 → u2 +Dv2 +2uv = x2 +Dy2 +2x1y1 → (u+v √ D)2 = (x1 +y1)2 → u = x1;v = y1. Định lý đưoc chứng minh.
  • 33. 33 Viết đề tài giá sinh viên – ZALO:0973.287.149-TEAMLUANVAN.COM 6 = 2xy n−1 Ví dụ là m t áp dụng của định lý 2. Nó cũng chỉ ra rang đieu ki n của định lý 1 chỉ là đieu ki n can. Ví dụ 2.2.4. Chứng minh rang phương trình x2 − 34y2 = −1 vô nghi m (Ő đây 34 = 2.17 không so chính phương và cũng không có ưóc nguyên to dạng 4k + 3). Giái Phương trình x2 −34y2 = 1 có nghi m nhỏ nhat là (a;b) = (35;6). Xét h 35 = x2 +34y2 Từ phương trình thứ nhat của h suy ra (x;y) = (1;1). Tuy nhiên (1;1) không thoả mãn phương trình thứ hai. V y h vô nghi m do đó theo định lý 2 phương trình x2 −34y2 = −1 vô nghi m. Ta thừa nh n định lý Định lí 2.2.5. Phương trình Pell loại 2 x2 − Dy2 = −1 có nghi m khi và chỉ khi trong bieu dien √ D thành liên phân so √ D = [a; a1, a2,...,an, 2a] chu kỳ n là so lé. Trong trường hợp đó (pn−1, qn−1) là nghi m nhó nhat của phương trình ớ đó. là giái phân thŕ n−1. C = pn−1 qn−1 Ví dụ 2.2.6. i) Xét phương trình x2 − 13y2 = −1. Ta có √ D = [3;1,1,1,1,6]. Chu kỳ n = 5 là so lẻ. V y phương trình có nghi m. Ta tính giản phân C4 = [3,1,1,1,1]
  • 34. 34 Viết đề tài giá sinh viên – ZALO:0973.287.149-TEAMLUANVAN.COM n=1 − n n 1 = 3 + 1 1+ 1 1+ 1 3 18 = 3 + 5 = 5 V y nghi m nhỏ nhat là (18,5) 1 + 1 ii) Xét phương trình x2 −34y2 = −1. Ta có √ 34 = [5;1,4,1,10]. Chu kỳ n = 4 là so chȁn. V y phương trình vô nghi m. Định lí 2.2.7 (Công thức nghi m). Giá sr phương trình Pell loại 2 x2 − Dy2 = −1 (II) có nghi m. Goi (α,β ) là nghi m nhó nhat của nó. Khi đó dãy (xn, yn)∞ cho bới x (α + β √ D)2n−1 + (α − β √ D)2n−1 n = 2 yn = (α + β √ D)2n−1 + (α β √ D)2n−1 2 √ D cho ta tat cá các nghi m của (II). Chŕng minh. Giả sử (xn,yn) cho bỏi công thức trên. Khi đó xn + yn √ D = (α + β √ D)2n−1 xn −yn √ D = (α −β √ D)2n−1 → x2 − Dy2 = (a2 − Dβ 2 )2n−1 = −1 Ngưoc lại giả sử (x,y) là m t nghi m của (2.11). Ta có (x + y √ D)(α + β √ D) = (xα + Dyβ )(yα + xβ ) √ D = s +t √ D ỏ đó s = xα + Dyβ,t = yα + xβ → s2 −Dt2 = (xα +Dyβ)2 −D(yα +xβ)2
  • 35. 35 Viết đề tài giá sinh viên – ZALO:0973.287.149-TEAMLUANVAN.COM 4 = (x2 − Dy2 )(α2 − Dβ 2 ) = (−1)(−1) = 1 V y (s,t) là nghi m của phương trình Pell loại 1 s2 −Dt2 = 1. Goi (a,b) là nghi m nhỏ nhat của nó. Theo công thức nghi m của phương trình Pell loại 1 và định lý 3 ton tại n ∈ N∗ sao cho (x + y √ D)(α + β √ D) = s +t √ D = (a + b √ D)n = (α2 + Dβ 2 + 2αβ √ D)n = (α + β √ D)2 )n = (α + β √ D)2n → x + y √ D = (α +β √ D)2n−1 = xn + yn √ D → x = xn,y = yn M t so bài toán chon loc Bài toán 2.2.8. So nguyên dương S được goi là so Heron neu nó là di n tích của m t tam giác có ba cạnh là ba so nguyên liên tiep. Chŕng minh rang S là so Heron khi và chỉ khi S khi là so hạng của dãy (Sn),n ≥ 1 xác định bới S0 = 0,S1 = 6,S2 = 84,Sn+2 = 14Sn+1 − Sn Lời giái. Goi S là di n tích của tam giác có ba cạnh là x − 1,x,x + 1 vói x > 2,x ∈ N∗. Theo công thức Heron. S = 1 x q 3(x2 − 4) → 16S2 = 3x2 (x2 − 1) (2.14) V y S là so Heron khi và chỉ khi phương trình (2.14) có nghi m nguyên dương(S,x). De thay x phải chȁn. Đ t x = 2y ta có 16S2 = 3x2 (x2 − 1) ⇔ S2 = 3y2 (y2 − 1) → S = y q 3(y2 − 1) → 3(y2 = −1) = h2
  • 36. 36 Viết đề tài giá sinh viên – ZALO:0973.287.149-TEAMLUANVAN.COM 2 2 2 2 √ √ √ n n 4 k → h = 3z → 3(y −1) = 9z → y −3z = 1,S = 3yz Ngưoc lại neu (y,z) là nghi m của phương trình Pell y2 − 3z2 = 1 (2.15) thì x = 2y,y > 1,S = 3yz là nghi m của (2.14) Nghi m nhỏ nhat của (2.15) là (2,1) V y tat cả nghi m của (2.15) (yn,zn) cho bỏi Do đó yn = zn = (2 + √ 3)n + (2 − √ 3)n 2 (2+ 3) −(2 − 3) 2 3 Sn = 3ynzn = √ 3 ((7+ √ 3)n −(7 − 4 √ 3)n ) → Sn+2 = 14Sn+1 −Sn,S0 = 0,S1 = 6,S2 = 84 Bài toán 2.2.9. Tìm tat cá các so nguyên dương n sao cho 2n + 1 và 3n + 1 đeu là so chính phương. Lời giái. Vì (2n + 1, 3n + 1) = 1 nên 2n + 1 và 3n + 1 đeu là so chính phương khi và chỉ khi (2n + 1)(3n + 1) = y2 , y ∈ N∗. Suy ra (12n + 5)2 − 24y2 = 1. Đ t x = 12n + 5 ta có x2 − 24y2 = 1. Goi (xk,yk) là nghi m của nó. Nghi m nhỏ nhat của phương trình Pell này là (5,1). Do đó (xk) cho bỏi h thức. x0 = 1,x1 = 5,xk+2 = 10xk+1 − xk. De chứng minh xk ≡ 5 (mod 12) khi vào chỉ khi k lẻ. V y n = nk,k ≥ 1 ỏ đó n = x2k+1 −5 12 Ta xác định h thức truy hoi của dãy (nk)
  • 37. 37 Viết đề tài giá sinh viên – ZALO:0973.287.149-TEAMLUANVAN.COM √ √ 2n 1 2n 1 Đ t uk = x2k+1 = 12nk +5. Ta có x2k+3 = 10x2k+2 − x2k+1 = 10(10x2k+1 − x2k) − x2k+1 = 100x2k+1 − 10x2k − x2k+1 = 99x2k+1 = x2k+1 − x2k−1 = 98x2k+1 − x2k−1 → uk+2 = 98uk+1 −uk ⇔ 12nk+2 + 5 = 98(12nk+1 + 5) = 12nk − 5 nk+2 = 98nk+1 −nk +40 vói n1 = 40,n2 = 3960 Bài toán 2.2.10. Cho dãy (xn,yn) xác định như sau (x0,y0) = (0,1),(x1,y1) = (3,5) và xn+1 = 3xn +2yn +1 yn+1 = 4xn +3yn +2 (2.16) Chŕng minh rang (xn,yn) là tat cá các nghi m nguyên dương của phương trình x2 +(x+ 1)2 = y2 Lời giái. Phương trình đã cho tương đương vói (2x + 1)2 − 2y2 = −1 (2.17) Đ t u = 2x + 1, → u2 − 2y2 = −1. Nghi m nhỏ nhat của phương trình này là (1.1). Do v y dãy nghi m (un,yn) cho bỏi un = (1 + √ 2)2n+1 + (1 − √ 2)2n+1 2 (1+ 2) + −(1− 2) + Từ đó yn = 2 √ 2 u0 = 1,u1 = 7,uk+2 = 6uk+1 −uk
  • 38. 38 Viết đề tài giá sinh viên – ZALO:0973.287.149-TEAMLUANVAN.COM y0 = 1,y1 = 5,yk+2 = 6yk+1 −yk Ta có un = 2xk+2 +1 → 2xk+2 +1 = 6(2nk+1 +1)−2nk −1 → x0 = 0,x1 = 3,xk+2 = 6xk+1 −xk +2. Thành thử dãy nghi m (xn,yn) của (2.17) cho bỏi x0 = 0,x1 = 3,xn+2 = 6xn+1 −xn +2 y0 = 1,y1 = 5,yn+2 = 6yn+1 = yk Thành thử ta chỉ can chứng minh dãy (2.16) thoả mãn quan h xn+2 = 6xn+1 −xn +2 yn+2 = 6yn+1 − yk Th t v y đ t zn = 2xn + 1. Khi đó de kiem tra zn+1 = 3zn + 4yn yn+1 = 2zn + 3yn → zn+2 = 3zn + 1 + 4yn+1 = 3zn+1 + 8zn + 12yn = 3zn+1 + 8zn +3zn+1 −9zn = 6zn+1 −zn → 2xn+2 + 1 = 6(2xn+1 +1) − 2xn−1 → xn+2 = 6xn+1 −xn +2 Tương tụ yn+2 = 6yn+1 −yk 2.3 Phương trình Pythagoras Trong mục này chúng ta sẽ đi tìm tat cả các nghi m nguyên dương của phương trình x2 +y2 = z2 (2.18) và xét m t so ứng dụng của nó. Những phương trình Diophantine này đưoc goi là các phương trình Pythagoras.
  • 39. 39 Viết đề tài giá sinh viên – ZALO:0973.287.149-TEAMLUANVAN.COM Định nghĩa 2.3.1. M t b ba so nguyên dương (x, y, z) thoả mãn phương trình (2.18) goi là m t b ba so Pythagoras. Các b ba so Pythagora bieu thị đ dài các cạnh của m t tam giác vuông. Từ m t b ba so Pythagoras (x, y, z) nào đó ta suy ra de dàng m t t p hop vô hạn các b ba so Pythagoras khác (tx,ty,tz) vói t là so nguyên dương. Ngưoc lại, cho (x,y,z) là m t b ba so Pythagoras bat kỳ và giả sử d = (x,y,z). Khi đó (x1,y1,z1) cũng là m t b ba so Pythagoras, ỏ đó x x y z 1 = d , y1 = d , z1 = d , và (x1, y1, z1) = 1. Định nghĩa 2.3.2. M t b ba so Pythagoras (x,y,z) là nguyên thuý neu (x,y,z) = 1. Rõ ràng từ lý lu n trên ta chỉ can đi tìm các b ba so Pythagoras nguyên thuý. De dàng nh n thay neu (x, y, z) là m t b ba so Pythagoras nguyên thuý thì chúng đôi m t nguyên to cùng nhau. Th t v y, giả sử d = (x, y). Khi đó d2 là ưóc của z2 nên d là ưóc của z do đó d là ưóc chung của x, y, z. V y d = 1. Định lí sau đây cho công thức mô tả tat cả các b ba so Pythagoras nguyên thuý. Định lí 2.3.3. Giá sr (x,y,z) là m t b ba so Pythagoras nguyên thuý. Khi đó x và y có tính chȁn lé khác nhau. Neu x chȁn, y lé chȁng hạn (x,y,z) có dạng x = 2mn, y = m2 − n2 , n = m2 +n2 trong đó m, n là hai so nguyên đương nguyên to cùng nhau, có tính chȁn lé khác nhau. Đáo lại, neu m, n là hai so nguyên dương nguyên to cùng nhau, m t chȁn, m t lé thì ba b (x,y,z) xác định như trên là m t b ba so Pythagoras nguyên thuý. Chŕng minh. Trưóc het vì (x, y) = 1 nên x và y không cùng chȁn. Hai so x và y cũng không the cùng lẻ vì neu như v y thì z2 = x2 + y2 ≡ 2 (mod 4), đieu này không the có vì m t so chính phương chỉ đong dư 0 ho c 1 theo modulo 4.
  • 40. 40 Viết đề tài giá sinh viên – ZALO:0973.287.149-TEAMLUANVAN.COM 0 0 2 2 2 2 2 2 0 0 0 x2 − y2 = t2 , Giả sử x chȁn, y lẻ. Khi đó z lẻ và x2 = z2 − y2 (z + y)(z − y), đieu này tương đương vói x 2 = z+y z−y . (2.19) De thay rang z+y , z−y = 1 vì (z,y) = 1. Suy ra ton tại m, n ∈ N∗ đe z+y = m2 và z− 2 y = n2 . Từ đó x = 2mn, y = m2 − n2 , z = m2 + n2 . M t khác (m2 ,n2 ) = 1 nên (m,n) = 1. Hơn nữa vì y và z lẻ nên m, n có tính chȁn lẻ khác nhau. Bây giò ta chứng minh phan đảo của định trong định lí trên. De dàng kiem tra rang x, y, z xác định theo công thức trên là b ba so Pythagoras. Ta còn phải chứng minh đó là b ba so Pythagoras nguyên thuý. Giả sử d = (y,z). Khi đó vì y, z lẻ nên d lẻ. Ta có d | y+ z = 2m2 suy ra d | m2 và d | z−y = 2n2 suy ra d | n2 . Vì (m,n) = 1 nên từ đó d = 1. V y (y,z) = 1 suy ra (x,y,z) = 1. Định lí đưoc chứng minh. Sau đây là áp dụng quan trong của Định lí 2.3.3. Định lí 2.3.4. Không ton tại hai so tự nhiên x và y đe tőng các bình phương và hi u các bình phương của chúng đeu là các so chính phương. Chŕng minh. Ta chứng minh bang phản chứng. Giả sử ton tại các so nguyên dương x, y, có tính chat đã nêu. Trong các c p so (x,y) ta chon (x0,y0) là c p so mà x nhỏ nhat. Giả sử x2 + y2 = z2 , vói z0, t0 ≤ N∗. 0 0 0 Ta có (x0,y0) = 1. Th t v y, giả sử d = (x0,y0). Từ (2.20) suy ra d2 | z2 và d2 | t2 . Từ đó d | z0 , d | t0 . Đ t x0 , y1 = y0 , z1 = y z0 , t1 = t0 . d d d d (2.20)
  • 41. 41 Viết đề tài giá sinh viên – ZALO:0973.287.149-TEAMLUANVAN.COM 0 0 1 1 1 1 1 1 1 1 0 0 1 1 0 0 0 Ta có x2 +y2 = z2 , x2 − y2 = t2 . Suy ra (x1,y1) là m t c p thoả mãn tính chat đã nêu. Do đó x2 +y2 ≥ x2 +y2 = d(x2 +y2 ). V y d = 1. Từ (2.20) suy ra z2 +t2 = 2x2 . Do đó z0 và t0 có cùng tính chȁn lẻ. Đ t 0 0 0 u = z0 +t0 , v = z0 −t0 2 2 vói u, v là các so nguyên dương. Khi đó u + v = z0, u − v = t0vu2 + v2 = x2 . (2.21) Ta có (u,v) = 1, vì neu d = (u,v) thì d2 | x2 . M t khác d | u + v = z0, suy ra d2 | z2 − x2 = y2 suy ra d | y0. V y d = 1. Theo (2.21) thì (u, v, x0) là m t b ba so Pythagoras nguyên thuý. Theo Định lí 2.3.3 ton tại các so nguyên dương m, n chȁn lẻ khác nhau vói (m, n) = 1, m > n sao cho u = 2mn, v = m2 − n2 ; ho c u = 2m2 − n2 , v = 2mn. Trong moi trưòng hop, uv = 2mn(m2 − n2 ). Ta lại có 2y = (u + v)2 − (u − v)2 = 4uv = 8mn(m2 − n2 ) suy ra y = 4mn(m2 − n2 ), tức là y0 = 2k. V y mn(m2 − n2 ) = k2. (2.22) Ta có (m,n) = 1 nên (m + n,m) = 1 và (m − n,m) = 1. V y (m2 − n2 ,m) = 1.
  • 42. 42 Viết đề tài giá sinh viên – ZALO:0973.287.149-TEAMLUANVAN.COM 4 2 r 2 r 2 r Tương tụ, (m2 − n2 ,n) = 1. Do đieu này nên từ (2.3) suy ra ton tại a,b,c ∈ N∗ đe m = a2, n = b2, m2 − n2 = c2. (2.23) Goi d = (m + n,m −n). Do m, n chȁn lẻ khác nhau nên m + n, m − n lẻ, từ đó d lẻ. Vì d | m+n, d | m− n nên d | 2m, d | 2n suy ra d | m, d | n (do d lẻ). Suy ra d = 1. Do đieu này nên từ (2.23) suy ra ton tại r, s ∈ N∗ đe m − n = r2 , m + n = s2 . V y a2 +b2 = s2 , a2 − b2 = r2 , tức là c p (a,b) thoả mãn tính chat đã nêu trong định lí. M t khác a2 + b2 = m + n ≤ 2m ≤ 2mn ≤ u = z0 +t0 < z0 ≤ z2 = x2 = y2 . 2 0 0 0 Đieu này trái vói cách chon c p (x0,y0). Định lí đưoc chứng minh xong. M t so bài toán chon loc Bài toán 2.3.5 (Đe dụ tuyen thi Toán quoc te năm 1979). Chŕng minh rang không ton tại hình chóp tŕ giác đeu mà các cạnh, di n tích toàn phan và the tích của nó đeu là các so nguyên. Chŕng minh. Giả sử g, f , h, S và V theo thứ tụ là cạnh đáy, cạnh bên, chieu cao, di n tích toàn phan và the tích của m t hình chóp tứ giác đeu và chúng là những so nguyên. Ta có f = h2 + g , 2 g2 h V = 3 , S = g2 +2g h2 + g . 4 Vì g, S là so nguyên nên 2g q h2 + g2 là so nguyên. Kí hi u x = 2g2 h2 + g , 4 y = g3 .
  • 43. 43 Viết đề tài giá sinh viên – ZALO:0973.287.149-TEAMLUANVAN.COM 3 0 0 0 0 0 0 0 0 0 0 0 0 0 0 0 0 0 0 0 0 0 0 0 0 0 0 0 0 0 0 1 1 0 1 1 0 De thay x, y ∈ N∗. Ta có x2 − y2 = 4g4 22 + g 4 —g6 = 4h2 g4 = (2g2 h)2 là so chính phương, vì V = g 2h ∈ N∗. Do đó g2 ∈ N∗. M t khác, x2 +y2 = 4g4 h2 +2g6 = 4g4 h2 + g 2 = 4g4 f2 = (2g2 f )2 . Vì 2g2 f ∈ N∗ nên x2 + y2 là so chính phương. Song đieu này mâu thuȁn vói Định lí 2.3.4. Ta có đieu can chứng minh. Bài toán 2.3.6. Chŕng minh rang phương trình x4 − y4 = z2 không có nghi m nguyên dương. Chŕng minh. Ta sẽ chứng minh bài toán này bang phương pháp phản chứng. Giả sử (x,y,z) là nghi m nguyên dương của phương trình đã cho và d = (x,y). Ta có x = dx0, y = dy0, (x0,y0) = 1. Thay vào phương trình cho ta d4 (x4 −y4 ) = z2 suy ra d2 | z. Đ t z = d2 z0 suy ra x4 −y4 = z2 , tức là (x2 −y2 )(x2 +y2 ) = z2 . Neu x0 và y0 chȁn lẻ khác nhau thì (x2 −y2 ,x2 +y2 ) = 1 bỏi vì neu d1 | (x2 −y2 ) và d1 | (x2 +y2 ) thì d1 | 2y2 và d1 | 2x2 . V y d1 = 1 (do d1 lẻ). Do đó x2 − y2 = a2 , x2 − y2 = b2 , trái vói Định lí 2.3.4. Neu x0 và y0 cùng lẻ thì x2 +y2 = 2a, x2 +y2 = 2b Suy ra x2 = a+b, y2 = a−b. Ta có 4ab = z2 suy ra ab = z2 . Vì (x0,y0) = 1 nên (a,b) = 1. Do đó a = a2 , b = b2 . 0 1 1 1 V y a2 + b2 = x2 , a2 − b2 = y2 , và đieu này mâu thuȁn vói Định lí 2.3.4. 2 2
  • 44. 44 Viết đề tài giá sinh viên – ZALO:0973.287.149-TEAMLUANVAN.COM Bài toán 2.3.7. Chŕng minh rang phương trình x4 +y4 = z2 không có nghi m nguyên dương. Chŕng minh. Giả sử trái lại, phương trình đã cho có nghi m nguyên dương. Goi (x0, y0) là nghi m nguyên dương sao cho tong x là nhỏ nhat. L p lu n tương tụ như trong chứng minh Định lí 2.3.4 ta thay (x0,y0) = 1. Như v y (x2 ,y2 ,z0) là m t 0 0 b ba so Pythagoras nguyên thuý. Không làm giảm tong quát ta giả sử x0 lẻ. Theo Định lí 2.3.3 ton tại m, n vói (m,n) = 1 sao cho , x = m2 −n2 , y = 2mn,z0 = m2 +n2 . (2.24) Vì (m,n) = 1 nên từ (2.24) ta có (x0,n,m) là m t b ba so Pythagoras nguyên thuý. V y lại theo Định lí 2.3.3 ton tại a, b vói (a,b) = 1 sao cho , x0 = a2 − b2 , n = 2ab,m = a2 + b2 . (2.25) Ta có y2 s = 2mn = 4abm suy ra abm = y2 (ỏ đây y0 = 2y1) suy ra a = a2 , b = b2 , 0 1 1 1 m = m2 . Thay vào (2.25) ta có m2 = a4 +b4 . M t khác 1 1 1 1 a4 +b4 = m2 = m < m2 +n2 = z0z2 = x4 +y4 . 1 1 1 0 0 0 Đieu này trái vói cách chon (x0,y0). V y bài toán đưoc chứng minh. Nh n xét 2.3.8. Qua chứng minh 2.3.4. và lòi giải của các Bài toán 2.3.5, 2.3.6, 2.3.7, ta có the nh n thay m t phương pháp chung sau đây trong lí thuyet phương trình Diophantine: Đe chứng minh m t phương trình Diophantine đã cho không có nghi m nguyên dương, ta hãy giả sử nó có và khi đó sẽ có m t nghi m nhỏ nhat (theo m t nghĩa nào đó). Sau đó ta sẽ co gang kien thiet m t nghi m nhỏ hơn nghi m nhỏ nhat và như v y ta dȁn đen mâu thuȁn.
  • 45. 45 Viết đề tài giá sinh viên – ZALO:0973.287.149-TEAMLUANVAN.COM Chương 3 Liên phân so và ng dụng trong phương trình Diophantine Như chúng ta đã biet, vói m t so thục cho trưóc, ta có the bieu dien nó theo nhieu cách, chȁng hạn bieu dien nó như là tích của các so nguyên to (Định lí cơ bản của Lý thuyet so), hay viet nó theo các h cơ so khác nhau. Trong các cách bieu dien ay, ngưòi ta quan tâm đ c bi t đen các liên phân so, hay còn goi là các phân so liên tnc (continued fractions). Các liên phân so cho phép ngưòi ta bieu dien các so hữu tý và vô tý thành các phân so nhieu tang. Liên phân so là m t chủ đe rat r ng lón và có nhieu ứng dụng đa dạng trong Lý thuyet so. Chương này dành đe trình này m t cách ngan gon các sụ ki n ve liên phân so và đ c bi t là ứng dụng của chúng đe giải phương trình Pell. 3.1 Liên phân so h u hạn Định nghĩa 3.1.1. Liên phân so hay phân so liên tnc là m t bieu thức có dạng 1 a0 + a1 + 1 1 a2 + ··· + 1 ai−1 + an trong đó a0,a1,...,an là các so thục và a1,...,an đưoc hi u là [a0,a1,...,an]. 0. M t liên phân so như v y
  • 46. 46 Viết đề tài giá sinh viên – ZALO:0973.287.149-TEAMLUANVAN.COM b + Từ định nghĩa de thay 1 [a0,a1,...,ak+1] = a0 + [a0,a1,..., ak 1] . Neu a0 ∈ Z và a1,...,an là các so nguyên dương thì ta nói [a0;a1,...an] là m t liên phân so hru hạn có đ dài n. Rõ ràng m t liên phân so hữu hạn là m t so hữu tý. Ngưoc lại ta có: Định lí 3.1.2. Mői so hru tý đeu có the bieu dien dưới dạng m t liên phân so hru hạn. Chŕng minh. Giả sử x = a trong đó a, b ∈ Z và b > 0. Đ t r0 = a, r1 = b. Thu t chia Euclide cho ta r0 = r1q1 +r2, 0 < r2 < r1, r1 = r2q2 +r3, 0 < r3 < r2, ... rn−2 = rn−1qn−1 + rn, 0 < rn < rn−1, Từ đó de thay rn−1 = rnqn. a b = [q1;q2,...,qn]. Ta có đieu phải chứng minh. Cho liên phân so hữu hạn [a0;a1,...,an]. Vói mői k ≤ n liên phân so Ck = [a0;a1,...,ak] goi là gián phân thŕ k của [a0;a1,...,an]. Công thức tính các giản phân đưoc cho bỏi định lí sau. Định lí 3.1.3. Cho liên phân so hru hạn [a0;a1,...,an]. Giá sr dãy so nguyên dương p0, p1,..., pn và q0,q1,...,qn được xác định truy hoi như sau p0 = a0, q0 = 1,
  • 47. 47 Viết đề tài giá sinh viên – ZALO:0973.287.149-TEAMLUANVAN.COM ak+1 k a k a p1 = a0a1 +1 q1 = a1, ... pk = ak pk−1 + pk−2, qk = akqk−1 + qk−2. Khi đó gián phân thŕ k là Ck = [a0;a1,...,ak) được cho bới Ck = pk . qk Chŕng minh. Ta chứng minh bang quy nạp. Vói k = 0 ta có C0 = [a0] = p0/q0. Vói k = 1 ta có C1 = [a0;a1] = a0 + 1 a1 = a0a1 +1 = p1/q1. a1 Giả sử định lí đúng cho moi 0 ≤ k < n. Khi đó vói 2 ≤ k < n, Ck = [a0;a1,...,ak] == pk = ak pk−1 + pk−2 . V y qk akqk−1 + qk−2 Ck+1 = [a0; a1,...,ak,ak 1] = a0;a1,...,ak − 1,ak + 1 (a + 1 k+1 (a + 1 k+1 )pk−1 + pk−2 )qk−1 + qk−2 = ak+1(ak pk−1 + pk−2) + pk−1 ak+1(akqk−1 + qk−2) + qk−1 = ak+1 pk + pk−1 = pk+1 . Định lí đưoc chứng minh. ak+1qk + qk−1 qk+1 Bang phương pháp quy nạp ta de dàng chứng minh đưoc đȁng thức quan trong sau giữa các (pk) và (qk). Định lí 3.1.4. pkqk−1 − pk−1qk = (−1)k−1 . + =
  • 48. 48 Viết đề tài giá sinh viên – ZALO:0973.287.149-TEAMLUANVAN.COM − k k−2 H qua 3.1.5. (pk,qk) = 1. Tiep theo, ta sẽ mô tả các quan h của giản phân. Định lí 3.1.6. Giá sr (Ck) là dãy gián phân của liên phân so hru hạn [a0;a1 ...,an]. Ta có Ck −Ck−1 = Ck −Ck−2 = (−1)k−1 qkqk−1 ak(−1)k qkqk−2 , 1 ≤ k ≤ n, , 2 ≤ k ≤ n. Chŕng minh. Vói đȁng thức thứ nhat ta có pkqk−1 − pk−1qk (−1)k−1 Ck −Ck−1 = qkqk = q q . Vói đȁng thức thứ hai ta có −1 k k−1 C C = pkqk−2 − pk−2qk . qkqk−2 Thay pk = ak pk−1 + pk−2, qk = akqk−1 = qk−2 vào tử so và áp dụng đȁng thức thứ nhat ta thu đưoc đieu phải chứng minh. Từ định lí trên ta thu đưoc ket quả quan trong sau. Định lí 3.1.7. Ta có C1 > C3 > C5 > ..., C0 < C2 < C4 < ... Hơn nra mői gián phân lé C2 j−1 đeu lớn hơn mői gián phân chȁn C2i. Chŕng minh. Từ định lí trên ta thay neu k lẻ thì Ck < Ck−2 và neu k chȁn thì Ck > Ck−2. Cũng theo định lí trên, ta có) (−1)2m−1 C2m −C2m−1 = q2mq2m < 0 suy ra C2m < C2m−1. − V y C2 j−1 > C2 j−1+2i > C2 j+2i > C2i. 1
  • 49. 49 Viết đề tài giá sinh viên – ZALO:0973.287.149-TEAMLUANVAN.COM + + − 3.2 Liên phân so vô hạn Như ket quả mục trên, ta biet rang mői so hữu tý sẽ đưoc bieu dien m t cách duy nhat bang m t liên phân so hữu hạn. Chuyen sang tình huong các so vô tý, ta sẽ thay, mői so vô tý sẽ đưoc bieu thị duy nhat dưói dạng m t liên phân so vô hạn. Định lí 3.2.1. Cho a0, a1, a2 . . . là dãy vô hạn các so nguyên với ai > 0 với i ≥ 1. Đ t Khi đó ton tại giới hạn Ck = [a0; a1,...,ak]. lim Ck = α. k→∞ Ta goi α là giá trị của liên phân so vô hạn [a0;a1;a2 ...] và viet α = [a0; a1; a2,...]. Chŕng minh. Theo Định lí 3.1.7 ta có C1 > C3 > C5 > ... > C2n−1 > C2n+1 > ... C0 < C2 < C4 < ... < C2n−2 < C2n < ... Hơn nữa dãy (C2k+1) là dãy giản và bị ch n dưói bỏi C0 còn dãy (C2k) tăng và bị ch n trên bỏi C1. V y ton tại lim C2k+1 = α1 và lim C2k = α2. k→∞ k→∞ Ta can chứng minh α1 = α2. Th t v y theo Định lí 3.1.7 ta có C2k 1 +1 −C2k = q2k 1q2k . Bang quy nạp) , ta có qk ≥ k. Do đó lim C2k 1 C2k = 0. k→∞ V y α1 = α2. Định lí đưoc chứng minh.
  • 50. 50 Viết đề tài giá sinh viên – ZALO:0973.287.149-TEAMLUANVAN.COM + q2 q n+1 2 n + ⇔ α − q2n ⇔ α − q2n Ket lu n quan trong cho ứng dụng là Định lí 3.2.2. α = [a0;a1;a2,...] là m t so vô tý. Chŕng minh. Ta sẽ chứng minh bang phản chứng. Giả sử trái lại α = a/b ∈ Q. Theo Định lí 3.1.7 ta có C2n < α < C2n+1. V y 0 < α −C2n < C2n 1 −C2n = 1 . Đieu này tương đương vói 0 < α − p2n < 1 q2n q2n+1q2n 1 ⇔ 0 < αq2n − p2n < q2n 1 1 0 < q2n bp2n < +1 1 1 < q2n bp2n < . +1 Cho k → ∞ ta có đieu mâu thuȁn. Như v y phép chứng minh đưoc ket thúc. Định lí 3.2.3. Mői so vô tý đeu bieu dien m t cách duy nhat dưới dạng m t liên phân so vô hạn. 3.3 Liên phân so vô hạn tuan hoàn Ta goi liên phân so vô hạn [a0; a1, a2, . . .] là tuan hoàn neu dãy (an)là tuan hoàn ke từ m t chỉ so nào đó tức là: ton tại so nguyên dương m và k vói moi n ≥ m ta có an = an+k. So nguyên dương k đưoc goi là chu kỳ. Trong trưòng hop đó ta viet [a0;a1,a2,...,am−1,am,am+1,...,am+k−1] Bài toán đ t ra là đ c trưng tat cả các so vô tý có bieu dien liên phân so vô hạn tuan hoàn. Ta có khái ni m sau
  • 51. 51 Viết đề tài giá sinh viên – ZALO:0973.287.149-TEAMLUANVAN.COM Định nghĩa 3.3.1. So vô tý α đưoc goi là so vô tý b c hai neu nó là nghi m của m t tam thức b c hai vói h so nguyên. Ví dụ 3.3.2. So vô tý α = 2+ √ 3 là so vô tý b c hai vì nó là nghi m của x2 −4x+ 1 = 0. Bo đe 3.3.3. So thực α là vô tý b c hai neu và chỉ neu có ton tại các so nguyên a,b,c với b > 0 và không chính phương, c /= 0 sao cho a + √ b a = c . Chŕng minh. Giả sử α là so vô tý b c hai. Khi đó ton tại các so nguyên A,B,C sao cho α là nghi m của phương trình Ax2 +Bx+C = 0. V y α == −B ± √ B2 − AC 2A Đ t a = −B, b = B2 − 4AC, c = 2A ho c a = B, b = B2 −4AC, c = −2A ngưoc lại neu α = a + √ b . c Thì α là so vô tý và nó là nghi m của phương trình b c hai c2 x2 − 2acx + a2 − b = 0. Bo đe 3.3.4. Neu α là so vô tý b c hai thì (rα +s)/(tα +u) cũng là so vô tý b c hai neu r,s,t,u là các so nguyên. Chŕng minh. Giả sử α = a + √ b . c Tính toán cho ta rα +s = (ar +c)(at + cu)−rtb+ (r(at +cu)−t(ar +cs)) √ b . tα + u (at + cu)2 − tb2 .
  • 52. 52 Viết đề tài giá sinh viên – ZALO:0973.287.149-TEAMLUANVAN.COM Định nghĩa 3.3.5. So vô tý α = a − √ b . c Đưoc goi là liên hop của α và ký hi u là α′. Bo đe 3.3.6. Neu so vô tý b c hai α là nghi m của phương trình Ax2 +Bx +C = 0 thì liên hợp của nó cũng là nghi m của phương trình đó. Chŕng minh. Th t v y α + α′ = 2a/c = −BA, (α)(α′) = a2 − b/c2 = C/A. Bang phép tính ta de thay Bo đe 3.3.7. Ta có các h thŕc sau: (α ±β)′ = α′ ±β′, (αβ)′ = α′β ′, α ′ = α′ . Ta có định lý cơ bản sau đây do Lagrange tìm ra Định lí 3.3.8. So vô tý α có bieu dien liên phân so tuan hoàn khi và chỉ khi nó là so vô tý b c hai. Chŕng minh. Trưóc het ta chứng minh rang neu α có bieu dien liên phân so tuan hoàn thì nó là so vô tý b c hai. Giả sử Đ t [a0;a1,a2,...,am−1,am,am+1,...,am+k−1] β = [am,am+1,...,am+k−1] β β′
  • 53. 53 Viết đề tài giá sinh viên – ZALO:0973.287.149-TEAMLUANVAN.COM y Khi đó β = [am,am+1,...,am+k,β] do đó β = βPk + pk−1 . (3.1) βPk + pk−1 trong đó pk/qk và pk−1/qk−1 là hai giản phân cuoi của [am, am+1, . . . am+k]. Từ công thức (3.1) suy ra qkβ2 + (qk−1 − pk)β − pk−1 = 0. V y β là so vô tý b c hai. Ta lại có α = [a0;a1;a2,...,am−1,β] do đó α = β pm−1 + pm−2 . βqm−1 + qm−2 Do đó theo Bo đe 3.3.6 ta có α là so vô tý b c hai. Ví dụ sau đây minh hoạ cách tìm so vô tý b c hai từ bieu dien liên phân so tuan hoàn của nó. Ví dụ 3.3.9. Tìm x biet rang x = [3;1,2]. Ta có x = [3;y] vói y = 1,2. Ta có y = [1;2,y] do đó 1 = 1 + 2 + 1 3y + 1 = 2y +1 . Suy ra 2y2 − 2y − 1 = 0. Vì y > 0 nên y = (1 + √ 3)/2. Vì x = 3 + 1/y nên ta tìm đưoc 2 4+ √ 2 x = 3 + 1 + √ 2 = 2 . Đe chứng minh phan ngưoc lại ta can bo đe sau Bo đe 3.3.10. Neu α là so vô tý b c hai thì nó có the bieu dien dưới dạng P+ √ d α = Q trong đó P,Q,d là các so nguyên sao cho Q | (d −P2 ).
  • 54. 54 Viết đề tài giá sinh viên – ZALO:0973.287.149-TEAMLUANVAN.COM Chŕng minh. Ta có α = (a + √ b)/c. Nhân cả tử và mȁu vói |c| ta đưoc α = (a|c|+)/c|c| . Đ t P = a|c|, d = bc2 , Q = c|c| = ±c2 . Khi đó d − P2 = c2 (b − a2) chia het Q = ±c2 . Giả sử α = α0 là so vô tý b c hai. Ta xây dụng dãy (a0, a1, a2 . . .) như sau. Theo Bo đe 3.3.10 ta có các so nguyên P0,Q0 và d sao cho α0 = (P0 + √ d)/Q0. Q0 | (d − P). Ta đ t a0 = [α0] và xác định P1 = a0Q0 − P1,Q1 = (d − P)/Q0, α1 = (P1)/Q1. Tiep đó đ t a1 = [α1]. M t cách tong quát neu có Pk + √ d Pk ∈ Z, Qk ∈ Z,Qk | (d −P), ak = Qk ak = [αk]. Ta sẽ đ t Pk+1 = akQk − Pk, Qk+1 = (d −P)/Qk, ak+1 = ak+1 = [αk+1]. Khi đó tính toán cho thay sQk+1 = (d −P)/Qk +(2kPk −aQk) do đó Qk+1 ∈ Z và vì Qk+1Qk = (d − P) nên Qk+1 | (d − P). Có the chứng minh đưoc rang α = [a0,a1,a2,...] và hơn nữa dãy (an) xác định như trên là tuan hoàn. Ví dụ 3.3.11. Khai trien liên phân so của so α = (6+ √ 28)/4. Chŕng minh. Ta có P0 = 6, Q0 = 4, d = 28, 4 | (28−62 ) = −8, a0 = (6+ √ 28)/4, a0 = [α0] = 2 và P1 = 2.4−6 = 2, Q1 = (28−22 )/4 = 6, α1 = (2+ √ 28)/6, a1 = [α1] = 1 P2 = 1.6 − 2 − 4 = 4, Q2 = (28 − 42 )/6 = 2,α2 = (4 + √ 28)/2,a2 = [α2] = 4 P3 = 4.2 − 4 = 4,Q3 = (28 − 42 )/2 = 6,α3 = (4 + √ 28)/6,a3 = [α3] = 1 P4 = 1.6 − 4 = 2,Q4 = (28 − 22 )/6 = 4,α4 = (2 + √ 28)/4,a4 = [α4] = 1
  • 55. 55 Viết đề tài giá sinh viên – ZALO:0973.287.149-TEAMLUANVAN.COM P5 = 1.4 − 2 = 2,Q5 = (28 − 22 )/6 = 4,α5 = (2 + √ 28)/4,a5 = [α4] = 1 Ta thay P1 = P5, Q1 = Q5 do đó a1 = a5 và dãy tuan hoàn chu kỳ 4. Ta có 6+ √ 28 4 = [2;1,4,1,1,] Tiep theo ta muon tìm đieu ki n đe so vô tý b c hai có bieu dien liên phân so tuan hoàn ngay từ đau, tức là đieu ki n đe ton tại so nguyên dương k sao cho an = an+k vói moi n ≥ 0. Ta có định lý sau Định lí 3.3.12. So vô tý b c hai α có bieu dien tuan hoàn ngay tr đau neu và chỉ neu α > 1 và −1 < α′ < 0. Chứng minh định lý này khá phức tạp nên ta bỏ qua. Bây giò ta sẽ xác định bieu dien liên phân so của √ d. Xét so α = √ d + [ √ d]. Ta có α′ = [ √ d] − √ d do đó α > 1 và −1 < α′ < 0. V y α có bieu dien tuan hoàn ngay từ đau. So hạng đau tiên a0 = [ √ d + [ √ d]] = 2[ √ d] = 2a vói a = [ √ d]. Ta có √ d +a = √ d + [ √ d] = [2a;a1,a2,...,an] = [2a; a1,a2,... an,2a; a1,a2,...,an] Suy ra √ d = [a; a1,a2,..., an,2a]. Phân tích can th n hơn ta còn có the chứng minh đưoc a1 = an, a2 = an−1,... tức là dãy (a1,...,an) đoi xứng, tức là nó có dạng √ d = [a;a1;a2,...,a2,a1,2a] ỏ đó a = [ √ d].
  • 56. 56 Viết đề tài giá sinh viên – ZALO:0973.287.149-TEAMLUANVAN.COM 3.4 Áp dụng vào phương trình Diophante 3.4.1 Phương trình b c nhat hai an Ax+ By = C Chúng ta biet rang phương trình có nghi m neu và chỉ neu d = (A,B) là ưóc của C. Trong trưòng hop này giả sử A = ad, B = bd, C = cd thì (a,b) = 1 và phương trình đã cho tương đương vói ax + by = c. (3.2) Neu (x0, y0) là m t nghi m của (3.2) thì tat cả các nghi m (x, y) của (3.2) đưoc cho bỏi công thức x = x0 + bt; y = y0 − at. Như v y vi c giải phương trình (3.2) quy ve tìm m t nghi m (x0,y0) của nó. Xét phương trình ax + by = 1. (3.3) Neu (x0,y0) là m t nghi m của (3.3) thì (cx0,cy0) là nghi m của (3.2). Thành thử ta quy ve bài toán: Cho (a,b) = 1. Hãy tìm m t nghi m của phương trình (3.3). Ta bieu dien so a/|b| thành liên phân so hữu hạn a |b| = [a0;a1;a2;......... ; an]. Goi pn−1/qn−1 và pn/qn là hai giản phân cuoi cùng của liên phân so này. Ta có a/|b| = pn/qn, (a, b) = 1, (pn, qn) = 1 nên a = pn, |b| = qn. Theo Định lý 3.3.8 ta có pnqn−1 − pn−1qn = (−1)n−1 suy ra aqn−1 − |b|qn−1 = (−1)n−1 . Đieu này kéo theo a(−1)n−1 qn−1 +|b|(−1)n−1 pn−1 = 1. V y, neu b > 0 thì phương trình (3.3) có m t nghi m là x = (−1)n−1 qn−1, y = (−1)n pn−1. Neu b < 0 thì phương trình (3.3) có m t nghi m là x = (−1)n−1 qn−1, y = (−1)n−1 pn−1.
  • 57. 57 Viết đề tài giá sinh viên – ZALO:0973.287.149-TEAMLUANVAN.COM d −P Qk k k Ví dụ 3.4.1. Giải phương trình 342x − 123y = 15. Lời giái. Vì (342,123) = 5 nên phương trình đã cho tương đương vói 114x−41y = 5. Ta bieu dien so 114/41 thành liên phân so. Ta có 114 = 2 · 41 + 32 41 = 1 · 32 + 932 = 3 · 9 + 5 9 = 1 · 5 + 45 = 1 · 4 + 1 4 = 4 · 1. Do v y 114 41 = [2;1,3,1,1,4]. Ta có n = 5, p4/p4 = [2; 1, 3, 1] = 25/9. Vì b = −41 < 0 nên m t nghi m của phương trình 114x − 41y = 1 là x = q4 = 9, y = 25. Suy ra m t nghi m của phương trình 114x − 41y = 5 là x = 5 · 9 = 54, y = 5 · 25 = 125. Nghi m tong quát của phương trình đã cho là: x = 45 + 41t, y = 125 + 114t vói .t ∈ Z 3.4.2 Phương trình x2 − dy2 = ±1 Nhac lại, ta goi phương trình Pell là các phương trình có dạng x2 − dy2 = ±1. Bo đe 3.4.2. Cho d là so không chính phương. Giá sr Pk, Qk, αk, ak là các so xác định trong vi c tìm khai trien liên phân so của √ d Pk + √ d αk = Qk , ak = [αk], Pk+1 = akqk − pk, Qk+1 = 2 k+1 . Qk αk+1 Pk+1 + √ d = , +1 k+1 = [αk+1]. Giá sr pk/qk là gián phân thŕ k của √ d. Khi đó p2 − dq2 = (−1)k−1 Qk+1. a
  • 58. 58 Viết đề tài giá sinh viên – ZALO:0973.287.149-TEAMLUANVAN.COM Q 2 2tr−1 k k Chŕng minh. Vì √ d = α0 = [a0;a1,a2,...,ak,αk+1] nên ta có √ d = α0 = αk+1 pk + pk−1 . αk+1qk + qk−1 Vì αk+1 = Pk+1+ √ d ta có k+1 √ (Pk+1 + √ d)pk +Qk+1 pk−1 Do đó d = (Pk+1 + √ d)pk + Qk+1 . qk−1 npk = (Pk+1qk + Qk+1qk−1) √ d = (Pk+1 pk + Qk+1 pk−1) + pk √ d. Từ đó suy ra (do √ d /∈ Q) nqk = Pk+1 pk + Qk+1 pk−1, Pk+1qk + Qk+1qk−1 = pk. Từ đó (nhân phương trình đau vói qk, phương trình thứ hai vói pk, roi trừ cho nhau ta đưoc. p2 − dq2 = (pkqk−1 − pk−1qk)Qk−1 = (−1)k−1 Qk+1. Ta có đieu phải chứng minh. Định lí 3.4.3. Giá sr chu kỳ của bieu dien liên phân so của √ d là r. Goi pk/qklà gián phân thŕ k của √ d. Neu r chȁn thì x = ptr−1, y = qtr−1 với t = 1,2... là nghi m của phương trình Pellx2 − dy2 = 1. Neu r lé thì x = p2tr−1, y = q2tr−1, với t = 1,2,... là nghi m của phương trình Pell x2 −dy2 = 1. Chŕng minh. Vì √ d = 0 + √ d/1 nên Q0 = 1, suy ra Qkr = Q0 = 1 vói moi k. Theo Bo đe 3.4.2 ta có 2 kr−1 — dqkr−1 = (−1)kr−2 Qkr = (−1)kr . Như v y neu n chȁn thì p2 kr−1 2 kr−1 = 1 vói moi k ∈ N, neu r lẻ thì p2 − 2 2tr−1 = 1. p — dq dq
  • 59. 59 Viết đề tài giá sinh viên – ZALO:0973.287.149-TEAMLUANVAN.COM r 1 . . α − . < . 1 ≤ |spk − rqk| = . pk − r. . qk 1 . s . 1 Bo đe 3.4.4. Ta có Qi cho r. 1 với moi i = 1,2... và Qk = 1 khi và chỉ khi k chia het Đe chứng minh đây là tat cả các nghi m của phương trình Pell ta can các bo đe sau Bo đe 3.4.5. Cho α là m t so vô tý và r/s là so hru tý toi gián với r > 0 và . s . 2s2 Khi đó r/s phái là m t gián phân của α. Chŕng minh. Giả sử r/s không là giản phân khi đó ton tại k sao cho qk ≤ s < qk+1. Theo Bo đe 3.4.2 ta có |qkα − pk| ≤ |sα − r| = s|α − r/s| < 1/(2s). Suy ra Vì |spk − rqk| ≥ 1 nên ta có |α − pk/qk| < 1/(2sqk). sqk sqk .qk s. = .α − k .+ . α + . < 2sqk + 2s2 . V y 1 < 1 , tức là qk > s. Đieu mâu thuȁn này đã chứng minh phát bieu của bo 2sqk 2s2 đe là đúng. Bo đe 3.4.6. Giá sr x, y là các so nguyên dương sao cho x2 −dy2 = n và |n| < √ d. Khi đó x/y là m t gián phân của √ d. p r
  • 60. 60 Viết đề tài giá sinh viên – ZALO:0973.287.149-TEAMLUANVAN.COM y i i Chŕng minh. Xét trưòng hop n > 0. Ta có (x + y √ d)(x − y √ d) = n suy ra x > y √ d. V y 0 < x − √ d. Ta có Lại có x √ x − √ d x2 − dy2 n √ d 1 y − d = y = y(x + y √ d < y(2y √ d < 2y2 √ d = 2y2 . Theo Bo đe 3.4.5 thì x/y là m t giản phân của √ d. Giả sử n < 0. Khi đó y2 − (1/d)x2 = −n/d. Ta có −n/d > 0, −|n|/d < 1/ √ d. V y theo bưóc trưóc y/x là m t giản phân của 1/ √ d. Nhưng khi đó x/y = 1/(y/x) là m t giản phân của 1/(1/( √ d) = √ d. Phép chứng minh định lí đưoc ket thúc. Định lí 3.4.7. Cho phương trình Pell x2 − dy2 = 1. Goi r là chu kỳ của bieu dien liên phân so của √ d. Neu r chȁn thì tat cá các nghi m của phương trình Pell là x = pkr−1, y = qkr−1. Neu r lé thì tat cá các nghi m của phương trình Pell là x = p2tr−1, y = q2tr−1 với t ∈ N∗. Chŕng minh. Giả sử (x,y) là nghi m của phương trình Pell. Theo Bo đe 3.4.6, ton tại i đe x = pi, y = qi. Từ đó p2 − dq2 = 1. Từ Bo đe 3.4.2 rút ra (−1)i−1 Qi+1 = 1. Suy ra Qi+1 = ±1. Vì qk+1 /= −1 nên Qi+1 = 1 và i lẻ. Theo Bo đe 3.4.4 ta rút ra ton tại ki + 1 = kr, kéo theo i = kr − 1 vàkr chȁn. Thành thử neu r lẻ thì k chȁn, k = 2t. Phép chứng minh định lí đưoc ket thúc.
  • 61. 61 Viết đề tài giá sinh viên – ZALO:0973.287.149-TEAMLUANVAN.COM i i Xét phương trình Ta có ket quả sau x2 − dy2 = −1 (3.4) Định lí 3.4.8. Phương trình x2 − dy2 = −1 có nghi m khi và chỉ khi chu kỳ r của bieu dien liên phân so của √ d là so lé. Trong trường hợp ay các nghi m của nó là x = p(2tr−r−1), y = q(2tr−r−1) với t = 1,2,.... Chŕng minh. Từ Bo đe 3.4.2 de thay neu chu kỳ r của bieu dien liên phân so của √ d là so lẻ thì x = p(2tr−r−1) , y = q(2tr−r−1) vói t = 1,2,... là nghi m. Giả sử (x,y) là nghi m của phương trình (3.4). Theo Bo đe 3.4.6 ton tại i đe x = pi, y = qi. Từ đó p2 − dq2 = −1. Từ Bo đe 3.4.2 ta rút ra (−1)i−1 Qi+1 = −1, suy ra Qi+1 = ±1. Vì Qi+1 /= −1 nên Qi+1 = 1 và i chȁn. Theo Bo đe 3.4.2 ton tại k ∈ N sao choi + 1 = kr, suy ra i = kr − 1 và kr lẻ. Thành thử neu r chȁn thì kr luôn chȁn do đó phương trình vô nghiêm. Trong trưòng hop r lẻ lý lu n tương tụ như trong trưòng hop phương trình Pell x2 − dy2 = 1 tat cả các nghi m phải có dạng x = pkr−1, y = qkr−1 trong đó kr lẻ tức là khi k lẻ hay x = p(2tr−r−1), y = q(2tr−r−1) vói t = 1, 2, . . . Phép chứng minh định lí đưoc ket thúc.
  • 62. 62 Viết đề tài giá sinh viên – ZALO:0973.287.149-TEAMLUANVAN.COM Ket lu n 1 Nh ng ket qua đã đạt đư c Lu n văn “M t so lớp phương trình Diophantine” đã đạt đưoc các ket quả sau: 1. Trình bày các kien thức cơ sỏ ve phương trình Diophantine tuyen tính (b c nhat hai an và nhieu an). 2. Trình bày ve m t so phương trình Diophantine phi tuyen, như Phương trình Pell loại 1, loại 2, phương trình Pell tong quát, Phương trình Pythagoras- Fermat. 3. Úng dụng của liên phân so trong phương trình Diophantine. 2 Đe xuat m t so hưỚng nghiên c u tiep theo Phương trình Diophantine là m t trong những chủ đe rat r ng của Lý thuyet so. Sau những ket quả đã đạt đưoc trong lu n văn, chúng tôi hi vong và co gang sẽ tiep tục nghiên cứu các chủ đe liên quan, chȁng hạn: • Các phương pháp và kỹ thu t khác đe tan công các phương trình Diophan- tine, ví dụ phương pháp xuong thang, các phương pháp so hoc, đại so và hình hoc, ... • Nghiên cứu ve các các lóp phương trình Diophantine phi tuyen khác như phương trình có chứa hàm mũ.
  • 63. 63 Viết đề tài giá sinh viên – ZALO:0973.287.149-TEAMLUANVAN.COM Tài li u tham khao Tieng Vi t [1] A.D. Aczel (2001), Câu chuy n hap dȁn ve bài toán Ferma (Ngưòi dịch: Tran Văn Nhung, Đő Trung H u, Nguyen Kim Chi), NXB Giáo dục. [2] Vũ Ngoc Khánh (2015), M t so van đe ve phương trình Diophantine, Lu n văn thạc sĩ toán hoc, [3] Hà Huy Khoái (2004), So hoc, NXB Giáo dục. [4] Đ ng Hùng Thang (2010), Bài giáng so hoc, NXB Giáo dục. Tieng Anh [5] K.H. Rosen (1986), Elementary Number Theory and its Applications, Addison–Wesley Publishing Company. [6] W. Siepinski (1964), Elementary Theory of Number, North-Holland Mathe- matical Library (volume 31).